You are on page 1of 111

Tng hp p n thi th s 14 ca math.

vn
Cu I.1)
Hm s y = x3 2009x c th (C)
Cu I.2) supervirus
Gi Mi (xi ; yi ) (C) yi = x3i 2009xi .
Ta c 2009xn + yn = 22010 x3n = 22010 xn = (2)670 (1) .
Mt khc tip tuyn ti Mi l
2
i : y = (3xi 2009)(x xi ) + yi Mi+1 = i (C) c honh xi+1 l nghim PT
(3x2i 2009)(x xi ) + yi = x3 2009x x = xi+1 = 2xi (Do xi+1 6= xi )
Thnh th xn = (2)n1 .x1 (2) (1) & (2) 1 = (2)671n n = 671
Cu II.1) supervirus
K: sin x 6= 0 P T 2 sin2 x(1 + cos 2x) = 2 sin x cos x(2sin2 2x + 2 cos 2x) + cos 2x
4 sin2 x cos2 x = 2 sin 2x(cos 2x sin2 x) + cos 2x
sin2 2x(1 + 2 sin 2x) = cos 2x(1 + 2 sin 2x)
1
sin2 2x = cos 2x
sin 2x =
2

1
1 + 5
sin 2x =
cos 2x =
2
2

7
1
1 + 5
x=
+ k x =
+ k x =
arccos
+ k
12
12
2
2
Cu II.2) up_pixar
x2 y 4 + 9y= x(9 + y y 3 ) (x y)(x
+ y 3 9) = 0

3
3
V y < 1 v 1 + x + 1 y = 2 nn 1 + x < 2 x < 7.
Do x + y 3 9 < 1 < 0.
Nn x =y.

3
3
1
+
x
+
1

x
=
2.
t
a
=
1
+
x;
b
=
1 x (b > 0) th
Th
vo
pt
ban
u
c
(
a+b=2
a3 + (2 a)2 = 2 a3 + a2 4a + 2 = 0 (a 1)(a2 + 2a 2) = 0
a3 + b 2 = 2

T suy ra nghim ca pt ban u x = 0; x = 11 + 6


3; x = 11 6 3
Vy h cho c 3 nghim l x = y = 0; x = y = 11 + 6 3; x = y = 11 6 3
Cu III.) up_pixar

T din S.ABC c trng tmG


SG ct mp(ABC) ti O th O l trng tm tam gic ABC suy ra SOAB = SOBC = SOAC nn
SM
SN
SP
1
;y =
;z =
VS.OAB = VS.OBC = VS.OAC = VS.ABC t x =
3
SA
SB
SC
Hnh chp S.M N G c cc nh tng ng thuc cc cnh ca hnh chp S.ABO nn
VSM N G
SO SM SN
3
=
.
.
= xy (do G l trong tm t din SABC)
VSABO
SG SA SB
4
VSM N G
xy
VSN P G
yz VSM P G
xz
VSM N P
Suy ra
=
Tng t ta c
= ;
=
m
= xyz
VSABC
4
VSABC
4 VSABC
4
VSABC
p
xy + yz + xz
27
nn xyz =
p dng BDT C-si c xy + yz + xz 3 3 x2 y 2 z 2 nn xyz
4
64
27
Suy ra th tch t din SMNP t GTNN bng VSABC
64
3
Du bng xy ra khi v ch khi x = y = z = . Khi mt phng () qua G song song vi mt
4
phng ABC theo nh l Thalets
Cu IV.) Hong H

ln



Z 3
1
1
1
v2 1
1
x
x
2x
C I =
1
+
e
de
t
e
=
v

=
th:
+
1
.
ex
ex
2
v
2v
0

2
3
s

2


Z 
2v
1
1
v
1
+1
1+
d v
I=
v
v2 1
4
v
v2 1
v
1
=
2
=

1
2

2
Z 3

2
Z 3

v 2 + 2v 1
+1
v2 1



v2 + 1
v



v2 + 1
v2




v
dv
v2 1

(v 2 + 2v 1)(v 2 + 1)2
dv = ....
v 2 (v 2 1)2

Cu V) up_pixar
x+1
y+1
z+1
Chia c 2 v iu kin ban u cho 4 v t
= a;
= b;
=c
7
7
7
4
4
4
a+b+c
1
1
. Khi A = 4
.
th a7 + b7 + c7 = 3 v a, b, c
7
7
4
4
a +b +c
4
43
r
1
1 7 5
7
4
Mt khc vi cc s a, b, c
. Xt hm s f (x) = 3x 7x + 7x 3 vi x [
;
]
7
7
2
4
4
f 0 (x) = 7(3x6 4x3 + 1) = 7(x3 1)(3x3 1)
Lp bng bin thin c minf (x) = 0 t c khi x = 1
3x7 3
1
Do x4
+ x (iu ny ch ng khi c x >
)
7
7
4
1
1
=
Thay x = a, b, c ri cng cc BDT c a4 + b4 + c4 a + b + c. Vy A
7
7
64
43

7
Du bng xy ra khi v ch khix = y = z = 4 1
Cu VIa1) up_pixar

1
Gi s OI ct AB ti H th ta c: AB 2 = BH 2 = OB 2 OH 2 = r2 OH 2
4

M theo quan h ng xin- ng vung gc th OH OM = 4 + 1 = 5 khng i


nn BH 2 r2 OM 2 = 9 5 = 4 AB 4 khi tm I thay i
Du = xy ra M trng H hay O, M, I thng hng theo th t
x
Khi tm I thuc ng thng OM c phng trnh y =
2

v OI = OM + M I = 5 + 16 4 = p5 + 2 3

5 + 2 15
85 + 20 15
2
T suy ra 5yI = 17 + 4 15 yI =
=
5
5
(v M nm gia O v I nn I thuc
gc
phn
t
th
IV
!

10 + 4 15 5 + 2 15
;
Vy I
5
5
Cu VIa1) DoBaChu
Ta d dng lp c PT cc mt ca t din nh sau : (ABC) : x + y + z 2 = 0 (ABD) :
x y z + 2 = 0 (ADC) : x y + z 2 = 0 (BCD) : x + y z 2 = 0

D&M cng pha vi(ABC)

C&M cng pha vi(ABD)


im M (x; y; z)(x, y, z Z) nm trong t din ABCD

B&M cng pha vi(ADC)

A&M cng pha vi(BCD)

x+y+z >2
(1)
(2 + 2 + 2 2)(x + y + z 2) > 0

x y z > 2 (2)
(2 0 0 + 2)(x y z + 2) > 0

x y + z < 2

(3)
(0 2 + 0 2)(x y + z 2) > 0

x+yz <2
(4)
(0 + 0 2 2)(x + y z 2) > 0
Cng theo v (1) v (2) , (3) v (4) suy ra 0 < x < 2 x = 1 (v x Z)
Thay x = 1 tr li (1) v (2) thu c 1 < y + z < 3 (5) ,
thay x = 1 vo (3) v (4) thu c 1
( < y z < 1 (6) .
y+z =2
Bi y, z Z nn t (5) v (6) cho ta
y=z=1
yz =0
Vy M (1; 1; 1) l im nguyn duy nht nm trong t din ABCD cho
Cu VIIa) Member_Of_AMC
t z = x + yi, z = x q
yi vi x, y R

q
|z + 2| + |z 2| = 6 (x + 2) + + (x 2)2 + y 2 = 6
q


2 (x2 + 4 + y 2 ) + 2 (x + 2)2 + y 2 (x 2)2 + y 2 = 36
q


(x + 2)2 + y 2 (x 2)2 + y 2 = 14 x2 y 2

q


(x2 4)2 + y 2 (x + 2)2 + (x 2)2 + y 4 = 14 x2 y 2

14 x2 y 2 0

x4 8x2 + 16 + y 2 (2x2
+ 8) + y 4 = 196 + x4 + y 4 28x2 28y 2 + 2x2 y 2
 2
x2 + y 2 14
x + y 2 14

x2 y 2
20x2 + 36y 2 = 180

+
=1
9
5
x2 y 2
Vy tp hp nhng im M l Elip (E) :
+
=1
9
5
v (E) ny nm trong min ca ng trn (C) : x2 + y 2 = 14
2

y2

Cu VIIa) Math213
Gi s M, N l cc im biu din s phc z, z trong mt phng phc v t A(2; 0), B(2; 0).
Khi |z + 2| + |z 2| = BM + AN = M A + M B = 6.
Do tp hp im M l elip vi hai tiu imA, B v c tiu c bng AB = 4.
3

Cu VIb1) hoainamss
Ch cn ch ci tnh cht: 2 ng tip xc vi nhau ti M khi v ch khi tip tip tuyn ca
ng ny ti M cng l tip tuyn ca ng kia ti M .
D dng lp c pt tip tuyn ca th hm s y = x2 ti x = 2 l: y = 4x 4.
Nh vy bi ton chuyn v lp phng trnh ng trn c tm thuc Ox v tip xc vi ng
thng y = 4x 4 ti im M (2; 4).
D roi: Tm I l giao im ca Ox vi ng thng qua M v vung gc vi y = 4x 4.
Cu VIb1) up_pixar
Cc im trn dm c ta tha mn :
x + (1 2m)y + 4mz 4 = 0 v 2x + my (2m + 1)z 8 = 0
Do [x + (1 2m)y + 4mz 4] + 2[2x + my (2m + 1)z 8] = 0
5x + y 2z 20 = 0 l phng trnh mt phng (P ) c vc t php tuyn ~n = (5; 1; 2)
Suy ra dm lun nm trong mp(P) c nh khi m thay i .
2
Vy cc giao im ca h ng thng dm vi mt cu (S) c pt x2 + y
+ z 2 4x + 4y 2z 7 = 0
thuc giao tuyn ca (P) v (S) . (S) c tm I(2; 2; 1) , bn knh R = 4 + 4 + 1 + 7 = 4
Khong cch t I ti mp(P ) l di on IH vi H l hnh chiu ca I trn (P )
|5xI + yI 2zI 20|
14

IH =
= < 4 (P) ct (S) theo giao tuyn l ng trn tm H bn
25 + 1 + 4
30
knh r
r
r

2.49
142
2
2
r = R IH = 16
=
15
15
H l hnh chiu ca I trn (P) nn H thuc ng thng qua I vung gc vi P
x2
y+2
z1
nhn ~n lm vtcp ptdt:
=
=
=t
5
1
2
x2
y+2
z1
H c ta l nghim h phng trnh :
=
=
= t v 5x + y 2z 20 = 0
5
1
2


13 23 1
7
t suy ra H
;
;
Th x,y,z theo t vo pt cui cng ta c 30t = 14 t =
15
3 15 15
Cu VIIb) up_pixar
Xt hm s f (x) = 6x + 7x + 555x2 543x 12x 13x
f 0 (x) = 6x ln 6 + 7x ln 7 + 1110x 543 12x ln 12 13x ln 13
v f 00 (x) = 6x ln2 6 + 7x ln2 7 + 1110 12x ln2 12 13xln2 
13
 x
x
1
1110
7
13
2
2
2
00
Phng trnh f (x) = 0 tng ng vi x ln 6 +
ln 7 +
ln2 13
= ln 12 +
x
2
12
12
12
Ta c v tri ca phng trnh l 1 hm s nghch bin , v phi li l 1 hm s ng bin nn pt
trn c nhiu nht 1 nghim
hm s f 0 (x) c nhiu nht 1 cc tr nn pt f 0 (x) = 0 c nhiu nht 2 nghim
Lp lun tng t ta cng c pt f (x) = 0 c nhiu nht 3 nghim
Mt khc f (0) = f (1) = f (3) = 0 nn pt f (x) = 0 c ng 3 nghim x = 0; x = 1; x = 3
Vy pt ban u c ng 3 nghim: x = 0; x = 1; x = 3

Tng hp p n thi th s 13 ca math.vn


Cu I.1)
Hm s y = x3 x2 + (m 2)x + m + 1 c th (Cm )

n
g

Cu I.2) dungnhi
pt hg x3 x2 + (m 2)x + m + 1 = x (x + 1)(x2 2x + 1 + m) = 0
ycbt tha mn nu nh (x2 2x + 1 + m) = 0 c 2 nghim dng x1 , x2
0 = 1 (1 + m) > 0; S = 2 > 0; P = 1 + m > 0 1 < m < 0
Gi s A(x1 , x1 ), B(x2 ; x2 ) PT ng trn : (x 1)2 + (y + 1)2 = 1
(C)

x 1 = 1 2
(x1 1)2 + (x1 + 1)2 = 1
2
Do A, B (C)

(x2 1)2 + (x2 + 1)2 = 1


x = 1 + 2
2
2
1
m x1 .x2 = m + 1 m =
2
Cu II.1) Nguyen An
k: sin x 6= 0
PT (2 sin x 1) cos2 2x = 2(sin x 1)(2 sin x 1) (2 sin x 1)[cos2 2x + 2(1 sin x)] = 0

5
1
+ k2
TH1: sin x = nhn x = + k2; x =
2
6
6
(
(
2
cos
2x
=
0
1 2 sin2 x = 0

: H v nghim.
TH2: cos2 2x + 2(1 sin x) = 0
sin x = 1
sin x = 1

5
Vy Pt c hai h nghim: x = + k2; x =
+ k2, k Z.
6
6
Cu II.2) supervirus
(p
4 (x2 y 1)2 = y 4 x2 (1 2x2 ) 2
p
Ta c (H)
1 + 1 + (x y)2 = x2 (x4 + 1 2x2 2xy 2 ) 2
T ta c y 4 x2 (1 2x2 ) x2 (2x2 + 2xy 2 x4 1) x6 2x3 y 2 + y 2 0 (x3 y 2 )2 0
x3 y 2 = 0
(3)(
p
4 (x2 y 1)2 = y 4 x2 (1 2x2 ) = 2
p
Vi (3) ta nhn c
x=y=1
1 + 1 + (x y)2 = x2 (x4 + 1 2x2 2xy 2 ) = 2

Cu III.) dungnhi

2
Z 2
Z 2
3
3
sin(x 3 )
dx
1
3
3
= cot(x ) =
I=2

dx = 2
2

3 sin(x 3 ) sin 3(x 3 )


2
3
3
4 sin (x 3 )
2
2
2

Ch

Cu III.) ktmktm

Z 2
3
sin x 3 cos x
 dx
I=

sin 3x + 3 sin x 3
2
t t = x 3 ; dx = dt; x = 2 t = 6 ; x = 2
t = 3
3



t l sin x 3 cos x = 2 sin x cos 3 sin 3 cos x = 2 sin x 3 = 2 sin t

mu l sin 3x + 3 sin x 3 = sin (3t + ) + 3 sin t = 3 sin t sin 3t

m sin 3t = sin (2t+ t) = sin 2t cos t + sin t cos 2t = 2 sin tcos2 t + sin t 1 2sin2 t
= 2 sin t 1 sin2 t + sin t 1 2sin2 t = 3 sin t 4sin3 t 3 sin t sin 3t = 4sin3 t



Z
Z

6
3 2 sin t
1 3 1
1
1
1
3
3 =
Nn I =
dt =
dt = cot t =
3
2

2 6 sin t
2
2
3

4sin t
3
6
3

Cu IV.) dungnhi

K IN ||AB IN (SAC) M N 2 = IN 2 + M I 2
t
D dng tnh ra IN =
2
5t2
2
2
2
o
M I = M A + AI 2AM.AI.cos45 =
4at + 2a2
2
M N 2 = 3t2 4at + 2a2
M N min M N 2 min
2a2
2a
2
2
Kho st hm f (t) = 3t 4at + 2a minf (t) =
khi t =
3
3
AN 2 = AB 2 + BN 2 2AB.BN cos45o
Nhn xt :M N 2 = AN 2 AM 2 M N SA
ch/m tng t vi cnh cn li
Cu IV.) trnh tnh
Bi ny dng pp ta l ngon. Chn h ta nh sau:
[ bng 45o )
trc 0z cha CS, trc oy chaCB,
trc ox to vi CA gc 45o (v ACB

Khi C(0 : 0 : 0), S(0; 


0; a. 2), B(0; 2a; 0).A(a;
a; 0)


a
a t
a
v N (0; t; 0)
C AS = 2 .AM M a ; a ;
t
t
t
2
r 




a
6
a
t
a 2
a2
MN =
suy ra M N = 3 t 2
a; 3 a;
+ 2 a.
t
t
3 ! 3
3
2

a
a
2
ng thc xy ra khi t = 2 khi M N = 2 ; 0; a.
3
3
3

M AS(a; a; a. 2 v BC(0; 2a; 0) suy ra M N AS = 0

do M N AS v M N BC = 0 suy ra M N BC
Vy M N l ng vung gc chung ca SA v BC .
Cu V.) up_pixar
t a = xy; b = yz; c = xz th a, b, c [1; 4] . Ta cn tm gi tr ln nht ca
|2 a| |2 b| |2 c|
A=
+
+
b+c
c+a
a+b
1
|2 a|

TH1: Nu a, b, c 3 th
b+c
2
3
Do ta c A
2
TH2: Trong 3 s a, b, c c ng 1 s ln hn 3 , khng mt tnh tng qut
|2 b|
|2 a|
3
gi s a > 3 b c th
14;
1 A
c+a
b+c
2
Du bng ca BDT khng xy ra
TH3: Trong 3 s a, b, c c nhiu hn 2 s ln hn 3 Gi s a, b 3 Khi
|2 a|
2
1
|2 c|
2
1
1
3
=
=
Do A 1 + <
b+c
4
2
a+b
6
3
3
2
3
3
Kt lun A
a, b, c [1; 4]. Nn A t gi tr ln nht bng a = b = c = 1
2
2
3
Vy f (x, y, z) t gi tr ln nht bng x = y = z = 1
2
Cu V.) Phm Kim Chung

|2 xy|
1

(1) .
x+y
2
Tht vy : (1) [2(2 xy)]2 (x + y)2 0
[2(2 x y) + x(1 y) + y(1 x)][(4 x y) + x(2 y) + y(2 x)] 0 ( lun ng )
|2 xy|
1
1
Do :
z
z(x + y)
2
2
...tng t cho 2 ci cn li.
(up_pixar) C th rt gn cch chng minh trn bng C-si
|2 xy|
|2 xy|

x+y
2 xy
|2 xy|
1
M vi 1 xy 4 th
(xy 1)(xy 4) 0
2 xy
2
|2 xy|
1
1
nn ta c dpcm:
z
z(x + y)
2
2
Cu VIa.1) trnh tnh
Nhn xt: (d1 )(d2 )
Do yu cu bi ton tng ng vi lp pt t (d) qua P (2; 1) v to vi t (d1 ) gc 45o
2
2
(d) : a(x 2) + b(y + 1) = 0 ax + by 2a + b = 0 vi
(a + b > 0)
|2a b|
2
=
(d) to vi (d1 ) gc 45o khi v ch khi
4(2a b)2 = 10(a2 + b2 )
2
2
2
a +b . 5
b
6a2 16ab 6b2 = 0 3a2 8ab 3b2 a = 3b v a =
3
TH1:a = 3b chn a = 3, b = 1 ta c (d) : 3x + y 5 = 0
b
chn a = 1, b = 3 ta c (d) : x 3y 5 = 0
TH2:a =
3
Cu VIa.2) trnh tnh
Cch1
Nhn xt: hai mt phng cho song song
+Khong cch gia hai mp l h = 2 suy ra bk mt cu R = 1
+Tm I ca mc nm trn (d) nn I(2 3t; 1 + 2t; 1 + 2t)
|2 3t + 2(1 + 2t) 2(1 + 2t) 2|

= |t|
+d[I; ()] =
3
+d[I; ()] = R = 1 t = 1
+t = 1 th I1 (1; 3; 3), I2 (5; 1; 1)
+im I1 nm gia hai mp cho,
+Pt mc (x + 1)2 + (y 3)2 + (z 3)2 = 1
Cch 2
Giao im ca (d) vi () l A(2; 1; 1)
Giao im ca (d) vi () l B(4; 5; 5)
Tm mc l trung im AB l I(1; 3; 3). BK mtcu R = d(I; () = 1
PT mt cu (x + 1)2 + (y 3)2 + (z 3)2 = 1
Cu VIIa.) Hong H
Do b C \ R nn b 6= 0 do x = 0 khng th l nghim ca phng trnh cho,

b

vi b C \ R v r R ty ta c tt c cc cp (a; b) tha bi ton l: (a; b) = r ; b .


r
Ta s CM :

Cu VIb.1)

trnh tnh

Phng trnh AB : x y 5 = 0 v AB = 2
|a b 5|

Gi C(a; b) th d(C; AB) =


2
1
3
m S = AB.d(C; AB) = |a b 5| = 3
2
2
5 + b
5+a
, yG =
m xG =
3
3
Do G thuc 3x y 8 = 0 suy ra 3a b 4 = 0 b = 3a 4
Th b = 3a 4 vo |a b 5| = 3 ta c | 2a 1| = 3 a = 2 v a = 1
T suy ra c hai im C tha mn bi ton l C(2; 10) v C(1; 1)
hoainamss

Phng trnh AB : x y 5 = 0 v AB = 2
Gi G(m, 3m 8). S dng cng thc to trng tm ta suy ra C(3m 5; 9m 19)
| 6m 9|
1
3

Ta c: d(C; AB) =
.
S = AB.d(C; AB) = |6m + 9| = 3...
2
2
2
Cu VIb.2)
dungnhi

vtcp ca t cn tm l
u (a, b, c) vtcp ca Ox, Oy l :
u1 (1, 0, 0);
u2 (0, 1, 0)

a
b
1
ycbt
=
= Chn a = 1 b = 1; c = 2
2
a2 + b 2 + c 2
a2 + b 2 + c 2
x1
y+5
z3
Vy(d) :
=
=
1
1
2
trnh tnh

u (a; b; c) l vtcp ca t cn lp (a2 + b2 + c2 > 0)

|b|
1
|
u.j |
1
|a|
|
u. i |

=
=
=
=

Theo gi thit ta c

2
2
a2 + b2 + c2
a2 + b 2 + c 2
|
u |.| i |
|
u |.| j |
|a|
1
Suy ra |a| = |b| v
=
2
2a2 + c2

|a|
1
TH1: a = b ta c
= c = 2.a Chn a = b = 1th c = 2
2
2a2 + c2
x1
y+5
z3
x1
y+5
z3
ta c (d) :
=
= hoc (d) :
=
=
1
1
1
1
2
2
TH2: a = b ta cng c c = 2.a Chn a = 1 th b = 1, c = 2
y+5
z3
x1
y+5
z3
x1
=
= hoc (d) :
=
=
Ta c (d) :
1
1
1
1
2
2
Cu VIIb.) trnh tnh
x

K x > 0 t u = log2 x, v = log7 (x + 3) Phng trnh tr thnh u2
+ 2v u + xv = 0
2
x

2
x 2
x
=
+ 2v 4xv = 2v
ta c u = 2v ,u =
2
2
2
x
x
TH1:u = ta c pt log2 x =
2
2
x
1
1
xt f (x) = log2 x vi x > 0 f 0 (x) =
f (x) = 2
< 0 khi x > 0
1
2
x ln 2
x ln 2 2
x
do pt log2 x = c khng qu hai nghim nhm thy x = 2, x = 4 l nghim ca pt
2
TH2:u = 2v ta c pt log2 x = 2 log7 (x + 3)
t 2t = log2 x x = 4t phng trnh tr thnh 2t = 2 log7 (4t + 3)
4t + 3 = 7t ( 47 )t + 3.( 17 )t = 1
v tri nghch bin do pt c nghim duy nht t = 1 suy ra x = 4
Vy pt u c hai nghim x = 2, x = 4

Tng hp p n thi th s 12 ca math.vn


Cu I.1)

2x + 1
5
c tp xc nh D = R\{2}. o hm y0 =
< 0 x D
x2
(x 2)2
lim y = lim y = + nn x = 2 l pt tim cn ng

Hm s y =

x2+

x2

lim y = lim y = 0 nn y = 2 l pt tim cn ngang


x+

Bng bin thin


f 0 (x)

f (x) 2

& 2

n
g

Ch
u

g
c

&
Hm s nghch bin
(; 2), (2; +)


 trn 
1
1
; 0 v ct Oy ti 0;
th ct Ox ti
2
2

Cu I.2)Li gii Math213

1,25

5
5
y0 =
.
x2
(x 2)2
5
5
Gi M(x0 ; 2 +
) v N(x1 ; 2 +
) l hai im phn bit trn (C)
x0 2
x1 2
5
5
Do tip tuyn ti M song song tip tuyn ti N nn
=
x0 + x1 = 4.
2
(x0 2)
(x1 2)2
5
5
(x x0 ) + 2 +
.
PT tip tuyn ti M l (d) : y =
2
(x0 2)
x0 2
Khong cch gia hai tip tuyn ny bng khong cch t N n (d), gi s l h.
By gi vi ch x1 x0 = 2(2 x0 ) v x1 2 = 2 x0 ta c th suy ra

2
5
5
5
2+
+
(x1 x0 ) + 2 +
80t
5
x1 2 (x0 2)2
x0 2
2
h =
=
vi t =
2
25
1+t
(x0 2)2
1+
(x0 2)4

2t
2 40 h 2 10
Nhng

1
nn
h
1 + t2
Vy khong cch gia hai tip tuyn song song ln nht khi v ch khi

5
=
2

=
1

x
=
2
+
5
hay
x
5.
0
0
(x0 2)2

Vy ta tm c cp im M(2 + 5; 2 + 5), N(2 5; 2 5) tha mn bi ton.


Cu II.1)Li gii
tranvanthuongvt
 ca


sin x + sin x +
+ sin 4x sin 2x
=0
3 
3 






2 sin x +
cos
+ 2 sin x +
cos 3x
=0
6   6
6 
6




sin x +
cos 3x
+ cos
=0
6
6
6





3x
3x


2 cos
cos
=0
sin x +
6
2
2
6
Li gii ca trnh tnh 



sin x +
Pt sin 4x + sin x = sin 2x
3 
3
x
x
x

2 sin 5 . cos 3 = 2 cos 3 . sin x
2
2
3
 x 2 
x
x

=0
cos 3 sin 5 sin
2
2
2 3


x

x
cos 3 cos 3
sin x +
=0
2
2 6
6
Cu II.2)Li
( gii ca nguyentatthu
xy 1/2
Ta c :
x2 < 1/3
2x
- T phng trnh th hai y =
1 3x2
2
2
Phng trnh th nht 7x = 4x y2 3xy + 2

16x4
6x2
4 13x2 + 2 = 6x2 1 3x2

7x2 =

+
2

37x
1 3x2
1 3x2
p

Ta c 37x4 13x2 + 2 = (9x4 + (1 3x2 )) + 28x4 10x2 + 1 > 2 9x4 (1 3x2 ) = 6x2 1 3x2
Cu III.) Li gii ca fanarsenal_vietnam


x + 1 ex
x+1
+1
ln x
x
ln
1
e
e
= 1
= lim
L = lim x
x
x+1e
x0
x0 e (x 1)
ex
ex
Cu IV)
Vit li y = 2 +

Ch
u

g
c

n
g

0,75

1,0

1,0

1,0

1,0

S
b

F
b

O
b

1,0

Cu V.) Li gii ca up_pixar


Theo Bt ng thc C-si Swatze:
a2 b2 c2
(a2 + b2 + c2 )2
A=
+ +

bc ac ab
abc(a + b + c)
(ab + bc + ac)2 1
=
3
3

M vi mi s a, b, c ta c: abc(a + b + c)

n
g

1,0

Ch
u

g
c

nn A 3(a2 + b2 + c2 )2 (1)
Mt khc t a(a b)(a c) + b(b a)(b c) + c(c a)(c b) 0 (BDT Schur) suy ra:
a3 + b3 + c3 + 6abc (ab + bc + ac)(a + b + c) = a + b + c
a3 + b3 + c3 a + b + c
Do :A =

6 (2)
abc
abc
1 2a2 1 2b2 1 2c2 a + b + c
+
+
=
2A 6 + 3(a2 + b2 + c2 )2 (theo (1) v (2)
Vy ta c:
bc
ac
ab
abc
)
t a2 + b2 + c2 = t th v tri ca BDT cn chng minh:
V T 6 3t 2 + 3(t + 2)2 = 12t + 18 = V P (dpcm)
Trong li gii c s dng 3 BDT ph sau:
a2 b2 c2 (a + b + c)2
BDT C-si Swatze:
+ +
suy ra t Bunhiacopski
x
y
z
x+y+z
BDT Schur dng :a(a b)(a c) + b(b a)(b c) + c(c a)(c b) 0 Chng minh bng gi
s th t bin: a b c (a b)(a c) (a b)(b c)
a(a b)(a c) + b(b a)(b c) + c(c a)(c b) (a b)2 (b c) + c(c a)(c b) 0
(ng )
BDT (a+b+c)2 3(ab+bc+ac) quen thuc tng ng vi (ab)2 +(bc)2 +(ca)2 0
1
Vy du bng ca BDT ban u xy ra a = b = c =
3
Cu VIa.1) Li gii ca trnh tnh
(
x = 2 + at
Gi (d) l t i qua M(2; 2) th (d):
y = 2 + bt
2
(2 + at)
+ (2 + bt)2 = 1 (a2 + 4b2 )t 2 + 2(2a 8b)t + 16 = 0 (?)
Xt phng trnh
4
t (d) ct elip ti hai im phn bit khi v ch khi pt (?) c hai nghim phn bit 0 =
12a2 32ab >
0 Vi k trn th pt (?) c hai nghim t1 ,t2
2 12a2 32ab
ta c A(2 + at1 ; 2 + bt1 ) ,B(2 + at2 ; 2 + bt2 )
ta c |t1 t2 | =
a2 + 4b2 p

2 (a2 + b2 )(12a2 32ab)


nn AB = a2 + b2 .|t1 t2 | =
a2 + 4b2

4
3
2
2
5 = 53a + 128a b + 88a b + 128ab3 + 80b4 = 0
a
53t 4 + 128t 3 + 88t 2 + 128t + 80 = 0 vi t =
b
3

1,0

1,0

Ch
u

g
c

n
g

Cu VIa.2)
Li gii ca trnh tnh
x = 1 + t

(d) : y = 1 + 2t

z = t
Giao im ca (d) vi (P) l im A(1; 1; 0) Ly B(2; 1; 1) thuc
 (d) 
5 1
; 0;
d xc nh c hnh chiu vung gc ca B trn (P) l im H
2 2

x = 1 + 3t
Hnh chiu ca (d) trn (P) l t AH c pt y = 1 + 2t

z=t
ng thng
() nm trong mt phng (P), i qua giao im ca (d)

x = 1 + at
v (P) c pt y = 1 + bt vi a2 + b2 + c2 > 0

z = ct
V () nm trong (P) suy ra a 2b + c = 0 (?)
1
|3a + 2b + c|
=
() to vi AH gc 45o khi
2
2
2
2
a + b + c 14
p
2
2
2
|3a + 2b + c| = 7(a + b + cp) (??)
T (?) v (??) suy ra |2a + 4b| = 7(2a2 4ab + 5b2 ) 10a2 44ab + 19b2 = 0
n y tm c a, b suy ra c v pt ca t
Cu VIIa.) Li gii ca nguyentatthu
V phng trnh c mt nghim thc x = a nn ta c: (2a3 a2 a) + (9a 2a2 + 5)i = 0
(
2a3 a2 a = 0
1
a=
Khi PT (2z + 1)(z2 (i + 1)z + 5i) = 0
2
2
2a 9a 5 = 0
1
z=
hay z2 (i + 1)z + 5i = 0(1) Gii (1) ta c hai nghim : z = 2 i; z = 1 + 2i
2
Cu VIIb.) Li gii ca Math213
n gin t z0 = 4 3i |z0 | = 5 v z20 = 7 24i By gi vit
A = |z2 + 7 24i|2 = |z2 + z20 |2 = (z2 + z0 )(z2 + z0 2 ) = |z|4 + |z0 |4 + (z.z0 + z0 .z)2 2|z.z0 |2
Li theo gii thit th (z + z0 )(z + z0 ) = 1 z.z0 + z0 .z = 1 |z|2 |z0 |2 Th th
2
2401
A = |z|4 + |z0 |4 + (1 |z|2 |z0 |2 )2 2|z.z0 |2 = 2|z|4 2|z|2
+ 1201 = 2 |z|2 21 + 2401
2 2
r
|z| = 1
2401
2
Vy |z + 7 24i| t GTNN bng
ti z tha mn
2

2
|z + 4 3i| = 1

1,0

1,0

Tng hp p n thi th s 11 ca math.vn

Ch

g
c

n
g

Cu Ia.) Hoc sinh t lm

Cu Ib.) DoBaChu
Trc tin bi ton tho mn
PT y = 0 x(2x2 + 3(m 1)x + 6m(1 2m)) = 0 phi c 3 nghim
2x2 + 3(m 1)x + 6m(1 2m) =
0 c 2 nghim phn bit khc 0
{

m 7 hay m 3
= 105m2 66m + 9 0
35
7

(1)
1

6m(1 2m) = 0
m = 0 v m =
2
By gi gi s (Cm ) ct Ox ti 3 im A(x1 ; 0); B(x2 ; 0); C(x3 ; 0)( gs x1 x2 x3 )
Ta c AB = BC x2 x1 = x3 x2 x1 + x3 = 2x2
3(1 m)
Do x1 ; x2 ; x3 l 3 nghim ca PT y = 0 nn x1 + x2 + x3 =
(L Viet)
2
1m
3(1 m)
x2 =
. Li do x2 l nghim PT y = 0 nn suy ra
hay 3x2 =
2
( ( 2 )
)

2
1m
3(m 1)(1 m)
4 3
1m
2
+
+ 6m(1 2m) = 0 m = 1; m =
2
2
2
13

4 3
Trong 3 gi tr ny ca m ch c m = 1; m =
tho (1) v l nhng gi tr cn tm .
13
Cu IIa.) Li gii 1: supervirus
iu kin: sin x = 0
x
x(
x )2 (
x
x) (
x)
x
x
P T 4 sin cos
sin + cos
+ 3 cos sin
cos + sin
=0
2
2
2
2
2
2
2
2
x
x
sin + cos = 0 (1)
2
2 (
x
x
x) (
x
x) ( 2 x
x)
x
hay 4 sin cos
+ sin2
sin + cos
+ 3 cos sin
cos
= 0 (2)
2
2
2
2
2
2
2
2

x
(1) tan = 1 x = + k2 (T m)
2

2 2x

x
x
3
(2) 3tan (4 + 3)tan (4 3) tan 3 = 0
2
2
2

x
5
tan = 2 + 3 x =
+ k2
2
6

5
Vy PT cho c nghim x = + k2; x =
+ k2
2
6
Cu IIa.) Li gii 2: fanarsenal_vietnam

3 cos x
iu kin x = k pt sin x + 1 =
2 sin x(sin x + 1) = 3 cos x
()
2 sin x
bnh phng 2 v ta c 4 sin2 (sin x + 1)2 = 3 cos2 x
4 sin2 x(sin x + 1)2 + 3(sin x + 1)(sin x 1) = 0 (sin x + 1)(4 sin3 x + 4 sin2 x + 3 sin x 3) = 0

TH1 sin x = 1 th phng trnh () nghim ng x = + k2


2

1
3

5
TH2 4 sin3 x + 4 sin2 x + 3 sin x 3 = 0 sin x = 1/2 thay sin x = vo () c cos x =
x=
+ k2
2
2
6

5
KL phng trnh c 2 h nghim x = + k2; x =
+ k2
2
6
Cu IIb.)

Cu III.) Li gii 1: supervirus

xdx
x(1 + cot x) dx =

sin4 x
4
4
(
)

x
1
2x cos x

u =

du =

dx
sin2 x
sin2 x
sin3 x

dx
dv =

v = cos x
sin2 x
sin


(x
)


3 cos x
3 cos x
3 x cos2 x
x cos x 3
1
2x cos x
(9 3 8)

+
S=

dx
=
+
dx

2
dx
3
3
4

sin x sin2 x
sin3 x
sin
x
sin
x
sin
x
18
3
4
4
4
4





3 d(sin x)
3 x(1 sin2 x)
3
(9 3 8)
(9 3 8)
1 3
x

=
+
2
dx =

dx 2S
3
4
2 + 2

sin x
sin x
2 sin x
sin2 x
18 3
18 3
4
4
4
4



3
3
(9 3 8) 1
x
x

3S =
dx
gi I =
dx
+ +2
2

sin x
sin2 x
18 3 3
4
4


u = x
du = dx

x cos x 3
(3 3 4) 1 3
3

I=
+ ln | sin x|| =
+ ln
cos x
dx
4
dv =
v =
sin x
2 2
12 3
2
4
sin x(
) sin x
1
10 3
3
+ + ln
Vy 3S = 1
27
3
2
Ta c S =

Cu III.) Li gii 2: ptkn


(
)
3
2 2
x 1 + cot x dx =

(
)

)
3
1 3
x 1 + cot x d (cot x) =
xd cot x + cot x

3
4
4
4
)
(

(
)
3
x
1
4
= x cot x + cot3 x +
cot x + cot3 x dx

3
3
4
) (3
(
)
(
)
3
27 10 3
2
1
2
=
+
cot x + cot x cot x + 1 dx

81
3
3
4
)
(


(
)
27 10 3
2 3
1 3
=
+
cot xdx +
cot x cot2 x + 1 dx
81
3
3
)
(
4
4
27 10 3
2 3 d (sin x) 1 3
=
+

cot xd (cot x)
81
3
sin x
3
4
4
)
(


4 1 2 4
27 10 3
2

=
+ ln (sin x) cot x
81
3
6

3
3
(
)
(
)
27 10 3
2
3
2
1
+
ln
=
ln
+
81
3
2
2
9
)
(
27 10 3
1 3 1
=
+ ln +
81
3 2 9
Cu III.) Li gii 3: Nguyen An

3
x
Tnh: I =
dx

sin4 x
4
3
dx
2 x)2 d(cot x) chn v = (cot x + cot x ).
t u = x du = dx;
dv =
=
(1
+
cot
4
3
x
sin

3
3
cot3 x
cot x 3
) +
(cot x +
)dx
Vy: I = x(cot x +

3
3

4
4

3
dt

1
3
Tnh 3M =
(3 cot x + cot x)dx t t = cot x
= dx v x = t = 1; x = t =

1 + t2
4
3
3

1
14

2t
1 1
1
1
3
1
1
(t +
)dt = 2 +
d(t2 + 1) = + ln(t2 + 1) 1 = + ln
Vy 3M =
2
2
1
1
1
+
t
2t
1
+
t
3
3
2
1
3

3
3
3

(27 10 3) 1 1 3
Vy I =
+ + ln
81
9 3 2
Cu IV.)

Cu V.) Li gii 1: can_hang2007

)(
)
(
)
9
5
14 5
2
2
2
2
S dng bt ng thc Cauchy-Schwarz, ta c

+1
(a + b) + c =
(a + b) + c
5
9
5 9
T suy ra ta ch cn chng minh c 5(a + b)2 9(a2 + b2 ). Bt ng thc ny tng ng vi
2(a 2b)(2a b) 0, ng do a 2b > 0.
Cu V.) Li gii 2: quybao
Bt ng thc cho tng ng vi 9(a2 + b(2 + c2 ) 10(ab
) (+ bc + ca) )
9
216 2
Tch v tri nh sau V T = 2(a2 + 4b2 ) + a2 + b2 + c2 + 6a2 +
c
25
25
6
72
6
44
8ab + 2ab + bc + ac 10ab + bc + bc + 10ac = 10(ab + bc + ca)
5
5
5
5
Cu V.) Li gii 3: up_pi xar
Chun ha a + b + c = 1 (bng cch chia c 2 v cho (a + b + c)2 v t n ph th c cc n mi c tng
5
bng 1) Khi bt ng thc cn chng minh tng ng vi: P = a2 + b2 + c2
14
25(a + b)2
5
2
2
2
2
p dng BDT Bunhiacopski ta c: (a + b )(4 + 1) (2a + b)
a + b2 (1 c)2
9
9
(
)2
5
5
5
14
5(1 c)2
c

+ c2 =
+
(ng vi mi c)
Do P
9
14
9
14
14
6
5
Vy BDT ban u ng. Du = xy ra khi a = 2b = ; c =
14
14
Cu V.) Li gii 4: supervirus
t x = a 2b 0 a = x + 2b. Thay vo (1) ta c
f (x) = 9x2 + 2x(13b 5c) + (5b 3c)2 0 vi mi x 0
5c 13b
Ta c f (x) = 18x + 2(13b 5c) f (x) = 0 x =
9
1: Nu 5c 13b 0 th f (x) 0 f((x) f (0)
) 0
5c 13b
28
2: Nu 5c 13b > 0 (2) th f (x) f
= (b 2c)(2b c)
9
9
Nhn thy vi k (2) th (b 2c)(2b c) > 0 nn f (x) > 0 Vy ta c pcm
Du = xy ra khi x = 0 v 5c = 13b a = 2b v 5c = 13b
Cu V.) Li gii 5: Trnh Nga
Vit li bt ng thc cn chng minh thnh: 9c2 10c(a + b) + 9(a2 + b2 ) 10ab 0
Xem v tri l 1 tam thc bc hai n c, c bit thc = 28(2a b)(a 2b) 0 nn ta c pcm.
Cu VIa.1.) Li gii 1: fanarsenal_vietnam
Gi Po l hnh chiu ca O ln d No thuc tia OPo sao cho ONo .OPo = 1
D thy No c nh. im P bt k thuc d. N thuc tia OP sao cho ON.OP = 1
Ta c ONo .OPo = ON.OP suy ra N P Po No l t gic ni tip.
o
o
\
Ta li c N\
o Po P = 90 nn No N P = 90 N thuc ng trn (C)ng knh ONo

ta i xc nh ta No p dng l thuyt khong cch ta c Po O = t


n vi
n = (1; 2) l VTPT ca d,

0 + 2.0 4
4
4 8
4 8
4
=
nn Po O = ( ;
) OPo = ( ; ); OPo =
t=
1+4
5
5 5
5 5
5

1
1
1 1
1 1
5
ONo .OPo = 1 ONo =
ONo =
.OPo = OPo ONo = ( ; ); No ( ; )
2
OPo
OPo
16
4 2
4 2

Cc imM (x; y) nm trn (C) tha mn OM .No M = 0
1
1
1
1
phng trnh (C) x.(x ) + y.(y ) = 0 x2 + y 2 x y = 0
4
2
4
2
Cu VIa.1) Li gii 2: supervirus

2
2
2
2
Gi s P (2p +
{4; p), N (x; y) (x + y = 0) R rng OP.ON = OP .ON = 1 = OP .ON
x(2p + 4) + yp = 1
(1)
) 2
do ta c h (
2
2
2
(2p + 4) + p (x + y ) = 1 (2)
(
)
Theo BCS thi t (1) 12 (x2 + y 2 ) (2p + 4)2 + p2 = 1 Theo (2)
4y
Do c c (1) v (2) th xp + y(2p 4) = 0 p =
(3)
x + 2y
Thay (3) vo (1) ta c 4x2 + 4y 2 x 2y = 0 ()
() Chnh l qu tch ca N
Cu VIa.2.) Li gii 1: fanarsenal_vietnam
(a+b+c)2

Mt cuS(x 2)2
+ (y + 2)2 + (z 4)2 = 25 S c tm I(2, 2, 4) v bn knh R = 5

x = 2 + t
ng thng (d) : y = 2 + 3t

z = 3 2t

(d) i qua M (2; 2; 3) v c VTPT


u = (1; 3; 2) IM = (0, 4, 7) ,[
u ;
IM ] = (13; 7; 4)

2
2
2
|[ u ; IM ]|
13 + 7 + 4
234
117
=
d(I; d) =
=
=

14
7
|u|
1 + 9 + 4)
nhn thy d(I; d) < 5 = R suy ra d ct (S) ti 2 im A, B phn bit

mt phng () chnh l mp i qua I v d () c VTPT


n = [
u ; IM ];
n = (13; 7; 4)
phng trnh () 13(x 2) 7(y 2) 4(z + 3) = 0 13x 7y 4z 24 = 0
Cu VI.1b.) Li gii: thichhoctoan
Nhn thy khi din tch ABC t gi tr ln nht th O chnh l trc tm ca n. V c COAB, AOBC
Khi gi B(xB , yB ); C(xC , yC ) th xB = xC (do AOBC)

C AB = (xB 1; yB ), OC = (xB , yC ), OB = (xB .yB ), AC = (xB 1; yC )



2 y 2 = x2 2x3 + x4 (1)
Ta c:
OC = 0 xB (xB 1) + yB yC = 0 yB yC = xB x2B yB
C
B
B
B
{ COAB AB.{
x2 + y 2 = 1
y 2 = 1 x2
V:

x2 + y 2 = 4
y 2 = 4 x2
thay vo (1) ta c: (1 x2 )(4 x2 ) = x2B 2x3B + x4B (xB 1)(x2B + 2xB 2) = 0 Gii ra th li ch thy
xB = 1 tha
T d dng tm c ta cc im cn li v vic cui cng l vit phng trnh cc cnh tam gic!
Cu VI.2b.) DoBaChu
mc(S) c tm I(2; 1; 3) , bn knh R = 3 . Ta c d(I, (P )) = 5 > 3 = R (P ) ngoi (S)
Gi H l hnh chiu ca I trn (P ) v K l giao im ca (S) v (P ) (K gia I v H) (h.v)
Ta lun c IM + M N IN IH = IK + KH R + M N R + KH M N KH = d(I, (P )) R =
5 3 = 2 (v IM = IK = R)
Du bng xy ra M K v N H Ta tm to K v H .
ng thng
d qua I v vung gc vi (P ) nn nhn VTPT (2; 2; 1) ca (P ) lm VTCP

x = 2 + 2t
nn c PT : y = 1 + 2t (1) Tham s t ng vi giao im H ca d v (P )

z =3t
5
14
l nghim PT: 2(2t + 2) + 2(1 + 2t) (3 t) + 16 = 0 t = , thnh th H( 34 ; 13
3 ; 3 )
3

xH + 23 xI

x
=
=0

1 + 23

yH + 23 yI

2
V KH = 2; IK = R = 3 nn KH = KI yK =
= 3 , vy K(0; 3; 4)

3
1 + 23

zH + 23 zI

z
=
=4
K
1 + 23
14
Ta i n kt lun :To cc im M ; N cn tm l M (0; 3; 4) v N ( 43 ; 13
3 ; 3 ) m bo M N nh nht
bng 2. ng thng qua M, N c PT (1)
Cu VII.b.) Li gii: thichhoctoan
z2
x 2 + yi
t z = x + yi c
=
thc hin php chia s phc a v:
z+2
x + 2 + yi
(
x2 + y 2 4
4yi

)
+
=

cos
+
i
sin
(x 2)2 + y 2 (x 2)2 + y 2
3
3
2 + y2 4
x

(x 2)2 + y 2
2
( > 0) suy ra
3
4yi

=
2
2
(x 2) + y
2
(
)
(
)
2 2
4 2
4y
=
Suy ra x2 + y 2 4 = hay x2 + y
3
3
3
(
)
2
4
Vy tp hp cc im biu din s phc z l ng trn c tm I 0; , bn knh R = trn trc thc!
3
3

TNG HP P N THI TH S 09 CA MATH.VN


Cu I.1 (1 im)
Cho hm s y = x3 mx + m 1 (Cm ) Kho st s bin thin v v th (C) ca hm s vi m = 3.
Gii .
th

y = x3 3x + 2

.V

M
AT

Cu I.2 (1 im)
Tm m tip tuyn ca th hm s cho ti im c honh x = 1 ct ng trn
(C) : (x 2)2 + (y 3)2 = 4 theo mt dy cung c di nh nht.
Gii .
trnh tnh
Phng trnh tt ti im c honh x = 1 l :
y =p
(3 m)x + m + 1 (3 m)x y + m + 1 = 0 (d)
2 (3 m)2 + 1
|4 m|
|1 + (3 m)|
d[I, (d)] = p
=p
p
= 2
(3 m)2 + 1
(3 m)2 + 1
(3 m)2 + 1
TT ct t ti hai im A, B sao cho AB min d(I, (d)) max Du = xy ra khi m = 2. Khi phng trnh tip tuyn l
xy+3 = 0
kienqb
D dng tm c tip tuyn ti im c honh x = 1 l y = (3 m)x + m + 1. t ta c im c nh m tip tuyn
lun i qua l M(1; 4).

Ta c tm I ca ng trn l I(2; 3) bn knh R = 2. Tnh c IM = 2 < R. Nn M nm trong ng trn.


Gi H l hnh chiu vung gc ca I ln tip tuyn. gi s tip tuyn ca (Cm) ct ng trn theo dy cung AB th

AB = 2AH = 2 R2 IH 2 .

Suy ra AB min khi IH max. Tc l H trng vi M. Khi tip tuyn nhn IM lm vc t php tuuyn

Ta c IM(1; 1) suy ra m = 2. Khi phng trnh tip tuyn l y = x + 3


Cu II.1 (1 im)
3
Gii phng trnh sau trn R: cos2 2x + cos 4x(tan 2x. cot x 1) =
4
Gii .
thaydoip1
3
cos2 2x + cos 4x(tan 2x. cot x 1) =
iu kin sin x 6= 0; cos 2x 6= 0
4
tan 2x. cot x 1 cos 2x = sin 2x. cot x cos 2x = 2 cos2 x cos 2x = 1
1
cos 4x 3
tan 2x. cot x 1 =
cos2 2x +
=
(1)
t = cos 2x 6= 0 cos 4x = 2 cos2 2x 1 = 2t 2 1
cos 2x
cos 2x
4
2t 2 1 3
1
(1) t 2 +
=
(2t 1)(2t 2 + 5t + 4) = 0 t =
t
4
2
1

cos 2x = 2x = + 2k
(k Z) x = + k
(k Z)
2
3
6
Cu II.2 (1 im)

x2 + 2 = y2 + 2
x
Gii h phng trnh trn tp s thc: p y

3 4(x3 + y3 ) + 2xy = 0
Gii .
Mercury

x2 + 2 = y2 + 2
y
x
p

3
4(x3 + y3 ) + 2xy = 0

(x y)(x + y + 2 ) = 0
xy
p

3
4(x + y) ((x + y)2 3xy) + 2xy = 0

"

p
x=y=0
3
Vi x = y: 3 4(x3 + y3 ) + 2xy = 0 8x3 + 2x2 = 0 2x + 2x2 = 0
x = y = 1
v
!
u

2
u
p
2
2
2
3

3xy + 2xy = 0
Vi x + y = : 3 4(x + y) ((x + y)2 3xy) + 2xy = 0 t
4
xy
xy
xy
r
4
4
3
= xy
3 = (xy)3 (xy)6 + 3(xy)3 4 = 0
3
3
(xy)

3
(xy)

x = 1
y = 1
(xy)3 = 1
x + y = 2


2
1

5
15
2
(xy)3 = 4 x + y =
x=

y=
3 2
3 2
3
3
4
4
4
4
Cu III. (1 im)
Z ln 2
3 x
Tnh tch phn:
e 1dx
0

Gii .
Z ln 2

3t 2
3 x
I=
e 1dx t = 3 ex 1 ex = t 3 + 1 d(t 3 + 1) = dex ex dx = 3t 2 dt dx = 3
dt
t +1
0
x = Z0 t = 0 x = Zln 2 t = 1 
Z 1
Z 1
1
1
3
1
3t 2
dt =
dt = 3x|10 3I1 = 3 3I1
3 3
dt 3
dt = 3
t. 3
I=
3
t +1
t
+
1
t
+
1
0
0
0
0

Z 1
Z 
1
1
1 1
2t 1
3
I1 =
dt
dt =

+
3
3 0 t + 1 2 (t 2 t + 1) 2 (t 2 t + 1)
0 t +1

Z 1
Z 1
Z 1
Z 1
Z
Z
1
1
1
2t 1
1
1
1
d (t + 1) 1 1 d t 2 t + 1
1 1
dt
=
dt
dt +
dt =

+

2
2
2
1
3 0 t +1
6 0 t t +1
2 0 t t +1
3 0 t +1
6 0 t t +1
2 0 t 2+ 3
2
4
1
1



1
1
1
1
1
= ln (t + 1) ln t 2 t + 1 + I2 = ln (2) + I2
3
6
2
3
2
0
0



Z 1
dt
3


I2 =
tan a
a
;
t =1a=
t =
t =0a=
2 3
1
2
2
2
2
6
6
0
t2 +4

Z
Z
Z
6
2 3
d tan a
2 3
3 6
2 3 6 d tan a
2 3 6
da =
= I2 =
I2 =
=
a =

2
2 6 3
3 6 tan2 a + 1
3 6
3
9
3
+
tan
a
6
2
4
!



3
1
1 2 3
1
1
= 3 ln (2)
ln (2) + I2 = 3 3
ln (2) + .
Vy I = 3 3I1 = 3 3
3
2
3
2 9
3

Cu IV. (1 im)
d = 60o , ASC
d = BSC
d = 120o . Xc nh tm mt cu ngoi tip hnh
Cho hnh chp S.ABC c SA = SB = SC = a v ASB
chp.Tnh t s gia th tch khi chp S.ABC v khi cu ngoi tip khi chp .
Gii .

Cu V. (1 im)
Cho a, b, c l cc s thc dng tha mn a + b + c = 3. Chng minh rng: (a + c)(b + 1) abc(a2 + b2 + c2 + 1)
2

Gii .

Cu VIa.1 (1 im)
Trong mt phng Oxy, cho im A(2; 1). Tm im B trn trc honh, im C trn ng phn gic ca gc phn t th nht
sao cho chu vi tam gic ABC nh nht.
Gii .
DoBaChu Gi E; F ln lt l cc im i xng ca A qua trc honh v qua ng phn gic d ca gc phn t th nht .
Ta c E(2; 1) v F(1; 2) . Khi theo tnh cht php i xng trc , ta thu c chu vi tam gic ABC l

= AB + BC +CA = EB + BC +CF EF = 10 (tnh cht ng gp khc)

Du bng
xy ra tc min = 10 E, B,C, F thng hng

B = EF Ox
y = 3x + 5
y = 3x + 5

to B,C theo th t l nghim cc HPT


v
C = EF d
y = 0
y = x
( Do EF : y = 3x + 5) B( 35 ; 0);C( 45 ; 54 )
tranvanthuongvt
B Ox = B(b; 0),C d : y = x = C(c; c)
p
p
p
Chu vi 2P = AB + BC +CA = (2 b)2 + 12 + (b c)2 + c2 + (2 c)2 + (c 1)2
q
p
1
3
1
Theo Bunhiacopki ((2 b)2 + 12 )(( 31 )2 + 12 ) (2 b) + 1 = (2 b)2 + 12 (2 b) +
(1)
3
10
10
q
p
3
1
1
5 2c
((2 c)2 + (c 1)2 )(( 43 )2 + ( 41 )2 ) (2 c) + (c 1) =
= (2 c)2 + (c 1)2 (5 2c) (2)
4
4
4
10
q
p
1
1
((b c)2 + c2 )(( 31 )2 + 12 ) (b c) + c = (b c)2 + c2 (b + 2c) (3)
3
10

3
1
T (1), (2), (3) = 2p (2 b + 5 2c + b + 2c) + = 10
10
10
ng thc xy ra khi b = 53 , c = 45 Vy B( 35 ; 0),C( 45 ; 54 )

Cu VIa.2 (1 im)
Trong khng gian vi h ta Oxyz, cho mt phng (P) : 2x y + z + 2 = 0, (Q) : x + 2y + 2z 4 = 0. Lp phng trnh
ng thng d nm trong mt phng P v cch mt phng Q mt khong bng 1.
Gii .
Gi s M(x; y; z) l mt im bt k thuc ng thng d cn tm. Do d nm trong P nn 2x y + z + 2 = 0.
Mt khc d cch mt phng Q mt
" khong bng 1 nn d||Q v khong cch t M n Q cng bng 1.
x + 2y + 2z 4 = 3
Suy ra |x + 2y + 2z 4| = 3 =
x + 2y + 2z 4 = 3

x = 4t

2x y + z + 2 = 0

5x + 4z 3 = 0
t R
TH1.
=
. t x = 4t ta c pt ca d l y = 11
4 + 3t
x + 2y + 2z 4 = 3

3x 4y + 11 = 0

z = 3 5t
4

x = 4t

2x y + z + 2 = 0
5x + 4z + 3 = 0
TH2.
suy
t R
. t x = 4t ta c pt ca d l y = 45 + 3t

x + 2y + 2z 4 = 3
3x 4y + 5 = 0

z = 3 5t
4
Cu VIIa. (1 im)

Tm tp hp im M biu din s phc z tha mn z + 3z = (2 + i 3)|z|


Gii .

T gi thit z + 3z = (2 + i 3)|z| = z + 3z = (2 i 3)|z| = (z + 3z)(z + 3z) = (2 + i 3)(2 i 3)|z|2


10|z|2 + 3(z + z)2 6z.z = 4|z|2"+ 3|z|2 (z + z)"2 = |z|2 .

3x
3)
y
=
z
=
x(1
+
i
.

t z = x + iy, suy ra y2 = 3x2


=
z = x(1 i 3)
y = 3x

Th li thy z = |x|(1 i 3) tha mn.

Kt lun tp hp cc s phc z cn tm c dng z = |x|(1 i 3), x R.


Cu VIb.1 (1 im)
Trong mt phng Oxy, cho im A(2; 1). Tm im B trn trc honh, im C trn ng phn gic ca gc phn t th nht
sao cho chu vi tam gic ABC nh nht.
Gii .

Cu VIb.2 (1 im)
x+3 y+1 z3
=
=
. Lp phng trnh mt phng (P) i qua
2
1
1
hai im A(1; 2; 0), B(0; 1; 3) v to vi ng thng d mt gc 30o .
Gii .

t d c vtcp
u (2; 1; 1) AB(1; 1; 3) (P) qua A suy ra (P) : a(x + 1) + b(y + 2) + cz = 0 (a2 + b2 + c2 > 0)

V (P) i qua A, B nn ta c
n AB a + b + 3c = 0 = a = (b + 3c) (P) : (b + 2c)(x + 1) + b(y + 2) + cz = 0
1
| 2(b + 3c) + b + c|
=
(P) to vi t d gc 30o th ta phi c p
2
2
2
(b 3c) + b + c ) 6 2
2b2 bc 10c2 = 0 = 2b = 5c hay b = 2c
TH1 2b = 5c chn b = 5, c = 2 = a = 11 (P) : 11(x + 1) + 5(y + 2) + 2z = 0 (P) : 11x + 5y + 2z 1 = 0
TH2: b = 2c chn c = 1, b = 2 = a = 1 (P) : (x + 1) 2(y + 2) + z = 0 (P) : x 2y + z 5 = 0
Vy c hai mp tha mn (P) : 11x + 5y + 2z 1 = 0 v (P) : x 2y + z 5 = 0
Cu VIIb. (1 im)
Tm s t nhin n ln nht tha mn bt phng trnh: Cn1 + 2Cn2 + 3Cn3 + ... + nCnn 38n n2
Gii .
Hin nay phn i s t hp c a vo chng trnh lp 11 (khi cha hc o hm) Do gii bi ny ta c th
lm nh sau:
k1
Cho k = 1, 2, 3, ..., n v cng cc ng thc li ta c
Ta c kCnk = nCn1
n1
1
0
) = n.2n1
V T = n(Cn1 +Cn1 + ... +Cn1
V T 38n n2 2n1 38 n
Bng cch th trc tp ta s cm n = 6 l gi tr ln nht tha mn bi ton
n y ta dng pp quy np chng minh 2n1 > 38 n vi n 7 vi n N
Trong khng gian vi h ta Oxyz, cho ng thng d :

TNG HP P N THI TH S 08 CA MATH.VN


Cu I.1 (1 im)
Cho hm s y = (m 3)x3 4(m 3)x2 (m + 1)x + m Kho st s bin thin v v th (C) ca hm s khi
7
m=
3
Gii .
3
7
y = 32 x3 + 83 x2 10
3 x+ 3

.V

M
AT

Cu I.2 (1 im)
Chng minh rng h th hm s lun i qua ba im c nh thng hng.
Gii .
(C) : y = (m 3)x3 4(m 3)x2 (m + 1)x + m
M(x0 ; y0 ) (C) y0 = (m 3)x03 4(m 3)x02 (m + 1)x0 + m
(C)i qua M vi mi m m(x03 4x02
x0 + 1) 3x03 + 12x02 x0 y0 = 0, Am
x3 4x2 x + 1 = 0
x3 4x2 x + 1 = 0
0
0
0
0
0
0

y0 = 3x3 + 12x2 x0 = 3(x3 4x2 x0 + 1) 4x0 + 3)


3x3 + 12x2 x0 y0 = 0
0
0
0
0
0
0

f (1). f (0) < 0


x3 4x2 x + 1 = 0 (1)
0
0
0

V (1) c 3 nghim phn bit f (0). f (1) < 0

y0 = 4x0 + 3

f (4). f (5) < 0

nn h th hm s lun i qua 3 im c nh, v ta 3 im ny lun tha phng trnh:y0 = 4x0 + 3


nn 3 im ny cng thuc ng thng.
Cu II.1 (1 im)

Gii phng trnh : cos x cos 7x = 3 3 sin x


Gii .

Bt u cos x cos 7x = 3 3 sin x 2 sin 4x sin 3x = 3 3 sin x

8 sin x. cos x. cos 2x. sin 3x = 3 3 sin x sin x(2 sin 4x(1 + 2 cos 2x) 3 3) = 0.
Trng hp 1. sin x = 0 x = k , k Z.

3 3
3 3

sin( 2 x). sin( 2 2x). sin 3x =


.
Trng hp 2. cos x. cos 2x. sin 3x =
8
8
V ( 2 x) + ( 2 2x) + 3x = nn 2 x, 2 2x, 3x l ba gc ca mt tam gic.

3
3
Do sin( 2 x). sin( 2 2x). sin 3x =
2 x = 2 2x = 3x, v nghim.
8
Kt lun phng trnh cho c h nghim x = k , k Z.
Cu II.2 (1 im)

3
Gii phng trnh : x3 x2 10x 2 = 7x2 + 23x + 12.
Gii .


3
x3 x2 10x 2 = 7x2 + 23x + 12


3
x3 x2 10x 2 + 7x2 + 23x + 12 = 7x2 + 23x + 12 + 7x2 + 23x + 12

3
(x + 2)3 + x + 2 = 7x2 + 23x + 12 + 7x2 + 23x+ 12


3
7x2 + 23x + 12
f (t) = t 3 + t f 0 (t) = 3t 2 + 1 > 0 f (x + 2) = f


3
"
x + 2 = 7x2 + 23x + 12 (x + 2)3 = 7x2 + 23x + 12 (x 4) x2 + 3x + 1 = 0
x=4

x = 23 + 25 hay x = 32 25
Cu III. (1 im)

Tnh tch phn

ln(1 + 3 tan x)dx.

Gii .

ln(1 + 3 tan x)dx

x = 3 t = 0, x = 0 t = 3 .
0

Z
Z
Z

3
3
3

3( 3 tant)
2dt

ta c I =
)dt =
) 2I =
ln(1 + 3 tan( t))dt =
ln(1 +
ln(
3
0
0
0
1 + tan 3
1 + 3 tant

Z
Z

3
3
2

)dx =
ln(1 + 3 tan x)(
ln 2dx = ln 2
3
0
0
1 + 3 tan x
Cu IV. (1 im)
Cho hnh chp SABCD c SA = x,cc cnh cn li bng 2. Vi gi tr no ca x th th tich ca khi chp ln
nht,tm gi tr ln nht .
Gii .
t x =

t dx = dt,

S
b

D
b

Gi O l giao im AC v BD OD = OB cng vi SB = SD
nn SOBD BOmp(SAC).
Mt khc SO2 = SB2 OB2 = AB2 OB2 = OA2
SO = OA = OC tam gic SAC vung ti S
AC2 = x2 + 4 4OA2 = x2 + 4.

Do 4OB2 = 12 x2 0 < x < 2 3


2
v 16SSOA
= x2 (4OA2 x2 ) = 4x2 .
Li c VS.ABCD t GTLN VSOAB t GTLN
x2 (12 x2 ) t GTLN x2 = 12 x2
(do x2 > 0, 12 x2 > 0 v x2 + 12 x2 = 12).

Suy ra x2 = 6 x = 6

Cu V. (1 im)
Cho bn s thc khng m x, y, z,t tha mn iu kin x + y + z + t = 4.

Tm gi tr ln nht,gi tr nh nht ca biu thc A = x 3 + yz + y 3 + zt + z 3 + tx + t 3 + xy.


Gii .

Ta c

+ A = x 3 + yz + y 3 + zt + z 3 + tx + t 3 + xy 3(x + y + z + t) = 4 3
ng thc xy ra khi x = 4, y = z = t = 0 v cc hon v

Vy minA = 4 3
+ Theo AM-GM

A = x 3 + yz + y 3 + zt + z 3 + tx + t 3 + xy



= 21 2 x 3x + xyz + 2 y 3y + yzt + 2 z 3z + zty + 2 t 3t + txy
41 (4x + 3x + xyz) + 14 (4y + 3y + yzt) + 41 (4z + 3z + zty) + 41 (4t + 3t + txy)
= 47 (x + y + z + t) + 41 (xyz + yzt + ztx + txy) = 7 + 14 (xyz + yzt + ztx + txy)
By gi ta chng minh xyz + yzt + ztx + txy 4 v ng thc c khi x = y = z = t = 1
Tht vy ta c:
xyz+yzt +ztx+txy = xy(z+t)+zt(x+y) 14 (x+y)2 (z+t)+ 14 (z+t)2 (x+y) = 41 (x+y)(z+t)(x+y+z+t)
= (x + y)(z + t) 14 (x + y + z + t)2 = 4
Khng nh c chng minh
Vy maxA = 8 khi x = y = z = t = 1
Cu VIa.1 (1 im)
Trn mt phng ta vi h trc ta Oxy cho ng trn (C) : x2 +y2 4y = 0 v ng thng d : 3x+4y+7 =
0. MP v MQ l cc tip tuyn k t mt im M bt k trn ng thng d ti ng trn (C),tip im l
P, Q.Chng minh rng nu im M di ng trn ng thng d th ng thng PQ lun i qua mt im c nh.
Gii .

x = 1 + 4t
2
2
Vit li (C) : x + (y 2) = 4 v (d)
.
y = 1 3t
Tm ca ng trn I(0; 2) v M(1 + 4t; 1 3t) (d).

Gi s P(x0 ; y0 ) (C) v do MP l tip tuyn nn MP.IP = 0.


Suy ra (x0 + 1 4t)(x0 0) + (y0 + 1 + 3t)(y0 2) = 0
cng vi x02 + (y0 2)2 = 4 ta c x0 (1 4t) + 3(t + 1)y0 6t 2 = 0.
Vai tr P, Q nh nhau nn PT ng thng PQ l (1 4t)x + 3(t + 1)y 6t 2 = 0.
Gi s PQ i qua im c nh J(a; b) khi M di ng trn (d), khi (1 4t)a + 3(t + 1)b = 6t + 2, AAt.
4
Cho t = 14 b = 14
15 Cho t = 1 a = 5
Thay ta J( 54 ; 14
15 ) vo pt PQ ta c
4
16
14
14
4
(1 4t) 5 + 3(t + 1) 14
15 6t 2 = 0 5 + 5 t + 5 + 5 t 6t 2 = 0 lun ng t.
Kt lun PQ lun i qua im c nh J( 54 ; 14
15 ).
Cu VIa.2 (1 im)
Trong khng gian vi h ta vung gc Oxyz,cho hai im M(2; 1; 4) N(1; 1; 3) v mt phng ( ) : 2x y
2z 12 = 0.Tm tp hp tt c cc im I trn ( ) sao cho tam gic IMN c din tch nh nht. Gii .

Gi s I(u; 2u 2v 12; v) l mt im bt k trn mp( ) vi u, v R no .

Tnh NI = (u 1; 2u 2v 13; v 3) v NM = (1; 0; 1).


Suy ra [NI, NM] = (2u 2v 13; v u 2; 13 2u + 2v).


Mt khc SIMN = 12 |[NI, NM]|,
do SIMN t GTNN biu thc P = (2u 2v 13)2 + (v u 2)2 + (13 2u + 2v)2 t GTNN.
t t = u v th P = (2t 13)2 + (t + 2)2 + (13 2t)2 = 9t 2 100t + 342.
50
8
; u 50
Suy ra P t GTNN t = 50
9 v = u 9 I(u; 9
9 ).

x = t
Kt lun tp hp im I tha mn l ng thng (d) : y = 89

z = 50 + t
9

Bn no khng thch dng tch c hng th lm nh sau:

Ta c
n = (2; 1; 1) v NM = (1; 0; 1) nn MN||( ).
Gi I l im bt k trn ( ), K l hnh chiu vung gc ca I ln MN v H l hnh chiu vung gc ca K
ln ( ).
Khi SIMN t GTNN IK t GTNN.
Mt khc IK IH (ng xin hnh chiu), do IK nh nht IK( ) I l hnh chiu vung gc ca
K ln ( ).
Do tp hp im I l hnh chiu vung gc ca ng
thng MN ln ( ).

x = 1 + 2t
PT ng thng (d) i qua N vung gc vi ( ) l y = 1 t

z = 3 2t
8
7
0 43
Gi N 0 l giao im ca (d) v ( ), d dng
tm c N ( 9 ; 9 ; 9 ).

43

x = 9 + t
Cui cng tp hp im I l ng thng y = 89

z = 7 + t
9

Cu VIIa. (1 im)

z 2i
l mt s o. Gii .
z + i
Trc ht |z + 1 2i| = |z + 3 + 4i| (z + 1 2i)(z + 1 + 2i) = (z + 3 + 4i)(z + 3 4i)
2(z + z) + 2i(z z) + 20 = 0 (z + z) + i(z z) = 10
(1).
z 2i
z + 2i
z 2i
l s o nn
=
(z 2i)(i z) = (z + i)(z + 2i) z2 + z2 3i(z z) 4 = 0
Th hai
z + i
z + i
zi
(z + z)2 [i(z z)]2 6i(z z) 8 = 0 10[(z + z) i(z z)] 6i(z z) 8 = 0
5(z + z) 2i(z z) + 4
=0
(2).
z + z = 24
7
23
T (1) v (2) ta suy ra
z = 12
7 + 7 i.
z z = 46 i
7

Tm s phc z tha mn ng thi hai iu kin sau: |z + 1 2i| = |z + 3 + 4i| v

Cu VIb.1 (1 im)

Cho tam gic ABC vung ti A c B(3; 0),C(7 : 0) v r = 5 2 5Tm ta tm ng trn ni tip tam gic
ABC. Gii .

Do B,C nm trn Ox nn gi I(x0 ; y0 ) l im cn tm, vi |y0 | = r.


d = 135o .
Tnh BI 2 = (x0 + 3)2 + r2 ,CI 2 = (x0 7)2 + r2 v BC2 = 100; BIC
Mt khc SBIC = 21 BC.r = 12 BI.CI. sin 135o ((x0 + 3)2 + r2 ).((x0 7)2 + r2 ) = 200.r2 .
t t = x0 2 ta c phng trnh ((t + 5)2 + r2 ).((t 5)2 + r2 ) = 200.r2

t 4 2t 2 (25 r2 ) + (r2 25)2 = 200r2 t 2 = (r 5 2)2 75 = 50 hay t 2 = (r + 5 2)2 75

Do t 2 = (r + 5 2)2 75 = (10 2 5)2 75 = 25((2 2 1)2 3) = 50( 2 1)2 .

Suy ra t = 10 5 2 hay t = 5 2 10 x0 = 12 5 2 hay x0 = 5 2 8


Cui cng c 4 im tha mn bi ton l

I1 (12 5 2; 5 2 5), I2 (12 5 2; 5 5 2), I3 (5 2 8; 5 2 5), I4 (5 2 8; 5 5 2).


Cu VIb.2 (1 im)
x3 y1 z1
x7 y3 z9
Trong khng gian cho hai ng thng (d1 ) :
=
=
(d2 ) :
=
=
.
7
2
3
1
2
1
Lp phng trnh ng thng (d3 ) i xng vi (d2 ) qua (d1 ), ( tc l vi im A bt k thuc (d3 ) lun c im
B thuc (d2 ) i xng qua (d1 ) v ngc li.
Gii .

x = 3 + 7t

Ta c vct ch phng ca (d1 ) l u 1 = (7; 2; 2) v pt tham s ca ng thng (d1 ) : y = 1 + 2t .

z = 1 + 3t
Ly hai im A(7; 3; 9) v B(6; 1; 10) thuc ng thng (d2 ) vi H, K ln lt l hnh chiu vung gc trn
(d1 ).

Khi H(3 + 7t1 ; 1 + 2t1 ; 1 + 3t1 ) AH = (7t


9).
1 4; 2t1 2; 3t1 8) v BK = (7t2 3; 2t2 ; 3t2

7(7t1 4) + 2(2t1 2) + 3(3t1 8) = 0


t1 = 28

31

Mt khc (AH. u 1 = 0, ), (BK. u 1 = 0, ) :


24
7(7t2 3) + 2(2t2 ) + 3(3t2 9) = 0
t2 =
31

87 115
261 79 103
Do H( 289
31 ; 31 ; 31 ) v K( 31 ; 31 ; 31 ).
Gi A0 , B0 l im i xng ca A, B qua (d1 ) tng ng.

81
49
104
25 46
55
0 336 127
0 0
Suy ra A0 ( 361
31 ; 31 ; 31 ) v B ( 31 ; 31 ; 31 ) A B = ( 31 ; 31 ; 31 ).
81
x 361
y 31
z + 49
31
31
Cui cng pt ng thng (d3 ) l
=
=
25
46
55
Cu VIIb. (1 im)
Gii phng trnh 4x+1 3x+1 + 41x 31x = 2x + 2x
Gii .
Ta c 4x+1 3x+1 + 41x 31x = 2x + 2x 3[(4x + 4x ) (3x + 3x )] + [(4x + 4x ) (2x + 2x )] = 0
Nhn thy nu PT c nghim l x0 th x0 cng l nghim do ta ch cn gii phng trnh ng vi x 0.
Nhn thy PT c mt nghim x = 0 ta chng minh n cng chnh l nghim duy nht .
42x + 1 32x + 1
x
x
x
x
>
Tht vy, v x > 0 nn (4 + 4 ) (3 + 3 ) > 0
4x
3x
x
x
x
x
(4 3 )(3 4 1) > 0(ng)
Tng t (4x + 4x ) (2x + 2x ) > 0
Vy trn R+ phng trnh c nghim duy nht v cng chnh l nghim duy nht ca PT

TNG HP P N THI TH S 07 CA MATH.VN


(1 im)
3x 1
Cho hm s y =
Kho st s bin thin v v th (C) ca hm s.
x1
Gii .
Cu I.1

6
b

3x 1
x1

y=

.V

2
b

M
AT

Cu I.2 (1 im)
Tm ta hai im B,C thuc hai nhnh khc nhau ca (C) sao cho tam gic ABC vung cn ti A(2; 1)
Gii .
2
t x = 1 + X, y = 3 +Y Y = khi A(1; 2) Ly B(a; a2 ),C(b; 2b ) nm trn ths v nm trn hai nhnh
X

2
nn gs a < 0, b > 0 AB(a 1; a + 2), AC(b 1; 2b + 2)

tam gic ABC vung cn ti A khi


v ch khi ABAC v AB2 = AC2
a2 b2 (a + b 2) 4(a + b + 2ab) = 0
T hai k ny bin i ta c h
(ab(ab (a + b) + 1) + 4(ab + a + b + 1) = 0

P2 (S 2) 4(S + 2P) = 0
2P2 + 8P
t S = a + b, P = ab ta c h
T pt th hai suy ra S = 2
P(P S + 1) + 4(S + P + 1) = 0
P 4
"
P=1
(loi)
Th vo pt cn li ta c P4 + 3P3 + 12P 16 = 0 suy ra
P = 4 (nhn)

a + b = 0
Vi P = 4 S = 0 ta c h
suy ra a = 2, b = 2 suy ra B(2; 1),C(2; 1)
ab = 4
vy vi h ta 0xy th hai im cn tm l B(1; 2),C(3; 4) hoc ngc li
Cu II.1 (1 im)

Gii phng trnh sin(2x 4 ) cos 2x 2 2 sin(x 4 ) = 0


Gii .


sin(2x 4 )(cos2 x sin2 x) 2(sin x cos x) = 0 (cos x sin x) sin(2x 4 ) sin(x + 4 ) + 2 = 0
cos x sin x = 0 tan x = 1 x = 4 + k
(ci tha s cn li v nghim do
| sin(2x 4 )| 1; | sin(x + 4 )| 1 | sin(2x 4 ). sin(x + 4 )| 1 sin(2x 4 ). sin(x + 4 ) 1)
1

(1 im)

6x4 (x3 x)y2 (y + 12)x2 = 6


Gii h phng trnh :
5x4 (x2 1)2 y2 11x2 = 5
Cu II.2

Gii .
trnh tnh
vi x = 0 khng tha mn

1
ta c h
5u2 u2 y2 1 = 0
x

t 2 y + ty2 = 6

t2 + y2 = 5

vi x khc 0 chia c hai v ca hai pt trong h cho x2 sau t u = x

6 ty2 t 2 y = 0
1
Li chia c hai v cho u2 v t t = ta c h
5 y2 t 2 = 0
u

6u2 uy2 y = 0

y l h x lai II ta gii tm c t, y u, y x, y
thaydoip1
(
(

2

6x4 x3 x y2 (y + 12) x2 = 6
6 x2 1 x x2 1 y2 yx2 = 0
2

2
2
5x4 x2 1 y2 11x2 = 5
5 x 2 1 x 2 1 y2 x 2 = 0

2  2


x2 1
x 1 2

(
(

y y = 0

6
2 ay2 y = 0
21
6a
6a2 y = ay2
x
x
x


a=
2  2
2
 2
2 a2 y2 1 = 0
2 5 y2 = 1

x
5a
a
x

1
x

y2 x 2 = 0
5
x
x



2
6a2 y 2 = a2 y4


6
y4
3 5y + 6 2 = y4 5 y2

y
=

1
a2 =
5 y2
5 y2
5 y2

(y 1) (y 2) 2y4 + 6y3 y2 3y + 18 = 0 2y4 + 6y3 y2 3y + 18 > 0
Cu III. (1 im)
Z
2
cos 2x + cos x + 2

Tnh tch phn I =


dx
0 1 + cos x + cos x cos2 x
Gii .
ta c cos 2x + cos x + 2 = 2 cos2 x + cos x + 1 = (cos x + 1)2 (cos x cos2 x)
Z 
Z p

p

2
2

2
2
do I =
1 + cos x cos x cos .dx = (x + sin x)|0
cos x cos2 x.dx = + 1 I1
2
0
0
s
r
Z
Z p
Z
cos x cos2 x
cos x
xt nguyn hm J =
sin
x.dx
=

d(cos x)
cos x cos2 xdx =
1 cos2 x
1 + cosx
r
Z
Z
Z
t
2t + 1 1
t

dt =
dt
t cos x = t
J=
.dt =
2
1+t
t
+
t
2
t 2 + t

Z
Z
p

d(t 2 + t)
1 1 p
dt
2 + t + ln t + + t 2 + t +C

=
+
t


1
1
2
2
(t + 2 )2 4
2 t2 + t

1
th vo tnh c I1 = 2 ln(3 + 2 2) = 2 ln( 2 + 1)
2
Cu IV. (1 im)
Trong mt phng (P), cho tam gic ABC vung ti A, AB = a, AC = b v M l trung im ca BC. Trn ng
thng d i qua M v vung gc vi (P) ly im S (S 6= M).
Mt phng (Q) cha BC v vung gc vi (SAB),
2
ab 2
ct SA ti D, bit th tch ca khi t din ABCD bng
. Tnh di ca on SM.
24
Gii .

D l giao im ca SA v mp(Q) BD l giao tuyn ca (Q) v (SAB)


K MHBD , H BD th MHmp(SAB)
ABMH m ABSM nn ABmp(SMN) vi N l giao im ca SH v AB SNAB
Mt khc ,MA = MB = MC, SM(ABC) nn SA = SB = SC
AC b
Do vy N l trung im ca AB MN =
=
2
2
Cu V. (1 im)
x
y
z
Cho cc s thc dng x, y, z thay i. Tm gi tr ln nht ca biu thc P = p
+p
+p
2
2
2
3x + yz
3y + zx
3z + xy
Gii .
yz
1
zx
xy
1
1
1
3
1
1
t a = 2 ; b = 2 ; c = 2 abc = 1 v P =
+
+
+
+
+

x
y
z
3+a
3+c 3+a 3+b 3+c 4
3+b
1
1
1
3
Ta chng minh
+
+

quy ng, nhn cho c:3(a + b + c) + 5(ab + bc + ca) 24 0


3+a 3+b 3+c 4
3
iu ny ng theo CoSi (ch abc = 1) Kt lun: maxP =
2
Cu VIa.1 (1 im)
Trong mt phng vi h ta Oxy cho ba im I(1; 1), E(2; 2), F(2; 2). Tm ta cc nh ca hnh vung
ABCD, bit I l tm ca hnh vung, AB i qua E v CD i qua F.
Gii .
dongoc_nam
ng thng AB c phng trnh dng: a(x + 2) + b(y 2) = 0
(a2 + b2 > 0) ng thng CD c phng
trnh dng: a(x 2) + b(y + 2) = 0
t iu kin khong cch t I n AB v CD bng nhau ta tm c a = b (trng hp a = b loi.)
Khi tm c phng trnh AB : x y + 4 = 0,CD : x y 4 = 0.
Phng trnh BC v DA c dng (d) : x + y + c = 0.

t iu kin khong cch t I n (d) bng khong cch t I n AB = 2 2 ta tm c c = 2, c = 6


T tm ra ta 4 nh hnh vung
THANHHUE67
Goi E 0 i xng ca E qua I E 0 (4; 0) thuc CD Goi F 0 i xng ca F qua I F 0 (0; 4) thuc AB
AB qua E v F 0 , AB : x y + 4 = 0 CD qua E 0 v F,CD : x y 4 = 0
Gi I 0 l hnh chiu ca I xung AB, I 0 (1; 3) A thuc AB v AI 0 = II 0 A, I 0 l trung im AB
B, I l trung im AC v BD B v D
Cu VIa.2 (1 im)
Trong khng gian vi h ta Oxyz, cho A(3; 3; 1), B(0; 2; 1) v (P) : x + y + z 7 = 0.
Vit PT ng thng d nm trong (P) sao cho mi im ca d cch u hai im A, B.
Tm ta im C trn d sao cho din tch tam gic ABC nh nht.
Gii .

Gi s M l im bt k thuc d v M( ; ; 7 ).
T MA = MB ( 3)2 + ( 3)2 + (6 )2 = 2 + ( 2)2 + (6 )2 = 7 3 .

x = t
Do M( ; 7 3 ; 2 ), t y ta c phng trnh ca ng thng (d) : y = 7 3t .

z = 2t

Ly C(t; 7 3t; 2t) (d) AC = (t 3; 4 3t; 2t 1), BC = (t; 5 3t; 2t 1).


1

1 p

Tip theo tnh c [AC, BC] = (1 2t; 6t 3; 10t 15) v SABC = [AC, BC] =
5(28t 2 68t + 47).
2
2
17
17 47 17
Do SABC t GTNN khi v ch khi 28t 2 68t + 47 t GTNN, tc l t =
; 14 ; 7 ).
Suy ra C( 14
14
Cu VIIa. (1 im)



z 1 z 3i
=
=1
Tm s phc z tha mn ng thi hai iu kin
zi z+i
Gii .
Vit li hai pt di dng sau
|z 1| = |z i| |z|2 (z + z) + 1 = |z|2 + i(z z) + 1 z + z = i(z z)
|z 3i| = |z + i| |z|2
+ 3i(z z) + 9 = |z|2 + i(z z) + 1 4i(z z) = 8 z z = 2i
z + z = 2
Thay vo trn ta c
z = 1 + i.
z z = 2i

Cu VIb.1 (1 im)
Cho parabol (P) : y2 = x v hai im A(9; 3), B(1; 1) thuc (P). Gi M l im thuc cung AB ca (P) ( phn
ca (P) b chn bi dy AB). Xc nh ta ca im M trn cung AB sao cho tam gic MAB c din tch ln
nht.
Gii .
t AB : x 2y 3 = 0
im M (P) M(t 2 ;t) t gi thit suy ra 1 < t < 3
|t 2 2t 3|

tam gic MAB c din tch ln nht d[M; AB]max


m d[M; AB] =
5
n y ta tm GTLN ca |t 2 2t 3| trong (1; 3) bng 4 khi t = 1 M(1; 1)
Cu VIb.2 (1 im)
Trong khng gian vi h ta Oxyz, cho hai mt phng (P) : x + y 5 = 0; (Q) : y + z + 3 = 0 v im A(1; 1; 0).
Vit phng trnh ng thng d vung gc vi giao tuyn ca (P) v (Q), ng thi ct (P), (Q) ln lt ti M, N
sao cho A l trung im ca MN.
Gii .

Trc ht ta tnh
nP = (1; 1; 0),
n
Q = (0; 1; 1) [nP , nQ ] = (1; 1; 1).
Gi thit M (P) M( ; 5 ; ) N(2 ; 3; ).

V N nm trong mp(Q) nn 3 + 3 = 0 = . Do MN = 2(1 ; 4; )

Theo gi thit ta suy ra ba vct


nP ,
n
Q , MN ng phng
5
2


nP , n
MN = (2; 7; 5).
hay MN.[
Q ] = 0 (1 ; 4; ).(1; 1; 1) = 0 =
3
3
x1 y1 z
=
= .
Cui cng phng trnh ng thng cn tm l
2
7
5
Cu VIIb. (1 im)

z z
Tm s phc z tha mn ng thi hai iu kin |z| = 1 v | + | = 3.
z z
Gii .

thaydoip1


2 2



z
z
+ = 3 z + z = 3 |z2 + z2 | = 3 |(z + z)2 2| = 3
z = a + bi
z.z = |z|2 = 1
z z
z.z
"

(2a)2 2 = 3

|(2a)2 2| = 3

(2a)2 2 = 3
ltdtrangt z = cos x + i sin x
(r = |z|2 = 1)
z = cos(x) + i sin(x)

2
2
2
... |z + z2 | = 3
p dng CT Moivre z v z

3
| cos 2x + i sin 2x + cos(2x) + i sin(2x)| = 3 |2 cos 2x| = 3 | cos 2x| =
2
Ta c argument v module ca s phc z

TNG HP P N THI TH S 06 CA MATH.VN


Cu I.1 (1 im)
Cho hm s y = x3 3x + 2
Gii

(1). Kho st s bin thin v v th (C) ca hm s (1).

4
3

n
g

2
1

Ch
u

g
c

Cu I.2 (1 im)

Tm tt c cc im M (C) tip tuyn ti M ct (C) ti im N vi MN = 2 6.


Gii
Gi s tip im l M(x0 ; y0 ), giao im ca tip tuyn vi th (C) l N(x; y).
y y0
Khi hsg ca tip tuyn l:
= x2 + x.x0 + x02 3 y y0 = (x2 + x.x0 + x02 3)(x x0 )
(1).
x x0
Mt khc hsg tip tuyn bng y0 (x0 ) = 3(x02 1)
(2).
2
2
Li t gi thit ta c: (y y0 ) + (x x0 ) = 24
(3).
2
2
2
T (1), (2), (3) ta c: (x x0 ) [9(x0 1) + 1] = 24 v (x x0 )(x + 2x0 ) = 0,(x 6= x0 ),
4
suy ra 9x02 [9(x02 1)2 + 1] = 24.
t x02 = t ta c 9t[9(t 1)2 + 1] = 24 t =
3
Cu II.1 (1 im)
Phng trnh 2(sin x + 1)(sin2 2x 3 sin x + 1) = sin 4x. cos x c bao nhiu nghim trong khong ( ; ).
Gii
thaydoip1
2 (sin x + 1) sin2 2x 3 sin x + 1 = sin 4x. cos x = 2 sin x. cos x. cos 2x. cos x




(sin x + 1) 4sin2 x. 1 sin2 x 3 sin x + 1 = 2 sin x. 1 2sin2 x 1 sin2 x
a = sin x x ( ; )
Phng trnh c dng (a + 1)(4a2 .(1 a2 ) 3a + 1) = 2a.(1 2a2 )(1 a2 )
a = 1 sin x = 1 x" = 2

10a3 8a5 +a2 4a4a4 +1 = 0 (a + 1)2 (1 2a)3 = 0


x = 6
1
1
a=
sin x =

2
2
x = 56
fanarsenal_vietnam


1 cos 4x
3 sin x + 1 = sin 4x. cos x (sin x + 1)(3 6 sin x cos 4x) = sin 4x. cos x
2(sin x + 1)
2
(sin x + 1)(3 6 sin x) sin x. cos 4x cos 4x = sin 4x. cos x 3(1 2 sin2 x) 3 sin x = sin 5x + cos 4x
x
9x
x

3"cos 2x + 3 cos(x + 2 ) = cos(5x 2 ) + cos 4x 3.2. cos( 3x


2 + 4 ). cos( 2 4 ) = 2. cos( 2 4 ). cos( 2 4 )
cos( 2x 4 ) = 0
2x 4 = 2 + k x = 32 + k2 x = 2

9x
3x
9x
3
3 3x
3 cos( 3x
2 + 4 ) = cos( 2 4 ) 3 cos( 2 + 4 ) = cos( 2 + 4 ) 4 cos ( 2 + 4 ) = 0 (?)

2k
(?) 3x
2 + 4 = 2 + k x = 6 + 3
Vy x = 2 , x = 6 , x = 56
1

Cu II.2 (1 im)
Tm iu kin ca tham s m phng trnh sau c 5 nghim phn bit
32x (2x2 + 2m + 1)3x + m(2x2 + 1)3x + m(4x2 + 1) = 0.
Gii
http://math.vn/showthread.php?t=1899

(1 im)

Cu III.

Tnh tch phn sau I =

Gii
Vit li: I =

x2 d(x tan x)
+
(x tan x)2


Z
3
x2 3
2
=

x tan x
6
6

x + tan x
dx +
x tan x

x tan x
x tan x

2

dx.

x + tan x
dx
x tan x


 3
Z 

3
x
2x
x2
dx +
1 dx =
x
x tan x
x tan x
x tan x

6
Z

Cu IV. (1 im)

Cho t din ABCD c DADBDC v DA = DB = a, DC = a 3. T mt im M bt k trong tam gic ABC k


MA0 (DBC), MB0 (DCA), MC0 (DAB).
Hy tm v tr ca im M t din MA0 B0C0 c th tch ln nht v tnh gi tr ln nht theo a.
Gii
dng pp ta ha.

D trng O, A thuc tia Ox, OA = a, B thuc tia Oy, OB = a,C thuc tia Oz, OC = a 3

x y
z
Ta A(a, 0, 0); B(0, a, 0);C(0, 0, a 3), nn pt theo mt chn ca mp(ABC)
+ + =1
a a a 3
Gi M(x0 , y0 , z0 ) l mt im trong tam gic ABC.(0 < x0 < a, 0 < y0 < a, 0 < z < a 3)
d( M, (OAB)) = z0 ; D( M, (OBC)) = x0 ; d( M, (OAC)) = y0
x0 .y0 .z0
D dng chng minh c MA0 MB0 MC0 Do V( MA0 B0C0 ) =
6
r
x
.y
.z
z0
x0 y0
0
0
0
Do M thuc tam gic ABC + + = 1 1 3 3
3
a
a
a 3
a 3

6
3
3
a 3
a 3
a 3

3 x0 y0 z0 x0 .y0 .z0
VMA0 B0C0
3
27
162

a
a 3
a3 3
t c khi x0 = y0 = ; z0 =
tc M l trng tm tam gic ABC
Vy max =
VMA0 B0C0
162
3
3
Cu V. (1 im)
Vi mi k Z+ , k hiu mk l s b nht trong cc phn t ca tp {|k n(1 + sqrt(7))| : n = 0, 1, 2, ..., k}.
Hy tm tt c cc s thc r sao cho mk < r vi mi k Z+ .
Gii
http://math.vn/showthread.php?p=6833

Cu VIa.1

(1 im)
2

Trong Trong mt phng h trc ta Oxy cho im A(1; 1)


v hai ng thng (d) : x y + 3 = 0 v (d 0 ) : x y + 1 = 0.
Hy lp phng trnh cc cnh ca tam gic u ABC, bit rng B (d) v C (d 0 ).
Gii
B (d) B(t;t + 3) v C (d 0 ) C(m; m + 1)

suy ra AB = (t + 1;t + 2), AC = (m + 1; m), BC =(m t; m t 2)


(t + 1)2 + (t + 2)2 = (m + 1)2 + m2
tam gic ABC u khi AB = AC v AB = BC
(t + 1)2 + (t + 2)2 = (m t)2 + (m t 2)2

(1)
(2)

T (1) (t + 1)2 m2 + (t + 2)2 (m + 1)2 = 0 (t + 1 m)(t + 1 + m) (t + 1 m)(t + 3 + m) = 0


(t + 1 m)(2t + 2m + 4) = 0 t = m 1 hay t = m 2
n y thay vo pt (2) cn li tm c t v m suy ra A, B,C
Cu VIa.2 (1 im)
x1 y+1 z1
x1 y1 z
Trong khng gian vi h ta Oxyz cho hai ng thng (d1 ) :
=
=
v (d2 ) :
=
= .
2
1
2
2
2
1
Lp phng trnh mt cu tip xc vi (d1 ) v (d2 ) ti A v B tng ng sao cho AB l ng knh ca mt cu.
Gii
Trc ht ta c nhn xt:
* c (d1 ), (d2 ) cho nhau.
* AB chnh l on vung gc chung ca hai ng thng cho nhau (d1 ), (d2 ).
+ Trn (d1 ), (d2 ) ln lt ly 2 im M(1 + 2t; 1 + t; 1 2t), N(1 2t 0 ; 1 + 2t 0 ;t 0 ).

+ Tm t,t 0 sao cho MN vung gc vi cc vect ch phng ca (d1 ), (d2 ).


+ n y th Ok ri.
Cu VIIa. (1 im)
Tm tt c cc s phc z tha mn phng trnh (z + i)4 + (z i)4 = 2(z2 + 1).
Gii
nguyentatthu
PT [(z + i)2 + (z i)2 ]2 2(z2 i2 )2 = 2(z2 + 1)
4(z2 1)2 2(z2 + 1)2 = 2(z2 + 1)

2z4 14z2 = 0 z = 0; z = 7
thaydoip1
z + i = a, z i = b, ab = z2 + 1; (a b)2 = 4, a4 + b4 = 2ab
2

2
a2 + b2 2a2 b2 = 2ab (a b)2 + 2ab 2a2 b2"= 2ab

2+1 = 8 z = 7
ab
=
8

z
(2ab 4)2 2a2 b2 = 2ab a2 b2 9ab + 8 = 0
ab = 1 z2 + 1 = 1 z = 0

Cu VIb.1 (1 im)
Trong mt phng h trc ta Oxy cho ng trn (C) : x2 2x + y2 3 = 0.
Gi B,C l giao im ca ng thng () : x + y 3 = 0 vi ng trn (C).
Hy tm cc im A trn ng trn (C) sao cho tam gic ABC c chu vi ln nht.
Gii .

Bng cch gii h to bi PT (C) v () ta tm c cc giao im B(3; 0) v C(1; 2).


V B;C c nh nn chu vi ca tam gic ABC ln nht khi v ch khi
L = AB + AC ln nht .
x = 1 + 2 sin
Vit li (C) : (x 1)2 + y2 = 4 , do A(x; y) (C) nn [0; 2 ) :
.
y = 2 cos
q
p

2
2
T L = (2 sin 2) + 4 cos + 4 sin2 + (2 cos 2)2 = 2 2( 1 sin + 1 cos )
q
p
p

L 2 2 (1 + 1)(1 sin + 1 cos ) = 4 2 2 sin( + 4 ) 4 2 + 2

1 sin = 1 cos
sin = cos
Du bng xy ra

= 54
sin( + ) = 1
sin( + ) = 1
4
4

Khi A(1 2; 2). Ta i n kt lun :Chu vi tam gic ABC nh nht khi v ch khi A(1 2; 2)
Cu VIb.2 (1 im)
Trong khng gian vi h ta Oxyz cho mt phng (P) : 4x 7y + z + 25 = 0 v ng thng
x+1 y z1
= =
vi A l giao im ca (d1 ) v mp(P). Lp phng trnh ng thng (d2 ) i qua A, nm
(d1 ) :
1
2
1
trong mp(P) v to vi (d1 ) v hnh chiu vung gc ca (d1 ) ln mp(P) nhng gc bng nhau.
Gii
http://math.vn/showthread.php?t=1907

Cu VIIb. (1 im)
Cho a, b l cc s phc v phng trnh az2 + bz + 2010 = 0 c hai nghim z1 , z2 .
Chng minh rng nu |z1 | = |z2 | th a.b = |a|.b.
Gii
http://math.vn/showthread.php?t=1908

p n thi th s 5 - Din n Ton Hc

1 of 9

http://math.vn/showthread.php?t=1789

Cu I
1. Bn c t gii
3x + 2
= x x = 1 hoc x = 2
x+2
gi ta A( 1; 1), B(2; 2) th AB=3\sqrt(2)
2. Ta A,B l nghim PT :

do C, D thuc ng thng d: y = x + m v CD / AB nn m q0 v CD = AB = 32
3x + 2
PT hong giao im ca d v (C)
= x + m x 2 + (m 1)x + 2m 2 = 0 (*)
x+2
d ct (C) ti 2 im p/b th = m 2 10m + 9 > 0 m > 9 hoc m < 1
Gi ta C(a, a + m), D(b, b + m), vi a, b l nghim PT (*)
CD = 32 2(a b) 2 = 32 m 2 10m + 9 = 9 m = 0 ( loi ) hoc m = 10 ( tha mn)

Cu II
sin3x + sin5x 0
k
1. K :
sin4x 0 x
4
.
cosx 0
PT (sinx cosx) 2 (1 + 2sin2x) = 2sin4x(cosx sinx)
sinx cosx = 0 (1) hoc cosx sinx)(1 + 2sin2x) = 2sin4x (2)

(1) x = + l , loi
4
(2) cosx sinx + sin3x + sinx + cos3x cosx = 2sin4x

2sin 3x + = 2sin4x

4
3 l2

x = + l2 ( loi ) hoc x =
+
4
28
7
kt hp vi K ta c
3 l2
x=
+
vi l 7m 3 , l, m Z
28
7
2.

2/23/2010 5:42 PM

p n thi th s 5 - Din n Ton Hc

2 of 9

ngoi ra , PT 2x + 3 + x = x 2 3 2x + 3 + 2x + 3 +

http://math.vn/showthread.php?t=1789

1
1
= x2 + x +
4
4

1 2
1 2
2x + 3 + = x +

2
2
n y c th d dng gii tip hai PT cn c bn

Cu III
t 1 + x + 1 x = t
2 + 21 x 2 = t 2
2

(t 2 2)
4
2(t 2 2).2t
2xdx =
dt
4
2xdx = t.(t 2 2)dt
i cn x = 0 t = 2
x = 1 t = 2
2 lnt.t(t 2 2)
I =
dt
2
t
2
2
=
2 (t 2)lntdt
2
1 3
1 3
|2 2 1 3
.1
=
2 lntd 3 t 2t = 3 t 2tlnt 2 2 3 t 2t t dt
2 1
4
22

t 2 2dt
= ln2
22ln(2)

2
3
3
3

4
22
1
2
= ln2 +
ln2 t 3 2t|2
9

3
3
2
8 22

= ln2.(2 2)
2(2 2)
3
9

2
28 162
= ln2.(2 2) +
3
9
1 x2 =

Cu IV.

2/23/2010 5:42 PM

p n thi th s 5 - Din n Ton Hc

3 of 9

http://math.vn/showthread.php?t=1789

Gi H l hnh chiu ca S trn y v I, K, J tng ng l hnh chiu ca H trn AD, DC, CB khi gc to bi cc mt bn
^
^
^
^
^
^
(SAD), (SDC), (SCB) vi y tng ng l SIH , SKH , SJ H v SI H = SKH = SJ H = 60 0 (gt) . T 3 tam gic vung
SH I, SHK, SH J bng nhau v ta thu c H cch u cc cnh AD, DC, CB , nh th H l giao im ca cc ng phn
^
^
gic trong ca 2 gc ADC v DCB.

ng cao hnh thang ABCD l DE = AC 2 (2CD) 2 = a3 , ta s c


DE
a3
3a
^
^
^
^
tan DAB =
= 3 DAB = 60 0 ADC = 120 0 H DK = 60 0 H K = DK.tan(60 0 ) =
SH = H Ktan(60 0 ) =
AE
2
2
DE(AB + DC)
2
SABCD =
= 23a
2
1
Vy VS.ABCD = SH.S ABCD = 3a 3
3

Cu V
Cch 1t t=1/z th bi ton tr thnh
Cho x,y,t >0 tha mn : .x 2 + y 2 + t 2 + 2xyt = 1. Chng minh rng
xy + yt + tx + 1 14

x yt + x + y + t 13
T gi thit ta c 0 < x, y, t < 1

t x = cosA, y = cosB vi A, B 0; . K tr thnh :


2
cos 2 A + cos 2 B + t 2 + 2tcosAcosB = 1
(t + cosAcosB) 2 (1 cos 2 A)(1 cos 2 B) = 0
(t + cosAcosB) 2 sin 2 A}sin 2 B = 0
(t + cos(A + B))(t + cos(A B)) = 0

Do A, B 0; nnt + cos(A B) > 0 nn t = cos(A + B) = cosCtrong A, B, C l 3 gc trong mt tam gic.


2
A B
B C
C A
A
B
C
1
Mtan tan + tan tan + tan tan = 1 tan 2 tan 2 tan 2
2
2
2
2
2
2
2
2
2 27
A
B
C
A
B
C
27sin 2 sin 2 sin 2 cos 2 cos 2 cos 2
2
2
2
2
2
2
27(1 x)(1 y)(1 t) (1 + x)(1 + y)(1 + t)

2/23/2010 5:42 PM

p n thi th s 5 - Din n Ton Hc

4 of 9

http://math.vn/showthread.php?t=1789

27(1 x)(1 y)(1 t) (1 + x)(1 + y)(1 + t)


14(x + y + t + xyt) 13(1 + x y + yt + tx)
x y + yt + tx + 1 14

xyt + x + y + t 13
1 1
Du = xy ra khi x = y = =
z 2

Cch 2
1
t t = , th th t gi thit ta c x 2 + y 2 + t 2 + 2x yt = 1, suy ra 0 < x, y, t < 1. Ngoi ra cng theo bt php t ny th
z
bt ng thc cn chng minh c th c vit li thnh
xy + yt + tx + 1 14
,
x + y + t + xyt 13
tng ng
14(x + y + t) + 14x yt 13 + 13(xy + yt + tx).
Thay 2xyt = 1 x 2 y 2 t 2 vo ta c
14(x + y + t) + 7(1 x 2 y 2 t 2 ) 13 + 13(xy + yt + tx),
hay
1 + xy + yt + tx 7(x + y + t 1) 2 . (*)
3
t a = 1 x > 0, b = 1 y > 0, c = 1 t > 0 v p = a + b + c. D dng chng minh c 1 < x + y + t , cho nn
2
3
p < 2.
2
Thc hin php i bin nh trn, ta c th vit li gi thit x 2 + y 2 + t 2 + 2xyt = 1 thnh (2 p) 2 = 2abc. Bn cnh , bt
ng thc (*) tr thnh
4 2 p + ab + bc + ca 7(2 p) 2 .
(a + b + c)(ab + bc + ca)
S dng bt ng thc c bn abc
, ta suy ra
9
2 p(ab + bc + ca)
(2 p) 2
,
9
hay
9(2 p) 2
ab + bc + ca
.
2p
V nh th ta ch cn chng minh c
9(2 p) 2
7(2 p) 2 .
4 2p +
2p
Bt ng thc ny tng ng vi
(2 p)(2 p 3)(7 p 6)
0.
2p

Cu VIa
1.Trong mt phng Ox y, cho ng trn (C): x 2 + y 2 4x 6y + 12 = 0 c tm I v ng thng d : x + y 4 = 0 . Tm trn
ng thng d im M sao cho tip tuyn ca (C) qua M tip xc vi (C) ti A, B v tam gic I AB c din tch ln nht.
Li gii
Vit li (C) : (x 2) 2 + (y 3) 2 = 1 suy ra (C) c tm I(2; 3) , bn knh R = 1.
1
1
1
1
^
^
dt(I AB) = I A.I B.sin AI B = R 2 .sin AIB R 2 =
2
2
2
2
1

^
^

max dt(I AB) = sin AI B = 1 AIB = khi M AIB l hnh vung cnh I A = 1 nn M I = 2 ( )
2
2
3 3
V M d nn M(x; 4 x) v do ( ) (x 2) 2 + (1 x) 2 = 2 x =
2
3 + 3 5 3
3 3 5 + 3
Vy c 2 im M cn tm l M
;
v M
;
tho bi ton .
2
2
2
2
z 1 4
= 1.
2z i

Cu VIIa.Cho z1 , z2 , z3 , z4 l cc nghim phc ca phng trnh:


Tnh (z12 + 1)(z22 + 1)(z32 + 1)(z42 + 1).
Li gii
z 1 4
Ta c
= 1 (z 1) 4 (2z i) 4 = 0.
2z i

t f (z) = (2z i) 4 (z 1) 4 f (z) = 15(z z1 )(z z2 )(z z3 )(z z4 ).


f (i). f ( i)
V z12 + 1 = (z1 + i)(z1 i) nn (z12 + 1)(z22 + 1)(z32 + 1)(z42 + 1) =
.
225
Cui cng tnh f (i) = (i 1) 4 i 4 = 5, f ( i) = ( i 1) 4 ( 3i) 4 = 85,
17

2/23/2010 5:42 PM

p n thi th s 5 - Din n Ton Hc

5 of 9

http://math.vn/showthread.php?t=1789

suy ra (z12 + 1)(z22 + 1)(z32 + 1)(z42 + 1) =

17
.
9

2.Trong khng gian vi h ta Oxyz, cho im M (4; 3; 2)v hai ng thng:


x = 2 t
x = 2 + t '

d1 : y = 3 2t
v d2 : y = 1 2t '

z = 1 + 2t
z = 2 + t '
Vit phng trnh ng thng d qua M ct d1 , d2 ln lt ti A, B sao cho M A = 2M B
Li gii
A(2 t; 3 2t; 1 + 2t), B(2 + t '; 1 2t '; 2 + t ')

M A = ( 2 t; 2t; 1 + 2t), M B = ( 2 + t '; 4 2t '; 4 + t ')


Yu cu bi ton

M A = 2 M B hoc M A = 2 M B
2 t = 4 + 2t '
2 t = 4 2t '

2t = 8 4t ' hoc 2t = 8 + 4t '

1 + 2t = 8 + 2t '
1 + 2t = 8 2t '
t = 3
t = 5

1 hoc
1
t ' = 2
t ' = 2

t = 3

x4 y3 z+2

=
=
Vi
1 M A = ( 5; 6; 7) d :
5
6
7
t
'
=

t = 5

x4 y3 z+2

Vi
=
=
1 M A = (3; 10; 9) d :
3
10
9
t ' = 2

Cu VI.b
1.Trong mt phng Ox y, gi (C) l ng trn ngoi tip tam gic ABC vi A(2; 2), B(4; 0), C(3; 2 1) v ng thng
d : 4x + y 4 = 0. Tm trn d im M sao cho tip tuyn ca (C) qua M tip xc vi (C) ti N sao cho din tch tam gic N AB
ln nht.
Li gii
Cch 1
ng thng AB : x y 4 = 0
(C) : x 2 + y 2 + 2ax + 2by + c = 0; (a 2 + b 2 c > 0)
Ta c h phng trnh :
4a 4b + c + 8 = 0
a = 3

8a
+
c
+
16
=
0
b = 1

c = 8
6a + 2(2 1)b + c + 12 22 = 0
Ta tm I(3; 1) thuc AB nn AB l ng knh ca (C) , nn im N thuc (C) tha iu kin din tch tam gic N AB
ln nht N l trung im cung AB
ng thng I N : x + y 2 = 0)
y = 2 x
x = 4
x = 2
Ta im N tha h phng trnh

hoc
Vy N(4; 2) hoc N(2; 0)
2
2
y = 2
y = 0
(x 3) + (y + 1) = 2
Vi N(4; 2) phng trnh tip tuyn ti N : x y 6 = 0 Ta im M tha :
x y 6 = 0
x = 2

M(2; 4)

4x + y 4 = 0 y = 4
Vi N(2; 0) phng trnh tip tuyn ti N : x y 2 = 0 Ta im M tha :
6

x = 5
x y 2 = 0
6
4

M ;

5
5
4
4x + y 4 = 0
y = 5
Cch 2
d dng kim tra tam gic ABC vung ti C hay AB l ng knh ng trn ngoi tip tam gic ABC.
Gi H l hnh chiu ca N trn AB th
1
1
AB 2
SN AB = AB.N H AB.R =
2
2
4
Du = xy ra khi N l trung im dy AB hay tip tuyn ti N song song vi AB

C AB = (2; 2)
Gi a l tip tuyn qua N th pt ca a c dng x-y+c=0

2/23/2010 5:42 PM

p n thi th s 5 - Din n Ton Hc

6 of 9

http://math.vn/showthread.php?t=1789

Gi a l tip tuyn qua N th pt ca a c dng x-y+c=0


| 3 + 1 + c|
a l tip tuyn nn d(I, a) = R
= 2 c = 2 hoc c = 6
2
ta c pt ca a l x y 2 = 0 hoc x y 6 = 0
6
4
t tm c N l giao ca a v d l M(2, 4) hoc M ;
5
5
2.Trong khng gian vi h ta Oxyz, cho tam gic ABC c im A(2; 1; 1) trung tuyn CM v ng cao BH c phng
x = 1
z
x3 y5

=
=
v y = 3 + t . Vit phng trnh cc cnh ca tam gic ABC.
trnh ln lt l
7
2
3

z = 4 + t
Li gii

2/23/2010 5:42 PM

p n thi th s 5 - Din n Ton Hc

7 of 9

http://math.vn/showthread.php?t=1789

Cu VII.b
z+i

2/23/2010 5:42 PM

p n thi th s 5 - Din n Ton Hc

8 of 9

http://math.vn/showthread.php?t=1789
6

z + i
Gii phng trnh trn tp s phc:
=1.
z i
Li gii
Cch 1
z + i6
Ta c
= 1 (z + i) 6 = (z i) 6 .
z i
Trc ht nhn xt rng nu z0 0 l mt nghim ca pt th z0 ,
t a = (z + i) 2 , b = (z i) 2 a + b = 2z 2 2, ab = (z 2 + 1)
v pt thnh (a b)(a ab + b)(a + ab + b) = 0.

1
1
, cng l nghim ca pt.
z0
z0

TH1. a = b (z + i) 2 = (z i) 2 z = 0.
TH2. a ab + b = 0 2z 2 2 (z 2 + 1) = 0 z = 3.
Theo nhn xt trn pt cho c 5 nghim l 0, 3, 3,

1
3

1
3

Cch 2

2/23/2010 5:42 PM

p n thi th s 5 - Din n Ton Hc

9 of 9

http://math.vn/showthread.php?t=1789

2/23/2010 5:42 PM

Phng trnh honh giao im ca hai ng l:


x3 3mx2 + (m 1)x + m + 1 = 2x m 1
(x 1)(x2 + (1 3m)x 2m 2) = 0 ()
t x = t 1 phng trnh () tr thnh (t 2)(t 2 (1 + 3m)t + m 2 = 0)(**)
t d ct (Cm ) ti ba im phn bit c honh ln hn 1
khi v ch khi pt() c 3 nghim phn bit ln hn 1
tng ng vi pt () c 3 nghim phn bit t > 0
m () t = 2vt 2 (1 + 3m)t + m 2 = 0
do yu cu bi ton t vi tm m g(t) = t 2 (1 + 3m)t + m 2 = 0 c hai
nghim dng phn bit khc 2
= 9m2 + 2m + 9 > 0, P = m 2 > 0, S = 1 + 3m > 0, g(2) = 5m 6= 0
Gii h ta c m > 2
Bi ton trn c chuyn v mt 0 do li gii n gin hn.

LI GII THI TH S 04 CA MATH.VN


Cu I.1 (1 im)
Cho hm s y = x3 3mx2 + (m 1)x + m + 1 c th l (Cm )
Kho st s bin thin v v th (C1 ) khi m = 1

n
v
.
h

Li gii
m = 1 hm s y = x3 3x2 + 2 Tp xc nh D = R
o hm y0 = 3x2 6x = 3x(x 2)
Bng bin thin
x
y0

+ 0

+
+

t
a
+

y % & 2 %
th
2
1

m
1
2

Cu I.2 (1 im)
Tm tt c cc gi tr ca m d : y = 2x m 1 ct th (Cm ) ti ba
im phn bit c honh ln hn 1.
Li gii 1

Li gii 2
Phng trnh honh giao im l: x3 3mx2 + (m 1)x + m + 1 = 2x m 1
(x 1)(x2 + (1 3m)x 2m 2) = 0
Gi f (x) = x2 + (1 3m)x 2m 2.
d : y = 2x m 1 ct th (Cm ) ti ba im phn bit ta cn c
f (x)
= 0 c 2 nghim a, b phn bit khc 1
= (1 3m)2 4(2m 2) > 0
(3m + 1 )2 + 80 > 0
3
9

m 6= 0
f (1) = 1 + (1 3m) 2m 2 6= 0
5m 6= 0

a > 1
a + 1 > 0
a + 1 + b + 1 > 0

Theo bi
b > 1
b + 1 > 0
(a + 1)(b + 1) > 0

a + b + 2 > 0
a + b = 3m 1

(?)
m
ab + a + b + 1) > 0
ab = 2m 2

3m 1 + 2 > 0
3m + 1 > 0
nn (?)

m>2
2m 2 + 3m 1 + 1 > 0
m 2 > 0
Vy m > 2 tha mn yu cu bi.

Cu II.1 (1 im)
Gii phng trnh : sin 3x = cos x cos 2x(tan 2x + tan2 x).

Cu III. (1 im)
Xt hnh phng (H) b chn pha di bi Parabol (P) : y = x2 v pha trn
bi ng thng i qua A(1; 4) c h s gc k. Tm k (H) c din tch
nh nht.

n
v
.
h

Li gii
K cos x 6= 0 v cos 2x 6= 0

Li gii
PT ng thng i qua A(1; 4) v c hsg k l (d) : y = k(x 1) + 4
Honh giao im ca (P) v (d) l x2 = k(x 1) + 4 x2 kx + k 4 = 0
c = k2 4k + 16 > 0 vi mi k do (d) lun ct (P) ti hai im phn bit

Gi x1 , x2 l hai nghim ca pt. Gi s x1 < x2 th x2 x1 = k2 4k + 16


Z x2
3
1 p 2
2
Din tch ca (H) bng S =
k 4k + 16
(kx k + 4 x2 )dx =
6
x1

2
2
M k 4k + 16 = (k 2) + 12 12 do min S = 4 3 t c khi k = 2


sin 2x sin2 x
+
sin 3x = cos x cos 2x
cos 2x cos2 x
sin 3x cos x = sin 2x cos2 x + cos 2x sin2 x
1 + cos 2x
1 cos 2x
sin 3x cos x = sin 2x
+ cos 2x
2
2
sin 4x + sin 2x = sin 2x(1 + cos 2x) + cos 2x(1 cos 2x)
sin 2x cos 2x + cos2 (2x) cos 2x = 0 sin 2x + cos 2x = 1


Cu II.2

(1 im)

y(1 + 2x3 y) = 3x6


Gii h phng trnh :
1 + 4x6 y2 = 5x6

Li gii

t
a

1
D thy x 6= 0 chia c hai v ca hai pt trong h cho x6 v t t = 3 ta c
x

y.t 2 + 2y2t = 3
y.t(t + 2y) = 3

t 2 + 4y2 = 5
(t + 2y)2 4yt = 5

SP = 3
t S = t + 2y v P = y.t ta c h
S2 4P = 5

1
gii h ta c S = 3, P = 1 suy ra t = 1, y = 1 hoc t = 2, y =
2
q
3 1 1
t ta thy h c hai cp nghim l (1; 1) v ( 2 ; 2 ).
2

Cu IV. (1 im)
Cho hnh chp tam gic u S.ABC, y c cnh bng a. Gi M, N ln lt
l trung im ca cc cnh SA, SC. Tnh th tch ca hnh chp S.ABC bit
BM vung gc vi AN.

Li gii
Gi O l tm tam gic u ABC, I l trung im AC nn SO mp(ABC).

t
a = AB, b = AC,
c = AS v |
c | = x, khi y |
a | = | b | = a.


1

Ta c 2.AN = b +
c v BM = AM AB =
c
a.
2
Theo gi thit
 th


2
1

( b + c ).
c a = 0 b .
c 2.
a . b +
c 2.
a .
c = 0 (1).
2
Do cc cnh bn to vi cc cnh y nhng gc bng nhau
AI
a
v cos =
=
nn
AS 2x

a2
a
6
a
42
(1) a2 + x2 a2 = 0 x =
SO =
.
2
2
6

1 a 42 a2 3 a3 14
Do VS.ABC =
=
3 6
4
24

Cu V. (1 im)

Cho x, y, z l cc s thc dng tho x + y + z = 1. Tm gi tr ln


nht ca biu thc:
r
r
r
xy
yz
zx
P=
+
+
.
x + y + 2z
y + z + 2x
z + x + 2y

Li gii 1
Theo BT Cauchy th:

1
1

( x + y + 2 z).(
+ ) 4.
x+ z
y+ z
p

M li c x + y + 2 z 4(x + y + 2z).

xy
xy
xy
1
Do
(
+ ).
y+ z
x + y + 2z 2 x + z

1
1

T :V T ( x + y + z) = .
2
2

n
v
.
h

t
a

Li gii 2
t x = a2 ; y = b2 ; z = c2 Khi ta c:
x + y + 2z = a2 + b2 + 2c2 c2 + ab + ac + bc = (c + a)(c + b)
bc
ac
ab
Do : P p
+p
+p
(c + a)(c + b)
(a + b)(a + c)
(b + a)(b + c)
p dng BT Bunhiacopski:


bc
ac
ab
2
P (ab + bc + ac)
+
+
(c + a)(c + b) (a + b)(a + c) (b + a)(b + c)
1
M: ab + bc + ac
3
(a + b)(b + c)(c + a) = ab(a + b) + ac(a + c) + bc(b + c) + 2abc
3
ab(a + b) + ac(a + c) + bc(b + c) (a + b)(b + c)(c + a)
4
13 1
1
2
Vy P
= P
34 4
2

Cu VIa.1 (1 im)
Xc nh ta cc nh
 gic ABC bit M(1; 4), N(1; 3) l trung
 ca tam
1 5
;
l trc tm tam gic ABC.
im ca BC,CA v H
3 3

Li gii 1

ng cao CH i qua H v nhn MN(2; 1) lm vct php tuyn


CH : 2x + y + 1 = 0 ta c C(t; 2t 1).
V B v A i xng vi C qua M v N B(2 + t; 9 2t) v A(2 + t; 7 2t)

AH(t 73 ; 26
3 2t) CB(2t + 2; 10 4t)

m HA v CB vung gc vi nhau nn ta c
(t 37 )(2t + 2) + ( 26
3 2t)(10 4t) = 0
41
15t 2 86t + 123 = 0 t1 = 3 hay t2 =
15
TH1: t = 3 th A(1; 1), B(5; 3),C(3; 5)
41
23
71 53
41 67
th A( 11
TH2: t =
15 ; 15 ), B( 15 ; 15 ), C( 15 ; 15 )
15
Li gii 2

x = t

T gi thit suy ra MN CH v NM = (2; 1) nn c pt (CH) :


y = 1 + 2t

Gi C(t; 1 + 2t) A(t 2; 7 2t)

HA = (t 73 ; 26
3 2t) v CM = (t + 1; 5 2t).

Do HA.CM = 0 (t 73 )(t + 1) + ( 26
3 2t)(5 2t) = 0
2
15t 86t + 123 = 0 t = 3 hay t = 41
15
41 67
71 53
11 23
Kt lun C(3; 5) B(5; 3), A(1; 1) v C( 15 ; 15 ) B( 15 ; 15 ), A( 15
; 15 ).
Cu VIa.2 (1 im)
Trong khng gian vi h ta Oxyz cho (P) : x y + z + 1 = 0 v ba im
A(1; 1; 1), B(0; 1; 2),C(2; 0; 1). Tm N (P) sao cho:
2NA2 + NB2 + NC2 = 8.
Li gii


Gi I im tha mn iu kin 2IA + IB + IC = 0 I(0; 43 ; 54 )
3
H l hnh chiu ca I trn (P) IH 2 = d 2 (I; (P)) =
4
9 2 5
37
2
2
IA = ; IB = ; IC =
8
8
8
Theo bi : 2NA2 + NB2 + NC2 = 8


2(NI + IA)2 + (NI + IB)2 + (NI + IC)2 = 8



4NI 2 + 2IA2 + IB2 + IC2 + 2NI(2IA + IB + IC) = 8
15
1
4NI 2 +
= 8 NI 2 =
2
8
5
1
2
2
2
NH + IH = NH =
8
8
Vy khng tn ti im N tha mn yu cu bi ton
Cu VIIa. (1 im)
Mt hp ng 9 tm th c nh s t 1 n 9.
Hi phi rt t nht bao nhiu th xc sut c t nht mt th ghi s chia
5
ht cho 4 phi ln hn .
6

n
v
.
h

t
a

Li gii 1
Gi Pn (A) l sx rt t nht c mt th ghi s chia ht cho 4 t n ln rt
Gi Pn (B) l sx ko rt c th ghi s chia ht cho 4 t n ln rt
R rng Pn (A) + Pn (B) = 1
Vi n = 8 hoc n = 9 th Pn (B) = 0
Vi n 7 th cc kh nng c th rt c n s u ko chia ht cho 4
t 7 s ko chia ht cho 4:
An
An
An7 Pn (B) = n7 Pn (A) = 1 7n
A9
A9
n
A
5
Cn tm s n nh nht tha 1 n7 >
A9 6
S: n = 6

2
Khi k = 0; P(A) = 0 ; khi k = 1; P(A) = (khng tha )
9
Khi 2 k 9
C1 .Ck1 +C22 .C7k2 5
>
P(A) = 2 7
6
C9k
12.7!
6.7!
5.9!

+
>
(k 1)!(8 k)! (k 2)!(9 k)! k!(9 k)!
2
1
60

+
>
k 1 9 k k(k 1)(9 k)
2k(9 k) + k(k 1) > 60 k2 17k + 60 < 0 5 < k < 12 5 < k 9
Vy phi rt t nht l 6 th

Li gii 2
Gi k l s th rt ra 0 k 9 k N
v P(A) l xc sut rt it nht ra 1 th c s chia ht cho 4
4

Cu VIb.1 (1 im)
ng trn (C) ni tip hnh vung ABCD c phng trnh:
(x 2)2 + (y 3)2 = 10. Xc nh ta nh cc ca hnh vung, bit
cnh AB i qua M(3; 2) v xA > 0.

Li gii.
PT ng thng AB i qua M(3; 2) c dng ax + by + 3a + 2b = 0.

ng trn (C) c tm I(2; 3) v bn knh R = 10 nn

|2a + 3b + 3a + 2b|

10 =
10(a2 + b2 ) = 25(a + b)2
2
2
a +b
(a + 3b)(3a + b) = 0 a = 3b hay b = 3a
Do PT AB : x 3y 3 = 0 hoc AB : 3x y + 7 = 0.
TH1. AB : x 3y 3 = 0, gi A(3t + 3;t) t > 1 v do IA2 = 2.R2 = 20
nn (1 + 3t)2 + (t 3)2 = 20 10t 2 + 10 = 20 t = 1 hay t = 1
Suy ra A(6; 1) C(2; 5) v B(0; 1) D(4; 7).
TH2. AB : 3x y + 7 = 0, gi A(t; 3t + 7) t > 0 v do IA2 = 2.R2 = 20
nn (t 2)2 + (3t + 4)2 = 20 10t 2 + 20t + 20 = 20 t = 0 hay t = 2 (Khng
tha mn)

17
|4t + 15| = 19 t = 1 hoc t =
2

x = 3 + 3t

Cu VIb.2 (1 im)
Trong khng gian vi h ta Oxyz cho ba im
A(2; 1; 0), B(0; 4; 0),C(0; 2; 1) v ng thng
y+1
z2
x1
=
=
. Lp phng trnh ng thng vung gc
d:
2
1
3
vi mt phng (ABC) v ct ng thng d ti im D sao cho bn im
19
A, B,C, D to thnh mt t din c th tch bng .
6

n
v
.
h
Vi t = 1 D(3; 0; 5) :

Vi t =

Li gii 1

x = 1 + 2t

y = 2t

z = 5 4t

x = 16 + 3t

17
47
19
D(16; 19
2 ; 2 ) : y = 2 + 2t
2

z = 47 4t
2

Cu VIIb. (1 im)
2y + 3 2x 3 = 2
Gii h phng trnh:

2x+y + 22xy 2) = 4(2y + 1) log 2 2


log
3 (2
9
3

y = 1 + t v gi D(1 + 2t; 1 + t; 2 + 3t).

z = 2 + 3t

Hn na AB = (2; 3; 0), AC = (2; 1; 1) [AB, AC] = (3; 2; 4)


Li gii
1

1
3
3
SABC = . [AB, AC] =
29.
K x , y
2
2

2
2
Phng trnh mp(ABC) vi vtpt
n = (3; 2; 4) nn c pt 3x + 2y 4z 8 = 0.
T pt th hai ca h ta c 22x+y + 22xy 2 = 22y+1
Gi h l khong cch t D n mp(ABC), khi y
22x+y1 + 22xy1 1 = 22y
|3(1 + 2t) + 2(1 + t) 4(2 + 3t) 8| |4t + 15|

h=
=
t u = 22x+y1 +,v = 22xy1 th ta c pt:
9 + 4 + 16
29
u
3.VABCD
19
u + v 1 = uv + v2 v = u (u + v)(v 1) = 0
Mt khc h =
= ,
v
SABC
29
v = 1 2x y 1 = 0 2x 1 = y

17
47
thay
vo
pt
u
ca
h
ta
c
pt
2y
+
3

y2 = 2
;

).
suy ra |4t + 15| = 19 t = 1 hay t = (D(3; 0; 5) hay D(16; 19
2
2
2

Gi thit mp(ABC) u = (3; 2; 4). Do c hai pt ng thng tha mn gii pt ny ta c y = 3 hay y = 11


Do h c hai cp nghim (2; 3) , (6; 11)
z + 47
x3 y z5
x + 16 y + 19
2
2
:
= =
v :
=
=
3
2
4
3
2
4
Pt tham s ng thng (d) :

t
a

Li gii 2

AB = (2; 3; 0); AC = (2; 1; 1); [AB, AC] = (3; 2; 4)

phng trnh ng thng c VTCP


u = (3; 2; 4)

Gi D(1 + 2t; 1 + t; 2 + 3t); AD = (2t 1;t 2; 3t + 2)


1

19
VABCD = [AB, AC]AD =
6
6

p n thi th s 03 ca Math.vn - Din n Ton Hc

http://math.vn/showthread.php?t=1468

THI TH S 03
Din n Ton Hc MATH.VN
(Thi gian: 180 pht)
I. PHN CHUNG CHO TT C TH SINH (7,0 im)
Cu I (2,0 im)
4x + 3
.
1 2x
1. Kho st s bin thin v v th (C) ca hm s .
2. Chng minh rng tn ti mt php i xng trc bin (C) thnh chnh n .
1.Bn c t gii.
2. Chng minh rng tn ti mt php i xng trc bin (C) thnh chnh n .
Li gii 1
3
Xt ng thng d : 2x + 2 y + 3 = 0 y = x , ta s chng minh d : (C) (C)
2
5
Hm s cho vit li thnh y = 2 +
.
1 2x
1
5
( x0 .
(C) l tp hp cc imM(x0 ; y0 ) vi y0 = 2 +
2
1 2x0
3
Gi H a; a d l hnh chiu ca M trn d ta c

2

3
5

M H .ud = 0 1.(a x0 ) + 1. a + 2
=0
2
1 2x0

5
x0 1
a= +
2 4 2(1 2x0 )
bi: Cho hm s y =

( ud = ( 1; 1) l VTCP ca d)
5
x0 7
5
x0 1

Do H +
; +
2 4 2(1 2x0 )
2 4 2(1 2x0 )
N l im i xng ca M qua d khi v ch khi H l trung im ca
M N , v th :
1
5

xN = 2xH xM = 2 1 2x
0

y = 2y y = x 3
0
H
M
N
2
Ta c :
5
5
3
2 +
= 2+
=

= yN
0
1 2xN
2
1 1 + 1 102x
0
ng thc ny chng t N (C)
Nh vy vi mi im M (C) , im N i xng vi M qua d
cng thuc (C) . iu ny chng t c php i xng trc d bin
(C) thnh chnh n (pcm)
Li gii 2 (Hng dn )
Thc hin php di trc di h ta Oxy v h ta mi (I X Y )
y = Y 2

qua php di trc:


1 Trong h ta mi (C)
x = X + 2

1 of 10

2/23/2010 11:51 AM

p n thi th s 03 ca Math.vn - Din n Ton Hc

http://math.vn/showthread.php?t=1468

5
2X
n y ta d dng chng minh c Y = X l trc i xng ca (C)
5
Vy qua php i xng trc d vi d : y = x bin (C) thnh chnh n.
2
Bnh lun:
*) chng minh hnh (H) c trc i xng l ng thng d ta ly im
M bt k thuc (H) ri chng minh im N i xng vi M qua d thuc (H)
*) Bn c c th dng nh ngha ca php i xng trc tm ra (C)
ch c hai trc i xng c PT : d1 : 2x 2y 5 = 0 v d2 : 2x + 2y + 3 = 0
*)i vi cc hm phn thc hu t (nht trn nht hoc hai trn nht )
th th s nhn hai ng thng l cc ng phn gic ca gc to bi 2
tim cn lm trc i xng . Chng hn vi bi trn (C) c TC : 2x 1 = 0
, TCN : y + 2 = 0 nn cc ng phn gic ca gc hp bi 2 ng tim
2x 2y 5 = 0(d1 )
|2x 1| | y + 2|
=

cn c PT l :
2
1
2x + 2y + 3 = 0(d2 )
Do (C) c hai trc i xng l d1 v d2
*)Bi tp lm thm :
1)Chng minh rng tn ti 2 php i xng trc bin th (C) ca
x2 + x + 1
hm y =
thnh chnh n .
x+2
2)Chng t rng hai im trn hai nhnh ca Hypebol
x 2 2x + 2
(H) : y =
c khong cch ngn nht khi n l giao
x+1
im ca (H) v cc trc i xng
ca (H).
Cu II (2,0 im)
Cu 2 (2 im)
8
1, Gii phng trnh : 3cotx tanx = 8sin x .

3
Li gii
K : sinx.cosx 0 sin2x 0
3cosx sinx
2
= 8sin x 2
PT

sinx cosx
3
2
2
3cos x sin x
2
= 8sin x

sinxcosx
3
(3cosx sinx)(3cosx + sinx) = 2sin2x(3cosx + sinx)
T xy ra 2 TH:

+) 3cosx + sinx = 0 tanx = 3 x = + k


3

+)3cosx sinx = 2sin2x sin2x = sin x x = + k2 hoc

3
3
4
2
x=
+k
9
3
Kt hp cc trng hp trn v i chiu K ta c nghim ca PT ban u l :

4
2
x = + k v x =
+k
(k )
3
9
3
Bnh lun:
*)PT trn thuc dng c bn , mu cht l phn tch nh cng thc cng
c phng trnh: Y =

2 of 10

2/23/2010 11:51 AM

p n thi th s 03 ca Math.vn - Din n Ton Hc

http://math.vn/showthread.php?t=1468

*)PT trn thuc dng c bn , mu cht l phn tch nh cng thc cng
cung v hng ng thc bin i v PT tch . Vic kim tra K c th th
trc tip nghim tm c vo K hoc biu din cc cung trn cng ng
trn lng gic loi nghim.
*)Bi tp lm thm : Gii cc phng trnh sau :
1) sin2x = 2tanx + tan 2 x
sin 3 xsin3x + cos 3 xcos3x
1
2)
=

8
tan x 6 tan x + 3
3

2. Gii bt phng trnh : 4x + 6 x 3 + 7x 2 + 12x + 6 x 2 2.


Li gii
3
K :x
2
3
BPT (x + 2) 2 (x 2 2) (x + 2) (x + 2) 3 (x 2 2) (x + 2) x 2 2
t
3
3
A = 4x + 6 + x + 2; B = (x 3 + 7x 2 + 12x + 6) 2 + (x + 2) x 3 + 7x 2 + 12x + 6 + (x + 2) 2
3
ta c A > 0; B > 0 (ch B l bnh phng thiu ca t ng )
2
(x + 2) 2 (x 2 2) (x + 2) 2 (x + 2) 3 (x 2 2) (x + 2) 3
x2 2
Khi BPT
+
A
B
3
1 1
(2 x 2 ) + + 1 0 2 x 2 0 2 x 2 (tho mn Kx )
A B

2
Vy nhim ca BPT l 2 x 2
Bnh lun:
*) BPT c gii nh php nhn chia vi lng lin hp da trn vic tch s
hng vng l biu thc cha bin s v c th y l dng mi ca lp PT
hoc BPT v t .
*)Bi tp lm thm : Gii bt phng trnh sau :
3
2x 3 + 4x 2 + 4x 16x 3 + 12x 2 + 6x 3 4x 4 + 2x 3 2x 1
Th th vi x

x
2 e sinx
Cu 3 (1 im): Tnh tch phn I =
0 1 + sin2x dx.
Li gii 1
e x sinx
1 e x (sinx + cosx) e x (cosx sinx)
dx =
dx
(sinx + cosx) 2
2
(sinx + cosx) 2

1 (e x ) '(sinx + cosx) e x (sinx + cosx) '


ex

dx
=
d

2
2
(sinx + cosx)
2(sinx + cosx)

ex

| 2 e 2 1
Nn I =
0 = 2
2(sinx + cosx)
Li gii 2

x
2 e .cosx

dx, khi
t J =
0 1 + sin2x

ex
2

I+J =
dx v
0 sinx + cosx
x

1
2 e .(sinx cosx)

2 e x .d
IJ =
dx
=
2
0 (sinx + cosx)
0
sinx + cosx

ex
ex
1
|02 2
= e 2 1 (I + J ) I = .e 2 1
=
dx
0 sinx + cosx
sinx + cosx
2
Bnh lun:

3 of 10

2/23/2010 11:51 AM

p n thi th s 03 ca Math.vn - Din n Ton Hc

http://math.vn/showthread.php?t=1468

Bnh lun:
*)Da trn cng thc tch phn tng phn ta c th to ra rt nhiu bi

ton nh
uvdx + vdx = (uv)dx = uv + C
*)Bi tp lm thm : Tnh cc tch phn
1 (x + 1)e x dx
1) I =
0 (x + 2) 2

2 (1 + x)sinx + (1 x)cosx
2) I =
dx
0
1 + sin2x
Cu 4 (1 im) : Cho t din gn u ABCD (c cc cp cnh i bng nhau)
v mt phng () lun song song vi AB v CD . Tm v tr ca ()
() chia t din thnh hai phn c th tch bng nhau .
Li gii
Gi s m p() ct cc cnh ca t din nh hnh v.

V cc mt ca t din c din tch bng nhau nn cc ng cao bng nhau.


Mt phng () song song vi AB v CD nn FG||H E||DC
v EG||FH || AB.
Ta cn tm v tr ca m p()
VF.H DCE + VF.GCE = VF.AEGB + VF.AH E VF.ADC 2.VF.AH E = VF.ABC 2.VF.GEC
S
1 2. SGEC
V
V
S

F.AH E
F.GEC
FDC
ABC
VA.FDC . 1 2.
= VA.FBC . 1 2.

S
AH
E
VF.AH E
VF.ABC
S FBC 1 2.

S AH E
EC 2
1

2.
EA
DF
H D AD 2 2.H D 2
3
=
=

2
2
2 (H D H A) = 0.
FB
AH AD 2.AH

1 2. AH
AD
Kt lun: Mt phng i qua trung im ca cnh AD.
Bnh lun:
4 of 10

2/23/2010 11:51 AM

p n thi th s 03 ca Math.vn - Din n Ton Hc

http://math.vn/showthread.php?t=1468

Bnh lun:
y l bi ton tnh t s th tch kt hp vi vic dng thit din song
song vi cp ng thng cho nhau , v vy i hi nhiu k nng dng
hnh v tnh ton bin i .
Cu 5 (1 im) . : Cho cc s dng a, b, c. Chng minh rng
23a
6b
6c
5 3 .
+
+
(a + b)(a + c) (b + a)(b + c) (c + a)(c + b)
Li gii 1
BT cho tng ng vi
2a
2 3 b
2 3 b
5. ( )
+
+
(a + b)(a + c) (b + a)(b + c) (c + a)(c + b)
p dng BT Cauhy ta c
a
a
2a

+
(1)
a + b a + c (a + b)(a + c)
2 3 b
b
3b

+
(2)
b + a b + c (b + a)(b + c)
c
3c
2 3 b

+
c + a c + b (c + a)(c + b)(3)
Cng theo v ca (1), (2), (3) ta thu c
a b
3b c
3c
a
+
+
+
+
+
= 5 (pcm)
V T ( )
a + b a + c b + a b + c c + a c + b
Li gii 2
Ta c:
b b
2 3 a
c c
VT =
+ 6.
+ 6.
c+a c+b
(a + b)(a + c)
b+a b+c

b
2 3 a
c
+ 6.
+
b+a c+a
(a + b)(a + c)

= 23 xy + 62 x 2 y 2 3(x 2 + y 2 ) + 3(2 x 2 y 2 + 3) = 53.


b
a
, y=
.
b+a
b+a
Bnh lun:
y l bi BT 3 bin khng kh , vn l vn dng mt cch kho lo cc
BT c bn th mi t c pcm .
II. PHN RING (3,0 im) Th sinh ch c lm mt trong hai phn: A hoc B.
Phn A
Cu 6a (2 dim)
1,Trong mt phng vi h to Oxy cho ng thng : x + y 2 = 0
v ng trn (T) : x 2 + y 2 2x + 2y 7 = 0. Chng minh rng
ct (T) ti hai im phn bit A, B v tm to im C trn (T)
Vi: x =

sao cho tam gic ABC c din tch bng (3 + 2)7 .


Li gii
PT (T) : (x 1) 2 + (y + 1) 2 = 9 c tm I(1; 1) v bn knh R = 3.
|1 1 2|
Ta c d(I; ) =
= 2 < 3 = R nn
1 + 1
ct (T) ti hai im phn bit A, B
Gi s C(x; y) (T) nn (x 1) 2 + (y + 1) 2 = 9(1)
1
1
Ta c S ABC = d(C; ).AB = d(C; ).R 2 d 2 (I; ) = 7.d(C; )
2
2
|x + y 2| 1
5 of 10

2/23/2010 11:51 AM

p n thi th s 03 ca Math.vn - Din n Ton Hc

Li c :d(C; ) =

http://math.vn/showthread.php?t=1468

|x + y 2| 1
|(x 1) + (y + 1) 2|
=
2
2

1
(|1.(x 1) + 1.(y + 1)| + 2)
2
1 2

2
2
2

1 + 1 . (x 1) + (y + 1) + 2 = 3 + 2 (do(1))
2

Do S ABC = (3 + 2)7 d(C; ) = 3 + 2

(x 1) 2 + (y + 1) 2 = 9

x 1 y + 1

=
1
1
((x 1) + (y + 1))( 2) 0

x = 1 2


y = 1 3

2
3
3

Vy to im C cn tm l C 1 ; 1 .

2
2
Bnh lun:
y l bi ton quen thuc v ng thng v ng trn ,
nm vng v tng giao ca ng thng v ng trn l
gii quyt c bi ton .
2, Trong khng gian vi h to Oxyz cho ng thng
z = 1 + t

d : y = 2t
. Vit phng trnh mt phng

z = 1 t .
() cha d sao cho khong cch t im M(2; 0; 1) n
() bng 2 .
Li gii
PT m p() : ax + b y + cz + d = 0 (a 2 + b 2 + c 2 0) , t gi thit
ta c
a + 2b c = 0

a c + d = 0

2
2
2
2
(2a + c + d) = 4(a + b + c )
2b = c a

d = c a
2
2
4a 6ac + c = 0
2b = d = c a

c = (3 + 5)a

c = (3 5)a.
Chn a = 1 ta thu c 2 PT mt phng cn lp l
() : 2(2 + 5)x y + 2(1 + 5)z 2 = 0 v
() : 2(2 5)x y + 2(1 5)z 2 = 0
Cu 7a (1 im) : Cho cc s phc p, q (q 0).
2

6 of 10

2/23/2010 11:51 AM

p n thi th s 03 ca Math.vn - Din n Ton Hc

http://math.vn/showthread.php?t=1468

Chng minh rng nu cc nghim ca phng trnh x 2 + px + q 2 = 0


p
c mun bng nhau th l s thc .
q
Li gii 1
z1 + z2 = p
Gi z1 = a + bi, z2 = c + di l hai nghim ca PT. Th th theo vi t:
2
z1 .z2 = q
p 2 (z1 + z2 ) 2 z12 + z22
=
+2
2 =
z 1 z2
q
z 1 z2
Do |z1 | = |z2 | = a 2 + b 2 = c 2 + d 2 = k
z 2 + z22
z1 .z2 + z2
.z1 2(ac + bd)
+ 2 =
=
+2
Nn 1
k
k
z 1 z2
2(ac + bd)
2
Ta c: |ac + bd | (a 2 + b 2 )(c 2 + d 2 ) = k
k
p
p2
Do : 2 = m 0 = m l s thc.
q
q
Li gii 2
Gi z1 , z2 l hai nghim ca PT x 2 + px + q 2 = 0, suy ra z1 .z2 = q 2 .
t k = |z1 | = |z2 | q.q = k 2 v z1
.z1 = k 2 .
p z1 q
Mt khc do q 0 nn z1 0 v z12 + p.z1 + q 2 = 0 = + .
q q z1
2
q.z1 q z1
p
z1 q
k
+ 2 = + = z . Do z , pcm.
t z = th z = + =
q z1 q.z
q
1 k
z1 q
Bnh lun:
chng minh s phc l s thc ta c th
1) Chng minh phn o bng 0
2) Chng minh bnh phng s khng m
3) Chng minh s phc l lin hp ca chnh n
PHN B
Cu 6b (2 dim)
1, Trong mt phng vi h to Oxy cho hai im
A(1; 1) v B(4; 3) .Tm to cc im C v D sao cho
ABCD l hnh vung .
Li gii

x = 4 4t
Ta c AB = (3; 4) nn PT (BC) :
y = 3 + 3t.
t = 1
Gi C(4 4t; 3 + 3t) v c BC 2 = AB 2 = 25 16t 2 + 9t 2 = 25
t = 1
Do c hai im C(0; 6) v C(8; 0) tha mn.
1 5
Suy ra trung im ca AC tng ng l I ; D( 3; 2)
2 2
9
1
v I ; D(5; 4).
2
2
Vy ta c C(0; 6) v D( 3; 2) hoc C(8; 0) v D(5; 4)
Bnh lun:
Bi ton ny kh c bn , c th c nhiu cch gii mu cht l
yu t cc cnh vung gc v bng nhau trong hnh vung m thi .
2,Trong khng gian vi h to Oxyz cho ng thng
x1 y z+1
:
= =
v mt phng () : x + 2y 2z 1 = 0. Vit phng trnh mt
7 of 10

2/23/2010 11:51 AM

p n thi th s 03 ca Math.vn - Din n Ton Hc

http://math.vn/showthread.php?t=1468

x1 y z+1
= =
v mt phng () : x + 2y 2z 1 = 0. Vit phng trnh mt
1
2
1
phng () cha
v to vi () mt gc nh nht .
Li gii

ng thng qua M0 (1; 0; 1) v c VTCP u ( 1; 2; 1). Gi n (a; b; c)(


a 2 + b 2 + c 2 0) l VTPT ca () .
V () cha nn ()qua M0 , do () : a(x 1) + by + c(z + 1) = 0
:


v n u n . u = 0 a + 2b + c = 0 c = a 2b
() : ax + by + (a 2b)z 2b = 0
Theo cng thc tnh gc gia 2 mt phng , ta c gc
to bi v () c cho bi :
|1.a + 2.b 2.(a 2b)|
cos =
2
2
1 + 4 + 4a 2 + b + (a 2b)
|a 6b|

=
(0 < )
2
2
2
2
3a + b + (a 2b)
| 7 a 10 b + 13 (a 2b)|
| 6
|
6
6
Ta c :cos =
2
2
3a 2 + b + (a 2b)

7 2 10 2 13 2 2
2
2
6 + 6 + 6 .a + b + (a 2b)
318

=
2
2
2
18
3a + b + (a 2b)
318
T cos ln nht bng
hay nh nht bng
18
a
b
a 2b
318
arccos
khi v ch khi 7 = 10 = 13

18
6 6
6
7
a=
b
10
n y ta chn b = 10 th a = 7 v ta thu c mt phng ()cn lp c PT:
7x + 10y 13z 20 = 0
Bnh lun:
Hin nay SGK khng a phng trnh tng qut ca ng thng vo bi hc
v phng php chm mt phng na . V vy khi gp cc bi ton kiu
''Vit phng trnh mt phng () i qua ng thng d v tho mn
mt iu kin no " , ta c th lm nh sau :
+)Gi s () : ax + by + cz + d = 0
M0 ()
VTPT (a; b; c) (a 2 + b 2 + c 2 0). V () qua d nn :
n . u = 0

(Vi M0 d v u l VTCP ca d)
Gii 2 trong 4 tham s a, b, c, d theo 2 tham s cn li ta thu c
d ch theo 2 tham s (chng hn l a v b)
+) S dng thm iu kin th 2 ca bi
ton tm rng buc gia a v b t thu c PT()
Cu 7b (1 dim) .
log2009 2010
log2010 2009
Gii phng trnh : (1 + x 2 + x)
(1 + x 2 x)
+ 2x = 0.
Li gii 1
Xt hm f (x) = 1 + x 2 + x trn

8 of 10

2/23/2010 11:51 AM

p n thi th s 03 ca Math.vn - Din n Ton Hc

http://math.vn/showthread.php?t=1468

f (x) = 0 v nghim kt hp vi tnh lin tc ca hm v f (0) = 1


nn f (x) > 0 x
t t = f (x), t > 0, phng trnh cho tr thnh :
1
1
t log2009 2010 log 2009 + t = 0
t 2010
t
1

log2009 2010

1 log2009
+ t =
t

2010

1
+
t

Xt h(t) = V T
h = log2009 2010t log2009 2010 1 + 1 > 0 t > 0
Xt g(t) = V P
1

1
1 log2009 2010 1 1
2010
< 0 t > 0
g =
t 2 t 2
log2009
t
T h(t) = g(t) c ti a mt nghim v li c h(1) = g(1)
nn t = 1 l nghim duy nht ca pt
x + 1 + x 2 = 1 x 2 + 1 = x 2 2x + 1 x = 0
Th li ta thy x = 0 tho mn PT v l nghim ca PT cho.
Li gii 2
t u = x 2 + 1 + x; v = x 2 + 1 x; = log2009 2010 > 1
1

Khi PT tr thnh : u v + u v = 0 u v = v u

1

Li t y = v ta c PT :u y = y u( )
Do = log2009 2010 > 1 nn
+Nu u > v th V T( ) > 0 > V P( )
+Nu u < v th V T( ) < 0 < V P( )
1

Suy ra : u = y = v . Khi x 2 + 1 + x
log2010 2009
log210 2009 1
= ( x 2 + 1 + x)
1 = ( x 2 + 1 + x)
x 2 + 1 + x = 1 ... x = 0
Li gii 3
Gi s x l nghim ca PT v k hiu ( ) l PT cho,
ta c x 2 + 1 > |x| x x 2 + 1 x > 0
v log2009 2010 > 1 > log2010 2009 > 0 . T y
+)Nu x > 0 th x 2 + 1 + x > 1 ( x 2 + 1 + x)
log2010 2009

log2009 2010

log2010 2009

> ( x 2 + 1 + x)
> ( x 2 + 1 x)
V T( ) > 0 ( )khng c tho khi x > 0
+)Nu x < 0 th 0 < x 2 + 1 + x < 1 ( x 2 + 1 + x)
log

2009

log

log2009 2010

2009

2010
2010
< ( x 2 + 1 + x)
< ( x 2 + 1 x)
V T( ) < 0 ( ) khng c tho khi x < 0

9 of 10

2/23/2010 11:51 AM

p n thi th s 03 ca Math.vn - Din n Ton Hc

http://math.vn/showthread.php?t=1468

+) Nu x = 0 th ( ) c tho .
Vy PT cho c nghim duy nht x = 0
..................Ht.................

10 of 10

2/23/2010 11:51 AM

p n thi s 2 ca math.vn - Din n Ton Hc

http://math.vn/showthread.php?t=1537

LI GII THI TH S 02.


Din n Ton Hc MATH.VN
(Thi gian: 180 pht)
I. PHN CHUNG CHO TT C TH SINH (7,0 im)
Cu I (2,0 im) Cho hm s y = x 4 2x 2 + 3 c th (C).
1. Kho st v v th hm s.
2. Vit phng trnh tip tuyn tip xc vi (C) ti hai im phn bit.
Li gii
Gi s y = ax + b(d) l tip tuyn cn tm
Phng trnh honh giao im x 4 2x 2 + 3 = ax + b x 4 2x 2 ax + 3 b = 0
V (d) tip xc vi (C) ti hai tip im nn pt trn phi c vit di dng
x 4 2x 2 ax + 3 b = (x + m) 2 (x + n) 2
Khai trin v ng nht h s ta c a = 0, b = 2
Bnh lun:
Trc ht v mt th th c th hnh dung l hm trng phng m c 3 cc tr th lun c mt
im cc tr nm trn Oy , hai im cc tr cn li i xng nhau qua Oy , v vy ta trc quan thy
rng tip tuyn tip xc vi th ti hai im phn bit chnh l ng thng qua 2 im cc tr i
xng nhau qua Oy. i vi bi ton cho th 2 ci im cc tr y l A( 1; 2) v B(1; 2) TT
cn lp l ng thng AB : y = 2 (Ch A, B cng c tung 2) .
Nhng l trc quan thi , cn gii c th nh sau :
D nhin cc tip tuyn ca cc hm s cp THPT(c dng y = f (x)) khng c dng cng phng Oy
, v vy nu gi d l tip tuyn (TT) cn lp tip xc vi (C) ti hai im phn bit A(a; f (a)) v
B(b; f (b))th c th gii s sng phng a < bcng khng nh hng tnh tng qut bi ton .
Khi y h s gc (hsg) ca d cng l hsg ca AB , cng l hsg ca TT ca (C) ti A v cng l hsg ca
TT ca (C) ti B .V th c
f (a) f (b)
= 4a 3 4a
f (a) f (b)

= f '(a) = f '(b) a b
ab
3
3
4a 4a = 4b 4b
Do a < b nn bin i cht c ngay a = b, thay vo ng thc trn dn ti
4a 3 4a = 0 a = 0; a = 1; a = 1(ch hm chn nn f (a) = f ( a)) . Ch c a = 1 l tho
a < bv a = bv t b = 1 . Ta thu c A( 1; 2) v B(1; 2) AB : y = 2 hay d : y = 2
Note: Ni chung bi ton trn i hi cng thc ct tuyn v tip tuyn v bin i HPT .
f (a) f (b)
+Ch rng nu A(a; f (a)); B(b; f (b))(a b) th hsg ca ng thng AB l
ab
+Bi tp luyn thm : Vit PT tip tuyn ca th (C) : y = x 4 4x 3 + 3 bit tip tuyn tip xc (C)
ti 2 im phn bit . Tm to cc tip im .
Cu II (2,0 im)
1. Gii phng trnh sau trn : 2cosx + tanx = 1 + 2sin2x.
Li gii
iu kin cosx 0.
Khi 2cosx + tanx = 1 + 2sin2x 2cosx 1 = 4sinxcosx

sinx
cosx

cosx(2cosx 1) = sinx(2cosx + 1)(2cosx 1)


2cosx 1 = 0

sinx cosx + 2sinx.cosx = 0.

x = 3 + k2
TH1. 2cosx 1 = 0
,k

1 of 8

2/23/2010 10:55 AM

p n thi s 2 ca math.vn - Din n Ton Hc

http://math.vn/showthread.php?t=1537

TH1. 2cosx 1 = 0

,k

x = + k2
3

TH2. sinx cosx + 2sinx.cosx = 0 (cosx sinx) 2 + (cosx sinx) 1 = 0

x = 4 + 2m

cos x + =
= cos
, m\in ZZ.

4
22

x = + 2m
4

2 x + y + 1 + 1 = 2 x + 2 y
.
2. Gii h phng trnh sau trn :
+
2 x 1 2 x 2 y = 1
Li gii
(1 2 x ).(2 y 1) = 1
2 + 2 x .2 y = 2 x + 2 y

H PT tng ng vi
2 2 2y = 2 x
1 2 x = 2 2 y 1.
t u = 2 x , v = 2 y , u, v > 0 ta c PT (v 2 1).(v 1) = 1 v 2 1 = v.
Thay vo ta c u + v = 1 2 + uv = 1 uv = 1 khng tha mn.
Vy h PT cho v nghim.
Cu III (1,0 im) Cho f (x) l hm c o hm trn (0; ) v tha mn f '(x).sinx = x; x (0; ).
2.

Tnh I = f f .
3
3
Li gii:
2

x
3
3 x dx
Gi thit ta c f '(x) =

f '(x)dx =

sinx
3
3 sinx
2

x
3
3 dx
3 x
3 dx
hay I = f f =
dx =
3 sinx 3 sin( x) d( x) = 3 sinx I
3
3 3 sinx
2

I=

3 dx
3 d(cosx)
1 cosx 2

| 3 = ln3.
=
=
ln

2
2 3 sinx 2 3 cos x 1 4 1 + cosx 3
2

Cu IV (1,0 im) Cho t din S.ABC c SA (ABC) v nh din cnh SB vung, vi


^

^
SB = 2; BSC = ; ASB = . Tm gc phng ca nh din cnh SC c ln l .
4
3
Li gii:

2 of 8

2/23/2010 10:55 AM

p n thi s 2 ca math.vn - Din n Ton Hc

http://math.vn/showthread.php?t=1537

K AF SB AF mp(SBC)
BC mp(SAB) AB BC, SB BC SBC vung
cn ti B.
Do AB = 2sin, SA = 2cos, BC = 2 AF = 2.sin.cos.
2.sin
.
1 + sin 2
Trong mp(SBH) k FK || BH FK mp(SAC). Suy ra gc gia hai na mt phng (SCB) v (SCA)
K BH AC BH mp(SAC) v BH =
^

bng gc AFK .
p dng nh l Talet ta c

FK SF SF.SB SA 2
=
=
=
= cos 2 FK = BH.cos 2 .
BH SB
SB 2
SB 2

AF = 2FK = 2.BH.cos 2
3
2
2.sin
.cos 2 1 + sin 2 = 2.cos cos = .
2.sin.cos = 2.
1 + sin 2
5
^

Gi thit AFK =

Cu V (1,0 im) Cho x; y + thay i tha (1 + 2x). 1 + = 5


y

52y
.
Tm gi tr nh nht ca P = x.(5 + 5)
1 + y
Li gii:

t x = tanu; y = cotv; u; v 0; c (1 + 2tanu).(1 + 2tanv) = 5 t c tan(u + v) = 2.


2
u + v 1 + cos(u + v) 5 + 5
u + v 5 5
Cng c lun cos 2
=
=
v sin
=
.

2
2
2
10
10
v
u u+
u + v
2

Ta c tanu tan
+
2 cos 2 u + v
2
u
+
v
u
+
v u + v
v
v cosv cos
.sin
.

2
2 2
u + v
u + v
u + v
u + v
u + v
Do : cos 2
.tanu sin
.cosv sin
.cos
cot
.
2
2
2
2
2
5 + 5
u + v
u + v
5 + 5
10
y
=
Xt cos 2
.tanu sin
.cosv =
.x
.
.P.

2
2
5 5 1 + y
10
10
u + v
u + v
u + v
5 + 35
Cn sin
.cos
cot
=
.
2
2
2
10
5 + 35
3 5
3 + 5
T P
du bng xy n khi u = v x =
;y=
.
5 + 5
2
2
5 + 35
Vy gi tr nh nht ca P l
.
5 + 5
II. PHN RING (3,0 im) Th sinh ch c lm mt trong hai phn: A hoc B.
Phn A
Cu VIa (2,0 im)
1. Trong (Oxy) cho hai ng thng m : mx + y m 1 = 0 v m' : x my 3 m = 0, (vi m l
tham s thc). Chng minh rng vi mi m hai ng thng lun ct nhau ti 1 im nm trn
3 of 8

2/23/2010 10:55 AM

p n thi s 2 ca math.vn - Din n Ton Hc

http://math.vn/showthread.php?t=1537

tham s thc). Chng minh rng vi mi m hai ng thng lun ct nhau ti 1 im nm trn
mt ng trn c nh.
Li gii I:

Hai ng thng trong ln lt c hai php tuyn l n = (m; 1) v n ' = (1; m) r l


n . n ' = m.1 + 1.( m) = 0 vy m m' . Do hai ng lun ct nhau bt bit gi tr m l th no

Hn na A(1; 1) m v B(3; 1) m' vi m do giao im ca hai ng lun chy trn


ng trn ng knh AB.
Li gii 2
To giao im l nghim ca h:
m(x 1) = 1 y
mx + y m 1 = 0

x my 3 m = 0 .
x 3 = m(y + 1) .
2
V D = 1 + m 0 nn h lun c nghim tc hai ng thng lun ct nhau.
-Xt x=1 th t ng thc 1 suy ra y=1.(1;1) thuc ng trn (x 2) 2 + y 2 = 2
-Xt y=-1 th t ng thc 2 suy ra x=3. (3;-1) thuc ng trn (x 2) 2 + y 2 = 2
-Xt x khc 1 v y khc -1 th
1 y x3
=
a n pt (x 2) 2 + y 2 = 2 .Vy 2 ng thng lun ct nhau ti 1 im trn
m=
x1 y+1
ng trn (x 2) 2 + y 2 = 2.
Li gii 3
mx + y = m + 1
.
Xt h phng trnh
x my = m + 3
Tnh 3 nh thc ta c
D = m 2 1 khc 0 vi mi m
Dx = m 2 2m 3
Dy = m 2 + 2m 1
Suy ra h c nghim duy nht (x; y) vi x =
(x 2) 2 + y 2 = .... = 2

(m 2 + 1)

m 2 + 2m + 3
m 2 2m + 1
=
,
y
m2 + 1
m2 + 1

=2
(m 2 + 1) 2
Vy giao im ca hai t lun nm trn ng trn c nh (x 2) 2 + y 2 = 2
Nhn xt:
Bi ny kh nht l kh m c pt ng trn,nhng ngc li bi cho bit ng c nh l
ng trn do ta c th tm pt ng trn nh sau:
2
2
m 2 + 2m + 3

m 2 2m + 1

B1)Ta vit (x a) 2 + (y b) 2 = =
+
1

a
+
+
1

m2
m2

B2)Khai trin t s v cho cc h s bc l bng 0 v cc h s bc chn t l vi cc h s ca


m 4 + 2m 2 + 1 t ta suy ra a = 2, b = 0
Bi ny s kh hn nu khng cho bit dng ca ng c inh
Bi tp:Cho hai t (d1 ) : mx y + m = 0 v (d2 ) : (1 m 2 )x + 2my (1 + m 2 ) = 0
Vi mi gi tr ca m xc nh giao im ca (d1 ) v(d2 )Tm qu tch giao im khi m thay i
2. Trn mt Oxy trong h (Ox yz) cho hnh vung OABC vi A(3; 4; 0). im S di ng trn Oz, k
OE SA v OF SC. Chng minh (OEF) SB v tnh VS.OEF theo OS = s.
Li gii:

4 of 8

2/23/2010 10:55 AM

p n thi s 2 ca math.vn - Din n Ton Hc

http://math.vn/showthread.php?t=1537

Gi thit ta c AB mp(SOA) OE mp(SAB) OE SB, tng t


BC mp(SOC) OF mp(SCB) OF SB. Kt hp li ta c SB mp(OEF).
Do A(3; 4; 0) OA = OC = 3 2 + 4 2 = 5.
2

VS.OEF SF.SE SF.SC.SE.SA


SO 2 .SO 2
s2
=
=
=
=
.
VS.OAC SC.SA
SC 2 .SA 2
(SO 2 + OC 2 ).(SO 2 + OA 2 ) s 2 + 25
2
1
25s
s 5s 2
Cng vi VS.OAC = VS.OABC =
.
ta c ngay VS.OEF = . 2
2
6
6 s + 25
Mt khc

Cu VIIa (1,0 im) Cho cc s nguyn dng x; y; z thay i tha mn x + y + z = 2010.


Tm gi tr nh nht v gi tr ln nht ca P = x !.y !.z !.
Li gii
B :
2

m + n
(m + n 1) ! m!.n !
! m; n + v m; n cng tnh chn l.
2
Chng minh: Khng mt tnh tng qut gi s m n v m + n = 2k; k + khi xt dy hu hn
un = n !.(2k n) ! vi n {k; ...; 2k 1} c un + 1 un = n !.(2k n 1) !.(2n + 1 2k) > 0 do dy
un l dy tng t u2k 1 > u2k 2 > .. > uk (2k 1) ! > n !.(2k n) ! > (k !) 2 .
T y c s khng nh cho tnh ng n ca B .
Quay li bi ton ..
V x + y + z = 2010 nn trong ba s x; y; z t c mt s chn khng gim tnh tng qut ta gi s z chn
c x + y = 2010 z cng chn!
Khi p dng B th:
5 of 8

2/23/2010 10:55 AM

p n thi s 2 ca math.vn - Din n Ton Hc

http://math.vn/showthread.php?t=1537

Khi p dng B th:


x + y 2
z + 670 2

>
0vz
!.670
!
x !.y !
!
! > 0.
2
2
x+
T x !.y !.z !.670 !
2

y z + 670 2
!. 2 !

Li p dng b pht na th
2
x + y + z + 670
2
x
+
y
z
+
670
2 ! = x + y + z + 670 ! = (670 !) 2 > 0.

!.
! 2
2 2


2
4


T : x !.y !.z !.670 ! (670 !) 4 tc x !.y !.z ! (670 !) 3 du bng xy ra khi x = y = z.
Vy gi tr nh nht cn tm l: (670 !) 3 .
Li vn theo B th c:
x !.y !.z ! (x + y 1) !.z ! (x + y 1 + z 1) 2008 ! du bng xy n chng hn khi
x = y = 1; z = 2008.
Vy gi tr ln nht l 2008 !.
Phn B
Cu VIb (2,0 im)
1
x2 y2
+
= 1 qua php i xng trc : y = .x. Vit phng trnh
2
4
9
ca elip (E ) l nh ca (E ') qua php i xng trc ' : y = 3x.
1. Cho elip (E ') l nh ca (E) :

Li gii 1.
Gi F '1 (x0 ; y0 ) v F '2 ( x0 ; y0 ) l hai tiu im ca elip (E ').
3

2(x0 5) + y0 = 0
x0 = 5

Do u = (2; 1) nn y

.
1 xo + 5
o
= .
y = 4
2
2 2
0 5
4
3 4
3
Ta c F '1 ; v F '2 ; .
5 5
5
5
Gi F1 (x0 ; y0 ) v F2 ( x0 ; y0 ) l hai tiu im ca elip (E ).
3
4

+ 3 y0 = 0
x0
x0 = 0

5
5

Do u' = (1; 3) nn

.
3
4

y0 = 5

y0 + 5 = 3. x0 + 5

Ta c F1 (0; 5) v F2 (0; 5).


Cui cng PT ca (E ) l

x2 y2
= 1.
+
4
9

Li gii 2

| 3 1 |
|
2 | = 1 vy = .
Gi (; ) = khi tan = |
1|
4
| 1 + 3. 2 |
Tch hai php i xng trc l 1 php quay vi gc quay gp 2 ln gc hp bi 2 trc do
(E ) = Q (E).
'

O;

6 of 8

2/23/2010 10:55 AM

p n thi s 2 ca math.vn - Din n Ton Hc

http://math.vn/showthread.php?t=1537

(E ) = Q

(E).
O; 2
2
2

Vy (E ) =

x
y
+
= 1.
4
9

2. Trn mt (Oxy) trong h (Ox yz) cho hnh vung OABC vi A(3; 4; 0). im S di ng trn Oz, k
OE SA v OF SC. Tm tp hp giao im P ca (OEF) v SB.
Li gii

Gi thit ta c AB mp(SOA) OE mp(SAB) OE SB, tng t


BC mp(SOC) OF mp(SCB) OF SB. Kt hp li ta c SB mp(OEF).
Do mp(SOB) ct mp(OEF) theo giao tuyn vung gc vi SB.
Nu gi P l giao im ca SB vi mp(OEF) th OP SB.
Hn na mp(SOB) c nh v OB = 52 nn khi S chuyn ng trn Oz th P chuyn ng trn ng
trn ng knh OB nm trong mt phng c nh (zOB).
Cu VIIb (1,0 im) Tm h s ln nht trong cc h s ca cc n thc bc 2010 sau khi khai trin
a thc P(x; y; z) = (1 + x) 2010 (1 + y) 2010 (1 + z) 2010 .
Li gii
B :
2

m + n
(m + n 1) ! m!.n !
! m; n + v m; n cng tnh chn l.
2
Chng minh: Khng mt tnh tng qut gi s m n v m + n = 2k; k + khi xt dy hu hn
un = n !.(2k n) ! vi n {k; ...; 2k 1} c un + 1 un = n !.(2k n 1) !.(2n + 1 2k) > 0 do dy
un l dy tng t u2k 1 > u2k 2 > .. > uk (2k 1) ! > n !.(2k n) ! > (k !) 2 .

7 of 8

2/23/2010 10:55 AM

p n thi s 2 ca math.vn - Din n Ton Hc

http://math.vn/showthread.php?t=1537

T y c s khng nh cho tnh ng n ca B .


Quay li bi ton:
Theo khai trin Newton th:
2010 k
2010 l
2010 m m
P(x; y) = C2010
x k . C2010
y l . C2010
z =

k=1
l=1
m = 1

k;l;m [0;2010]

k
l
m
C2010
.C2010
.C2010
.x k y l z m .

k
l
m
Mt n thc bc 2010 trong khai trin c dng C2010
.C2010
.C2010
.x k y l z m vi k + l + m = 2010.
k
l
m
Do h s ca n thc bc 2010 c dng C2010
.C2010
.C2010
vi k + l + m = 2010.

k
l
m
Li c: C2010
.C2010
.C2010
=

(2010 !) 3
.
k !.l !.m!.(2010 k) !.(2010 l) !.(2010 m) !

Mt khc do k + l + m = 2010 nn trong ba s k; l; m phi c mt s chn


khng mt g ca b ta gi s m chn lc k + l = 2010 m cng chn.
Theo B kia th:
2

k + l
m + 670
k !.l !
! > 0 v m!.670 !
! > 0.

2
2
2010 k + 2010 l
Mt cch tng t th (2010 k) !.(2010 l) !
!

2
2010 m + 1340 2
v (2010 m) !.1340 !
! .

T : k !.l !.m!.(2010 k) !.(2010 l) !.(2010 m) !.670 !.1340 !


k + l m + 670 2010 k + 2010 l 2010 m + 1340 2

2 !. 2 !.
!.
! .
2
2
2
k + l + m + 670
2
k + l m + 670

2
2
! = k + l + m + 670 ! = (670 !) 2 > 0.

Li tip tc c:
!.
!


2 2


2
4


V
2
2010 k + 2010 l !. 2010 m + 1340 ! 2010 k + 2010 l + m + 1340 ! = (1340 !) 2 > 0.

2
2
4

Do vy: k !.l !.m!.(2010 k) !.(2010 l) !.(2010 m) !.670 !.1340 ! (670 !.1340 !) 4 .


Tc l k !.l !.m!.(2010 k) !.(2010 l) !.(2010 m) ! (670 !.1340 !) 3
2010 ! 3
k
l
m
670 3

c: C2010
.C2010
.C2010
670 !.1340 ! = (C2010 ) .
3

670
) .
Du bng xy n khi k = l = m = 670 vy gi tr ln nht cn tm l: (C2010

8 of 8

2/23/2010 10:55 AM

THI TH S 01
Din n Ton Hc MATH.VN
(Thi gian: 180 pht)
I. PHN CHUNG CHO TT C TH SINH (7,0 im)
Cu I (2,0 im)
bi: Cho hm s y = x 4 2mx 2 + 2 (Cm ) vi m l tham s.
1. Kho st s bin thin v v th ca hm s cho vi m = 1.
2. Tm m th hm s c 3 im cc tr to thnh mt tam gic nhn gc ta l trc tm.
Li gii:
1. Kho st s bin thin v v th ca hm s cho vi m = 1.
Bn c t gii.
2. Tm m th hm s c 3 im cc tr to thnh mt tam gic nhn gc ta l trc tm.
Tp xc nh
Ta c y ' = 4x 3 4mx = 4x(x 2 m)
Hm s c ba im cc tr khi v ch khi y ' = 0 c 3 nghim phn bit (khi y ' i du qua 3
nghim ), hay phng trnh x 2 = m c 2 nghim phn bit khc 0.
Do m > 0.
Ta 3 im cc tr l A(0; 2), B(m; 2 m 2 ), C( m; 2 m 2)
V B, C i xng nhau qua trc tung nn BC lun vung gc vi OA

Do O l trc tm ca tam gic ABC khi v ch khi OB AC = 0


Hay m 4 2m 2 m = 0
1 5
Gii ra ta c m = 0, m = 1, m =
2
1 + 5
So snh iu kin ta c gi tr cn tm ca m l m =
.
2
Bnh lun:
Mun gii quyt cu hi ny cn phi nm chc kin thc ca phn cc tr v vc t. Mt trong
nhng sai lm thng gp l vic xc nh m th hm s c 3 im cc tr. Rt nhiu hc sinh
ni rng: hm s c 3 im cc tr khi v khi y c 3 nghim phn bit, iu khng chnh xc.
Nu bn cn thn, bn nn lp bng bin thin trong tng trng hp m = 0, m > 0, m < 0.
Cu II (2,0 im)
1. Gii phng trnh: 3(sin2x + sinx) + cos2x cosx = 2
Li gii 1

t x = + t ta c phng trnh tr thnh:


6

3. sin + 2t + sinx + t + cos + 2t cos + t = 2


3

6
3

6
1
3
.(sin2t + 3.cos2t + 3.sint + cost) + .(cos2t 3.sin2t 3.cost + sint) = 2
2
2
2cos2t + 2sint 2 = 0 2sint 4sin 2 t 2sint.(1 2sint) = 0
sint = 0
t = n.

n .
1

| |
sint =
t = ( 1) n + 2 . + n.

2
6

| |
Tc l phng trnh ra c tp nghim l: + n.; .
1 + ( 1) n + 2
.
+ n. : n
6

6
Li gii 2
1
1
3
3
PT
sin2x + cos2x +
sinx cosx = 1
2
2
2
2

1 of 7

2
2
2
2

cos 2x + sin x 1 = 0

3
6

2
2sin x + sin x = 0

6
6
Vy xy ra:

+) Nu sin x = 0 th x = + k

6
6
1

+) Nu sin x = th x = x = + k2 hoc x = + k2

6 2
3
Bnh lun:
* tng ca cch 1 c hn mt chuyn ca thy Nguyn Song Minh trn din n, cc bn
tham kho ti y y.
* Khi gii PTLG nu trong PT xut hin 3sinx cosx th ta lin tng n cch gii PT bc nht
i vi sinx v cosx.
1

3
* ch ny : sin2x + cos2x c hai hng bin i hoc a v cos 2x hoc a v

2
2
3

sin 2x . Tuy nhin a v cos 2x cho chng ta li gii n gin hn.

6
3
* Bi PT trnh c ch rt p t PT bc nht i vi sinx v cosx.
* Bi tng t:
(1 2sinx)cosx
1)
= 3 (Khi A - 2009
(1 + 2sinx)(1 sinx)
2) sinx + cosxsin2x + 3 cos3x = 2(cos4x + sin 3 x) (Khi B - 2009)
3) 3cos5x 2sin3xcos2x sinx = 0 (Khi D - 2009).
2. Tm tham s m phng trnh log2 (mx 6x 3 ) + 2log1 ( 14x 2 + 29x 2) = 0 c 3 nghim phn
2

bit.
Li gii
Ta c
log2 (mx 6x 3 ) + 2log1 ( 14x 2 + 29x 2) = 0
2

log2 (mx 6x 3) = log2 ( 14x 2 + 29x 2)


14x 2 + 29x 2 > 0

3
2
mx 6x = 14x + 29x 2
1
<x<2
14

3
2
mx = 6x 14x + 29x 2
1
14 < x < 2

3
2
m = 6x 14x + 29x 2 (1)

x
3
6x 14x 2 + 29x 2
1
Xt (C) : f (x) =
vi x ; 2
14
x
12x 3 14x 2 + 2
C f '(x) =
x2
f '(x) = 0
1

x = x = 1
1
2
<
x
<
2

14
(1) chnh l phng trnh honh giao im ca ng thng (d) : y = m v th (C) vi
1
x ; 2.
14

2 of 7

14
1
S nghim ca phng trnh cho cng chnh l s giao im ca (d) v (C) trn ; 2 nn
14
Phng trnh cho c 3 nghim phn bit khi v ch khi (d) ct (C) ti 3 im phn bit trn
1 ; 2
14
39
Da vo bng bin thin, yu cu bi ton 19 < m < .
2
Bnh lun:
y l bi ton thuc dng s dng phng php hm s, cc bn nn ch phng php ny,
c nhiu thi i hc cng dng.
V d:
Khi B nm 2006. Tm m phng trnh sau c hai nghim phn bit
x 2 + mx + 2 = 2x + 1
Khi A nm 2007. Tm m phng trnh sau c nghim thc
4 2
3 x 1 + m x + 1 = 2 x 1
Cu III (1,0 im) Tm gii hn lim

x0

ln(1 + tan2x sin2x)


.
2
x(e x 1)

Li gii
ln(1 + tan2x sin2x)
tan2x sin2x
2
x3
ex 1
tan2x sin2x . x 2
1 cos2x
tan2x sin2x
sin2x cos2x
4sin 2 x
= lim
=
=
=4
lim
lim
.
1 x2
x0
x 0 2x
x 0 x2
x3
2

ln(1 + tan2x sin2x)


= lim
lim
2
x0
x0
x(e x 1)

Bnh lun:
Khi gp cc gii hn c hm s lng gic, m, logarit th cc bn cn nh n cc gii hn c bn
ca cc hm s .
Cu IV (1,0 im) Cho hnh chp u S.ABCD c y ABCD l hnh vung tm O, cnh a. Gi
M , N ln lt l trung im ca SA v BC. Bit ng thng M N to vi mt y gc 30 o . Tnh
th tch khi chp S.ABCD.
Li gii

3 of 7

Trc ht gi O l tm ca hnh vung v I l trung im AO.


^
Khi M I ||SO M I m p(ABCD) I N M = 30 o .
5a 2
a30
Trong CI N c IN 2 = IC 2 + NC 2 2.IC.NC.cos45 o =
SO = 2.M I = 2.I N.tan30 o =
.
6
8
3
a 30
Do VS.ABCD =
.
18
Bnh lun:
Bi ton ny kh c bn, cc bi ton v th tch vn l mn n thng xuyn trong cc thi i
hc.
Cu V (1,0 im) Cho a; b l cc s thc dng. Chng minh rng
1
32(a 2 + b 2 )
1
4

+ 2+ 2
2
2
a
(a + b) 4
b
a +b
Li gii 1
2
2
(a 2 + b 2 + 2ab)
(a 2 + b 2 )
4a 2 b 2
(a + b) 4

+
'
2 2
2 =
2 2
2
2 2
2
2
2
2
2
2a b (a + b )
a b (a + b ) a b (a + b )
2a b 2 (a 2 + b 2 )
(a + b) 4
32(a 2 + b 2 )

Ta cn chng minh: 2 2 2
2a b (a + b 2 )
(a + b) 4
4
2
2
(a + b)
8ab(a + b )
2

((a 2 + b 2) + 2ab) 8ab(a 2 + b 2 ).


Li gii 2
a b
t x = + , x 2.
b a
1
4 32(a 2 + b 2 )
1

BT (a + b) 2 2 + 2 + 2
a
b
a + b 2
(a + b) 2
8
32x
x 2 + 2x + + 4
x
x+2
(x 2) 2 (x 2 + 8x + 4) 0.
Bnh lun:
Bi BT ny khng kh, nhng cng i hi hc sinh phi c t duy BT nht nh mi gii quyt
c. C nhiu cch gii khc cho bi ton ny, cc bn c th tm ti coi nh bi tp.
II. PHN RING (3,0 im) Th sinh ch c lm mt trong hai phn: A hoc B.
Phn A
Cu VIa (2,0 im)
1. Lp phng trnh cc cnh ca tam gic u ABC bit A(3; 5) v trng tm G(1; 1).
Li gii:

Gi M l trung im BC suy ra AG = 2 GM M (0; 4) v PT (BC) : x 3y + 12 = 0.


y 0 = 4 + 3
Gi s B(3 y0 12; y0 ) v t GA = GB dn ti PT (3y0 13) 2 + (y0 1) 2 = 40
.
y = 4 3 .
0
Nu y0 = 4 + 3 B( 33; 4 + 3) C(33; 4 3).
Nu y0 = 4 3 B(33; 4 3) C( 33; 4 + 3).
T c PT cc cnh ca tam gic ABC.
Bnh lun:
Khi gii cc bi ton v cc yu t trong tam gic, cc bn nn xem li cc kin thc v cc im
ng c bit trong tam gic.
2. Cho ba im A(5; 3; 1), B(2; 3; 4), C(1; 2; 0). Tm ta im S trong khng gian sao cho

4 of 7

2. Cho ba im A(5; 3; 1), B(2; 3; 4), C(1; 2; 0). Tm ta im S trong khng gian sao cho
hnh chp S.ABC c gc tam din nh S l tam din vung.
Li gii
Hnh chp S.ABC c gc tam din nh S l tam din vung suy ra S nm trn ng thng vung
gc vi m p(ABC) v qua trc tm ca tam gic ABC.
Nhn xt rng ABC u nn S nm trn trc ca ng trn tam gic ABC.

8 8
5
Khi tm ca ABC l G ; ; v
u = AB , AC = ( 3; 15; 3) nn PT trc ca ng

3 3
3
8

x = 3 t

8
trn tam gic ABC l y = + 5t
3

5
z = + t .
3

1
t = 3
2
5
2
1
5
7
Thm iu kin SC .SB = 0 t t + + 5t 5t + t t + = 0

3 3 3

3 3 3
1
.
t =
3

4
7 13
v ta c hai im S ; ; v S(3; 1; 2) tha mn.
3 3
3

Bnh lun:
C th gii bi ton thun ty vec t, tc l iu kin ca bi ton c tha mn nu xy ra ng

thi S A S B = 0, S B S C = 0, S C S A = 0. Cch lm ny i n mt h 3 phng trnh 3 n.


5 n
3
Cu VIIa (1,0 im) Tm s hng dng hu t khi khai trin nh thc 2 x 4 4 3 bit rng

x
An3 + 22Cn1+ 1 = 2(19Cnn++31 + 4) .
Li gii
An3 + 22Cn1 1 = 2.(19Cnn++31 + 4).
(n + 3) !
n!

+ 22(n 1) = 38
+8
(n 3) !
(n + 1) !.2 !
(n 2)(n 1)n + 22(n + 1) = 19(n + 2)(n + 3) + 8
n 3 22n 2 71n 100 = 0 n = 25.
Khi y
25(k 9)
25
3 25 k
5 25 25 k 4 k
3 4
k
.2 k .( 5) 25 k .x 12 .
2
x

= C25 . 2.x 3 .( 5) 25 k . x 4
= C25
4 3

x
k=0
k=0

Ta c:
25.(k 9)

k = 9

12

k = 21
0 k 25

Vy c 2 s hng hu t l:
21 21
9
.2 .( 5) 4 .x 25 .
.2 9 .( 5) 16 v C25
C25
Bnh lun:
y l dng ton tng gp nhiu ln trong thi i hc, cc bn cng khng nn ch quan.
Mt s v d:
n
1
Khi A nm 2003. Tm s hng cha x 8 trong khai trin nh thc Newton ca 3 + x 5 bit rng
x

Cn + 4 n + 1 Cn + 3 n = 7(n + 3).
8

Khi A nm 2004. Tm h s ca x 8 trong khai trin thnh a thc ca (1 + x 2 (1 x)) .

5 of 7

Khi A nm 2004. Tm h s ca x 8 trong khai trin thnh a thc ca (1 + x 2 (1 x)) .


Phn B
Cu VIb (2,0 im)
1. Vit phng trnh cnh AB (ng thng AB c h s gc dng), AD ca hnh vung ABCD
bit A(2; 1) v ng cho BD : x + 2 y 5 = 0.
Li gii
Do AC BD nn PT (AC) : 2x y 5 = 0 v tnh c tm ca hnh vung
I(3; 1) C(4; 3) AC 2 = 20.
y0 = 0
Gi B(5 2 y0 ; y0 ) v ng thc 2AB 2 = AC 2 dn ti PT (3 2 y0 ) 2 + (y0 + 1) 2 = 10
y0 = 2 .
Suy ra vi y0 = 0 B(5; 0) D(1; 2) cn y0 = 2 B(1; 2) D(5; 0).
V ng thng AB c h s gc dng nn PT (AB) : x 3y 5 = 0 v (AD) : 3x + y 5 = 0.
2. Cho ba im A(5; 3; 1), B(2; 3; 4), C(1; 2; 0). Chng minh rng tam gic ABC l tam gic
u v tm ta im D sao cho t din ABCD l t din u.
Li gii
th nht ch cn kim tra AB = BC = CA = 32 l xong.
xc nh ta D c rt nhiu cch tip cn, khai thc theo cc tnh cht ca t din u.

5
8 8

Chng hn ta tnh tm ca tam gic ABC l G ; ; v u = AB , AC = ( 3; 15; 3) nn PT

3 3
3
8

x = 3 t

8
trc ca ng trn tam gic ABC l y = + 5t
3

5
z = + t .
3

2
t = 3
5
5
2
Cui cng gii PT t + + 5t + + t = 18
v ta c hai im

3 3
3
2
.
t =
3

34
7
10 14
D 2; ; 1 v D ; ; tha mn.
9

3 9
3
2

Bnh lun:
Cng c th gii h DA = DB = DC = AB.
Cu VIIb (1,0 im) Chng minh rng vi mi gi tr khc khng ca tham s m , tim cn xin ca
mx 2 2(m 1)x m 3 + 4m 2 5m + 4
(Cm ) lun tip xc vi mt parabol
th hm s y =
xm+1
2
(P) : y = ax + bx + c c nh.
Li gii 1
Tim cn xin: y = mx + m 2 3m + 2
Tim cn xin tip xc vi (P) phng trnh :
ax 2 + bx + c = mx + m 2 3m + 2 ax 2 + (b m)x + c m 2 + 3m 2 = 0 c nghim kp vi m
(b m) 2 4a(c m 2 + 3m 2) = 0 (1 + 4a)m 2 (2b + 12a)m + b 2 4ac + 4a = 0 m
1 + 4a = 0

2b + 12a = 0
2
b 4ac + 4a = 0
1
3
1
T ta tm c: a = ; b = ; c = .
4
2
4
1 2 3
1
Vy (P) : y = x + x .
4
2
4

6 of 7

Li gii 2
m 2 3m + 4
nn c
xm+1
m 2 3m + 4
lim (y mx m 2 + 3m 2) = lim
= 0 ng thng y = mx + m 2 3m + 2(m
x
x x m + 1

Ta c : y = mx + m 2 3m + 2 +

khc 0) l T C X ca th hm s .
x 3 2 x 2 + 6x 1
+
Ta vit li T C X : y = mx + m 2 3m + 2 = m +

2
4
x 2 + 6x 1
Xt Parabol(P):y =
. Honh giao im ca T C X v (P) l nghim PT :
4
2
2
2
2
m + x 3 + x + 6x 1 = x + 6x 1 m + x 3 = 0 , PT ny lun c nghim kp

2
4
2
4
x = 3 2m. Chng t khi m khc 0T C X ca th hm s cho lun tip xc vi Parabol(P)
(pcm).
..................Ht.................

7 of 7

TNG

HP P N THI TH S

15 CA

Cu I.1 (1 im)
x+1
Cho hm s y =
Kho st s bin thin v v th ca hm s.
2x 1
Gii .
x+1
c tp xc nh D = R\{ 21 }.
Hm s y =
2x 1
3
o hm y0 =
< 0 x D
(2x 1)2
1
lim y = lim y = + nn x = l pt tim
2
x( 21 )+
x( 21 )
cn ng
lim y = lim y = 12 nn y = 21 l pt tim cn
x+
x
ngang
Bng bin thin ca hm s
x
f 0 (x)
f (x)

1
2

1
2

&

5
4
3
2
b

1
b

1
1

&

MATH . VN

1
2

Hm s nghch bin trn (; 12 ); ( 12 ; +);


4
th ct Ox ti (1; 0) v ct Oy ti (0; 1)
Cu I.2 (1 im)
Tm gi tr nh nht ca m sao cho tn ti t nht mt im M (C) m tip tuyn ti M ca (C) to vi hai trc
ta mt tam gic c trng tm nm trn ng thng y = 2m 1.
Gii .Mickey
3
(x xo ) + yo
D dng vit c pt tip tuyn ti M(xo ; yo ) l y =
(2xo 1)2
Gi A, B l giao im ca tip tuyn vi trc honh, trc tung tng ng
2x2 + 4xo 1
2xo2 + 4xo 1
Nn yB = o
v
trng
tm
G
ca
tam
gic
OAB
c
y
=
G
(2xo 1)2
3(2xo 1)2
2
2x + 4xo 1
= 2m 1
Theo gi thit n nm trn ng thng y = 2m 1 nn o
3(2xo 1)2
2x2 + 4xo 1 6xo2 (2xo 1)2
6xo2
Ta li c o
=
=
1 1
(2xo 1)2
(2xo 1)2
(2xo 1)2
1
1
Vy tn ti t nht mt im M tha mn iu kin bi ton th 2m 1 m .
3
3
Cu II.1 (1 im)

Tnh tng tt c cc nghim thuc (0; 2 ) ca phng trnh ( 3 1) sin x + ( 3 + 1) cos x) = 2 2 sin 2x.
Gii .Mercury

1
1
3
3
sin x sin x +
cos x + cos x = 2 sin 2x
2
2
2
2

1
1
3
3
sin x +
cos x sin x = 2 sin 2x
cos x +
2 
2
 2  2

sin x +
+ cos x +
= 2 sin 2x
6
6


5
2 sin x +
= 2 sin 2x
12
5
sin(x + ) = sin 2x
12

5
5
5
2x = x + 12 + 2k
x = 12 + 2k
x = 12

7
7
7 2k cc nghim thuc (0; 2 ) l
x=
x + 2k
2x =
x=
+
36
12
36
3
5 7
11
=
do tng cc nghim thuc (0; 2 ) l +
12 36
18
Cu II.2 (1 im)

x + 2y + 24x + y = 1
Gii h phng trnh sau trn R: p
46 16y(x + y) 6y + 44x + y = 8 4y
Gii .tuong_vb

T phng trnh u ta c 2 4x + y = 1 x 2y. Th vo phng trnh (2) ta c


p
p
46 16y(x + y) 6y + 2 2x 4y = 8 4y 46 16y(x + y) 6y = 6 + 2x
46 16xy 16y2 6y = 36 + 24x + 4x2 4x2 + 16xy + 16y2 + 24x + 6y 10 = 0
(2x + 4y)2 + 6(4x + y) 10 = 0 (a)

Kt hp (a) vi x + 2y + 2 4x + y = 1 cho ta h sau

4u2 + 6v2 10 = 0 v u + 2v = 1 (vi x + 2y = u, 4x + y = v, v > 0)


Gii ra v = 1 u = 1. Suy ra ra x, y
3
loi.
v=
11
Cu III. (1 im)
Z e
ln x(ln x + 1)
Tnh tch phn I =
dx
3
1 (ln x + x + 1)
Gii .Cch 1: fanarsenal_vietnam
Z e
Z e
Z e
ln x
x ln x
ln x(ln x + 1)
dx =
dx
dx
Ta c I =
3
2
3
1 (ln x + x + 1)
1 (ln x + x + 1)
1 (ln x + x + 1)
Z e
Z e
Z e
Z e
dx
dx
x ln xdx
xdx
I=

2
2
3
1 ln x + x + 1
1 (ln x + x + 1)
1 (ln x + x + 1)
1 (ln x + x + 1)
Z e
dx
Xt tp I1 =
dng tch phn tng phn ta c
1 ln x + x + 1
1
e Z e 
0
Z e
1+

1
e
1
x
x

dx
+
x
dx =
x
I1 =
2
ln x + x + 1 1
ln x + x + 1
e+2 2
1 (ln x + x + 1)
1
Z e
Z e
e
dx
1
xdx
I1 =
+
+
2
2
e+2 2
1 (ln x + x + 1)
1
Z e(ln x + x + 1)
xdx
xt tch phn I2 =
dng tch phn tng phn ta cng c
2
1 (x + ln x + 1)

e
Z e x2

I2 =
+
2
1 2
2(ln x + x + 1)
x2

e2

1
Z
e

1
(ln x + x + 1)

!0

dx =

x2 + x

e2

1
+
8
2(e + 2)
2

1
x
dx
2
2 (ln x + x + 1)3

Z e 2
x
1

1+

1
+
dx
3
8
1 (lnx + x + 1)
2(e + 1)
Z e
Z e
1
e
x(ln x + x + 1)
1
e2
xdx

dx
Do I =

+
2
3
e + 2 2 2(e + 2)2 8
1 (ln x + x + 1)
1 (ln x + x + 1)
e2 + 4e
3 1
3 1
2
2
I=
=
=
2
2
8 2 (e + 2)
8 8 (e + 2)2
2(e + 2)
Cch 2: up_pixar
x+1
dx
vi nhn xt d(ln x + x + 1) =
x
Z e
Z e
Ze
ln x(ln x + 1)
ln x + 1
(x + 1)(ln x + 1)
I=
dx =
dx
dx
3
2
3
(ln x + x + 1)
1 (ln x + x + 1)
1 (ln x + x + 1)
I2 =

x+1
[x(ln x + 1)]
x
Khi t I1 =
dx
th
I
=
dx
1
1)3
(ln x + x + 1)3
1 (ln
1
ex +Zx +
Z e
e
x(ln x + 1)
e
1
(lnx + 2)
(lnx + 2)
I1 =
dx =
+ +
dx
+

2
2
2
2
2(ln x + x + 1) 1
(e + 2)
8
1 2(ln x + x + 1)
1 2(ln x + x + 1)
Z e
e
1
lnx
dx +

T suy ra I =
2
2
(e + 2)
8
1 2(ln x + x + 1)
Z e
I2
e
1
lnx
dx
th I = +
.
t I2 =
2
2
2 (e + 2)
8
1 (ln x + x + 1)
Tng t ta tnh c
x+1
Z e
Z e
Z e
Z e
x
1
x+1
1
x
I2 =
dx
dx
=
dx

dx
2
2
1
(ln
x
+
x
+
1)
ln
x
+
x
+
1
(ln
x
+
x
+
1)
1 ln x + x +
1
1
1
e

x
= e 1
I2 =
ln x + x + 1 1 e + 2 2
1
e
1
e2 + 4e
3 1
2
2
e
3 1
+

=
=
Vy I =
=
2
2
2
2(e + 2) 4 (e + 2)
8 2(e + 2)
8 2 (e + 2)
8 8 (e + 2)2
Cch 3: ktmktm
ln x
Z e
Z e
ln x (ln x + 1)
(ln x + 1)2
I=
dx
=

3 dx
3
1 (ln x + x + 1)
1
x
+1
ln x + 1
x
e
ln (x)
t=
dx x = 1 t = 1 x = e t =
dt =
2
ln (x) + 1
2
(ln (x) + 1)
e

Z e
2
2
(t + 1)2
1
2
dt
e2 + 4e 12 1
1
=
=

=
I=
=



3
2
e
2
8
8 (e + 2)2
1 (t + 1)
8(e + 2)2
2 +1
1
2
Cu IV. (1 im)
Cho hnh chp S.ABCD c y l hnh ch nht AB = a, AD = b, SA vung gc vi y v SA = 2a. Tm im M
trn cnh SA sao cho mt phng (MBC) chia khi chp S.ABCD thnh hai khi c th tch bng nhau.
Gii .Cch 1: Mercury
Z e
(x + 1)(ln x + 1)

Z e

trn SC ly N sao cho MN//AC ta c N (MBC)


1
1
gt SSBC = SACD = SABCD VS.ABC = VS.ACD = VS.ABCD
2
2

VS.MBC SM
SM.VS.ABCD VS.MNC
SM 2 .VS.ABCD
SM 2
VS.MBC =
VS.MBC =
=
=
VS.ABC
SA
2SA
VS.ACD
SA 
2SA2

VS.ABCD SM SM 2
+ 2
Suy ra:VS.MNBC = VS.MBC +VS.MNC =
2
SA
SA

1
(MBC) chia khi chp S.ABCD thnh hai khi c th tch bng nhau th VS.MNBC = VS.ABCD
2



SM SM 2
51
SM
SM
do :
51
+ 2 = 1 m
> 0 nn
=
SM = a
SA
SA
SA
SA
2
Cch 2: supervirus
S
t AM = x (0 < x < 2a)
D thy CB(SAB); CD(SAD),
do VMN.ABCD = VC.ABM +VC.AMND
abx ax(MN + AD)
N
M
+
VMN.ABCD =
6
6
abx bx(4a x)
+
(1)
VMN.ABCD =
6
12
A
D
2a2 b
VS.ABCD =
= 2VMN.ABCD (2)
3

x = (3 5)a
B
C
Cu V. (1 im)
1
Tm m h phng trnh sau c nghim (x; y) tha mn x
2
x

>1
y
2x3 + 1

= 2m + 1.
y(x y)
b

Gii .up_pixar
x
ta c: > 1 nn y(x y) > 0 t a = y(x y) (a > 0)
y
Phng trnh bc 2 n y: a = y(x y) tng ng vi y2 xy + a = 0
x2
Phng trnh c nghim = x2 4a 0 hay a
4
x2
2x3 + 1
= 2m + 1 H c nghim nn a
Phng trnh ca h :
a
4
1
2x3 + 1 2m + 1

H cho c nghim x
.
Suy ra
x2
4
2
2x3 + 1
2m + 1
2
1
Do
min f (x) vi f (x) =
Ta c: f 0 (x) = 2 3 = 0 x = 1 >
2
4
x
x
2
x[ 1
2 ;+]

f (x) ng bin trn mi khong ( 12 ; 0) v (1; +) nghch bin trn (0; 1)


11
2m + 1
3 hay m
M f ( 12 ) = f (1) = 3 nn min f (x) = 3 Do vy
4
2
x[ 1
2 ;+]
2
3
11
1
x
2x + 1
Mt khc vi mi m
th tn ti xo v a o sao cho o
= 2m + 1
2
2
4
a
Khi h cho c nghim tha mn iu kin bi ra.
11
1
Vy iu kin cn v h c nghim x l m
2
2
Cu VIa.1 (1 im)
4

x 2 y2
+ = 1, ng thng dm : x my + 1 = 0 v im C(1; 0).
9
4
Chng minh rng (E) lun ct dm ti hai im phn bit A, B. Tm m tam gic ABC c din tch ln nht.
Gii .minhkhac + hoainamss
2
2
x + y = 1
4
(E) ct (dm) ti 2 im phn bit h pt 9
c 2 nghim phn bit

x my + 1 = 0

4(m2 y2 2my + 1) + 9y2 = 36


h pt
c 2 nghim phn bit (4m2 + 9)y2 8my 32 = 0
()
x = my 1

Trong mt phng vi h ta Oxy, cho Elip (E) :

c 2 nghim phn bit v 4m2 + 9 > 0 v 32 < 0


tung y1 , y2 ca A, B l nghim ca phng trnh (4m2 + 9)y2 8my 32 = 0.
nn A(my1 1; y1 ), B(my2 1; y2 ).
p
1
2
1
.
(my1 1 my2 + 1)2 + (y1 y2 )2 = |y1 y2 |.
V lp cng thc: S = AB.d[C; dm ] =
2
2 sqrt1 + m2
8m
32
V S2 = (y1 + y2 )2 4y1 y2 . n y lp y1 + y2 =
y1 y2 =
vo thu c 1 phn thc bc
2
4m + 9
4m2 + 9
2 trn bc 2 i vi t = m2 . n y th thiu g cch:
Cu VIa.2 (1 im)
x1 y+1
z
Trong khng gian vi h ta Oxyz cho ng thng () :
=
=
v im A(1; 2; 3). Lp phng
2
2
1
14 17
.
trnh mt phng (P) cha () v cch A mt khong bng
17
Gii .trnh tnh
z
x1 y+1
=
=
i qua im (1; 1; 0) v c vc t ch phng ~u(1; 2; 2)
ng thng () :
1
2
2
Gi (P) l mp cn lp.V (P) qua M nn (P) : a(x 1) + b(y + 1) + cz = 0(a2 + b2 + c2 > 0)
V (P) cha nn a + 2b 2c = 0 a = 2c 2b khi (P) : 2(c b)(x 1) + b(y + 1) + cz = 0
14 17
T gt suy ra d(A; (P) =
t y tm c b, c a
17
Cu VIIa. (1 im)

a + b + c + d
4
Cho cc s thc a, b, c, d tha mn
. Tm min (max{a, b, c, d}).
a2 + b2 + c2 + d 2 16.
Gii .Can_hang2007
Khng mt tnh tng qut, gi s a = max{a, b, c d}.
Ta chng minh a 2 vi ng thc xy ra khi a = b = c = d = 2.
rng, vi x max{y, z},
ta c y2 + z2 x2 (y + z x)2 = 2(x y)(x z) 0,
suy ra y2 + z2 x2 + (y + z x)2 .
S dng kt qu ny vi ch rng a max{b, c} v a max{d, b + c a},
ta c b2 + c2 a2 + (b + c a)2 , v d 2 + (b + c a)2 a2 + (b + c + d 2a)2 .
Cng tng ng v vi v hai bt ng thc ny cho ta b2 + c2 + d 2 2a2 + (b + c + d 2a)2 .
Nh vy, ta c 3a + x 4 v 3a2 + x2 16 vi a x, x = b + c + d 2a.
Do a x 4 3a nn ta c x2 max{a2 , (4 3a)2 }.
+ Nu a2 x2 th ta c 4a2 3a2 + x2 16 nn a 2 (v t gi thit, hin nhin a > 0).
+ Nu (4 3a)2 x2 th ta c 3a2 + (4 3a)2 3a2 + x2 16. Gii bt phng trnh ny vi ch a > 0,
ta tm c a 2.
5

Cu VIb.1 (1 im)
Trong mt phng vi h ta Oxy cho hnh ch nht ABCD tm I(1; 2), gi M l trung im cnh BC.
Tm ta cc nh ca hnh ch nht ABCD bit rng IOM c din tch bng 4, ng thng AB i qua im
N(11; 3) v cnh AD tip xc vi ng trn (C) : (x + 1)2 + (y + 2)2 = 2.
Gii .fanarsenal_vietnam

Ta thy (C) c tm I(1; 2) bn knh R = 2

ng trn (C) tip xc vi AD nn (C) l ng trn tm I i qua M IM = 2

1
1
din tch IOM = IM.d(O; IM) =
2.d[O; IM] = 4 d(O; IM) = 4 2
2
2

do OI = 5 < 4 2 = d[O; IM] nn khng tn ti im M tha mn bi ton


Bi ton khng c nghim hnh
Cu VIb.2 (1 im)
x y1 z+2
=
Trong khng gian vi h ta Oxyz cho ng thng () : =
2
1
1
v mt cu (S) : x2 + y2 + z2 2x + 4y 4 = 0. Gi s A, B l hai im di ng trn () sao cho AB = 2.
Tm im C trn mt cu (S) sao cho tam gic ABC c din tch nh nht.
Gii .

Cu VIIb. (1 im)
Gii phng trnh 22x+2 + 34 log22 x + 34 = 15.2x + 4(2x+2 + 1) log2 x + 2x.
Gii .

TNG HP P N THI TH S 16 CA MATH.VN


Cu I.1 (1 im)
x2
Cho hm s y =
Kho st s bin thin v v th (C) ca hm s.
x+1
Gii .
x2
Hm s y =
c tp xc nh D = R\{1}.
x+1
3
> 0 x D
o hm y0 =
(x + 1)2
lim y = + lim y = nn x = 1 l pt

5
4
b

x(1)+

x(1)

tim cn ng
lim y = lim y = 1 nn y = 1 l pt tim cn ngang

3
2

x+

Bng bin thin ca hm s


1
b

x
f 0 (x)

f (x) 1 %

1
2

Hm s ng bin trn (; 1); (1; +);


th ct Ox ti (2; 0) v ct Oy ti (0; 2)
Cu I.2 (1 im)
Tm tt c cc gi tr ca tham s m phng trnh

||x| 2|
= m c ng hai nghim phn bit.
|x| + 1

Gii .
T th y = f (x) ta s v th (C) ca y = | f (|x|)|
V th (C? ) ca y = f (|x|) Hm ny chn vi x nn nhn Oy
lm trc i xng
+ Gi nguyn phn th ng vi x 0
+ Ly i xng phn th qua Oy
T (C? )
+ Gi nguyn phn th pha trn Ox
+ Ly i xng phn pha di Ox ln Ox
3
2
Hp ca 2 phn trn chnh l (C)
T th ta tm c 1 m < 2
Cu II.1 (1 im)





cos 4x + sin 2x 2
= 2 2 sin x +
Gii phng trnh sau trn R:
+ 3.
cos 3x + sin 3x
4
Gii .
K:cos 3x + sin 3x 6= 0 (?)
cos 4x + sin 2x = cos 4x + cos(2x 2 ) = 2 cos(3x 4 ) cos(x + 4 )

cos 3x + sin 3x = cos 3x + cos(3x 2 ) = 2 cos(3x 4 )


V T PT = 2 cos2 (x + 4 ) = 2 2 sin2 (x + 4 )
PT sin(x + 4 ) = 12 x = 2 + k2 x = + k2 (T m (?))
Vy PT c 2 nghim
1

4
3
2

B
b

1
1

Cu II.2 (1 im)

x2 + 2y2
Gii h phng trnh sau trn R:
2x3 + 3xy2

= xy + 2y
= 2y2 + 3x2 y.

Gii .
x2 + 2y2 = xy + 2y
(1)
2x3 + 3xy2 = 2y2 + 3x2 y (2)
Nhn phng trnh (1) vi y ly (2) (1)
ta c (x y)[(x y)2 + x2 + y2 ] = 0 y = x hay x = y = 0
Thay x = y vo 1 ta c 2x(x 1) = 0 x = 0 hay x = 1
KL.Vy h cho c 2 nghim (x; y) = (0; 0), (1; 1)
Cu III. (1 im)
Z 1
x 3 ex
dx
Tnh tch phn : I =
2
0 (x + 3)
Gii .
1 Z 1


Z 1
Z 1
x3 ex
e
1
1
3 x
3 x
=
+
d(x e ) = +
x2 ex dx
I=
x ed
x+3
x + 3 0
4
0 x+3
0
0
Z 1
Z 1
1 Z 1
3e
e
e

ex d(x2 ) =
= +
2
x2 d(ex ) = + x2 ex
xex dx
4
4
4
0
0
0
0
Z 1
1

3e
3e
3e
=
2xex + 2
2e + 2(e 1) =
2
ex dx =
4
4
4
0
0
Cu IV. (1 im)
d = 60 . Mt bn (BCC1 B1 )
Cho hnh lng tr ABC.A1 B1C1 c y l tam gic vung vi cnh huyn BC = 2a, ABC

l hnh thoi (B[


1 BC < 90 ) v vung gc vi y, mt bn (ABB1 A1 ) to vi y mt gc 45 . Tnh th tch ca
khi lng tr ABC.A1 B1C1 .
Gii .
B
K BH vung gc vi mt phng (A1 B1C1 ).
b

V (BCC1 B1 ) vung gc vi y nn H thuc B1C1 .


K HK vung gc vi A1 B1 .
[ = 45o .
Gc gia 2 mt phng y v (ABB1 A1 ) l BKH
t BH = x th KH = BH = x.

Tam gic BHB1 vung gc ti H c HB1 = 4a2 x2


Xt 2 tam gic ng
dng A1 B1C1 v KB1 H
4a2 x2
HB1
HK
x
c
=
=
B1C1 A1C1
2a
a 3
2
2
2
2 12a4 = 7a2 x2
(4a x )3a = 4a2 x
2a 3
12a2
x=
x2 =
7
7

B1

K
b

A1

H
b

C1

Cu V. (1 im)
Cho a, b, c l ba s dng. Chng minh rng:

ab
bc
ca
a+b+c
+
+

a + 9b + 6c b + 9c + 6a c + 9a + 6b
16

Gii .

p
 dng BT Bunhiacopski:

1
1
1
1
3
16
1
+
+
+
+

(a + 9b + 6c) 16 Suy ra
a + 3c 3b + c 
3b + c 3b + c 
9b + 6c

 a + 3c
 3b + c a + 
3
3
3
ab
bc
ca
a+b+c
1
1
1
+
+
+
T suy ra P
+
+
=
16 a + 3c 3b + c
16 b + 3a 3c + a
16 c + 3b 3a + b
16
Du bng xy ra khi v ch khi (a = b = c); (a = 3b; c = 0); (b = 3c; a = 0); (c = 3a; b = 0)
Cu VIa.1 (1 im)
Trong mt phng vi h ta Oxy cho tam gic ABC c phng trnh cnh AB : 5x + 2y + 7 = 0,
BC : x 2y 1 = 0. Bit phng trnh ng phn gic trong gc A l x + y 1 = 0.
Tm phng trnh ng cao CH ca tam gic ABC.
Gii .
Ta tm c giao imA, B l A(3; 4) v B(1; 1).
Ly B1 i xng vi B qua ng phn gic trong ca A ta c B1 (2; 2)
Phng trnh cnh AC i qua A v B1 l 2x + 5y = 14
Ta tm c C l (11/3; 4/3) Phng trnh ng cao CH qua C c vecto php tuyn l (2; 5)
2x + 5y + 2/3 = 0
Cu VIa.2 (1 im)
Trong khng gian vi h ta Oxyz cho mt phng (P) : x + 2y + 2z + 5 = 0 v mt cu (S) : x2 + y2 + z2 10x
2y 6z + 10 = 0. T mt im M (P) k ng thng () tip xc vi (S) ti N. Tm v tr ca im M MN
t gi tr nh nht.
Gii .
Gi I(5; 1; 3) l tm mt cu th MN 2 = IM 2 R2
Vy MN nh nht th IM nh nht. Khi Ml hnh chiu ca I ln mt phng (P)

(x = 5 + t
ng thng (d) qua I v vung gc vi (P) l y = 1 + 2t

z = 3 + 2t

giao ca (d) v (P) l im M cn tm To M(3; 3; 1)


Cu VIIa. (1 im)
C 5 ngi n ng, 8 ngi n b v mt a tr c xp ngu nhin thnh mt vng trn. Tnh xc sut sao
cho a tr ngi gia hai ngi n b?
Gii .
xp 5 + 8 + 1 = 14 ngi ngi xung quanh mt bn trn th s c (14 1)! = 13! cch xp By gi ta
thy ch c 1 cch xp a tr
Chn ra 2 ngi n b xp ngi hai bn ca a tr nn c C82 = 28 cch chn, v c 2! = 2 cch xp
hai ngi n b ngi hai bn ca a tr.
Vy c c thy 56 cch xp 2 ngi n b ngi hai bn ca a tr.
Cui cng 11 ngi cn li c 11! cch xp vo 11 ch cn li.
Nu gi A l bin c "a tr ngi gia hai ngi n b" th n(A) = 56.11!
56.11! 14
Do P(A) =
= .
13!
39
Cu VIb.1 (1 im)
x 2 y2
+
= 1 c hai tiu im F1 , F2 . Tm mt im M trn (E)
Trong mt phng vi h ta Oxy cho elip (E) :
25 16
1
1
sao cho S = MF1 + MF2 +
+
t gi tr ln nht.
MF1 MF2
Gii .
3

Ta c MF1 = a + ex v MF2 = a ex
1
1
2a
S = MF1 + MF2 +
+
= 2a + 2
MF1 MF2
a e2 x 2

S max th x2 max. Khi x = 5 hoc x = 5 v S =

85
8

Cu VIb.2 (1 im)
x y+2
z
=
=
v mt phng (P) : 2x + y + z 1 = 0
1
2
1
giao nhau ti A. Lp phng trnh chnh tc ca ng thng () i qua A nm trong (P) v to vi (d) mt gc
30 . Gii .
Trong khng gian vi h ta Oxyz cho ng thng (d) :

Cu VIIb. (1 im)
Ngi ta lm ra nhng qu bng dng Italy 90 bng cch khu ghp nhng ming da sn en c hnh ng
gic u v nhng ming da sn trng c hnh lc gic u li vi nhau. Hy tnh xem c mt qu bng th
phi cn bao nhiu ming da sn trng v bao nhiu ming da sn en.
Gii .
Trc ht, qu bng trn l 1 khi a din li , c cc mt l ng gic hoc lc gic
Theo cng thc le th vi D, M,C ln lt l s nh , mt v cnh ca khi a din li th : D + M C = 2
Gi N5 ; N6 ln lt l s mt ng gic(en) v lc gic(trng) ca qu bng
Khi ta c:
-Mi ng gic gip vi 5 lc gic - Mi lc gic gip vi 3 ng gic 5N5 = 3N6
Mi ng gic c 5 nh , mi lc gic c 6 nh , v mi nh l nh chung ca 3 a gic phng
5N5 + 6N6
Do D =
3
Tng t tnh c M = N5 + N6
5N5 + 6N6
v C =
2
5N5 + 6N6
Do vy D + M C = 2 N5 + N6
=2
6
N5
= 2 N5 = 12 Suy ra N6 = 20

6
Vy qu bng c 20 mt trng v 12 mt en
Gi a,b l s lc gic v s ng gic. Khi M = a + b
mi nh u l nh ca ng gic nn D = 5b
Lc gic c 6 cnh, ng gic 5 cnh, mi cnh u tnh 2 ln nn C = 6b+5a
2
Mt lc gic gip 3 ng gic, 1 ng gic gip 5 lc gic nn 3a = 5b
n y s dng Euler: M + D C = 2 s tm ra p s.

TNG HP P N THI TH S 17 CA MATH.VN


(1 im)
1
5
Cho hm s y = x4 3x2 + . Kho st s bin thin v v th (C) ca hm s.
2
2
Gii
Cu I.1

5
1
y = x4 3x2 + .
2
2

2
1
b

1
2

Cu I.2 (1 im)
Tm tt c cc gi tr ca tham s m phng trnh |x4 6x2 + 5| = 2m2 4m c 8 nghim phn bit.
Gii
3
b

1
5
th hm s y = | x4 3x2 + | l:
2
2
Nh vy phng trnh
|x4 6x2 + 5| = 2m2 4m c 8 nghim phn bit
th ta phi c 0 < m2 2m < 2

1 3 < m < 0, 2 < m < 1 + 3

2
1
b

1
2

Cu II.1

(1 im)

3
3
+
3
3
sin 2x. cos 2x +
cos2 2x.
Gii phng trnh sau trn R: sin4 x + cos4 x =
2
2
Gii
Trnh Nga

Phng trnh cho tng ng: 2 sin2 2x = (3 + 3) sin 2x. cos 2x + (2 3 3) cos2 2x

sin2 2x (3 + 3) sin 2x. cos 2x + 3 3 cos2 2x = 0

(3 cos 2x + sin 2x)( 3 cos 2x sin 2x) = 0


n y coi nh xong
The_gunner
!

3 cos 2x
3
sin 2x
(1 cos 2x)2 (1 + cos 2x)2
+
+
= 3 cos 2x

sin 2x cos 2x + cos2 2x

4
4
2
2
2

1
1
= 3 cos 2x sin(2x 3 ) + cos 2x sin(2x + 6 ) = cos 2x cos(2x 3 ) + 3 sin(2x 3 )
2
2
1
t y = 2x 3 th ta c = cos(y + 3 )(cos y + 3 sin y)
2

sin2 y + cos2 y = (cos y 3 sin y)(cos y + 3 sin y) (3 3 + 1) sin2 y (3 3) sin y cos y = 0


1

Cu II.2

(1 im)

2010x4 + x4 x2 + 2010 + x2
= 2010.
Gii phng trnh sau trn R:
2009
Gii
Trnh Nga

Phng trnh cho tng ng: x4 ( x2 + 2010


+ 2010) + x2 2009.2010 = 0
x2 = 2009.2010
x4 (x2 2009.2010)
+ x2 2009.2010 = 0

x4 = 2010 x2 + 2010
x2 + 2010 2010
(2)

x4 = 2010 t 2

Gii (2): t t 2 = x2 + 2010 th ta c h:


Tr tng v ta c: x4 t 4 = (x2 +t 2 )
t 4 = 2010 + x2 (2)

suy ra t 2 x2 = 1 x2 + 2010 (x2 + 2010) + 2009 = 0. n y th n gin.


fanarsenal_vietnam

Phng trnh 2010x4 + x4 x2 + 2010 + x2 + 2010 = 20102

x4 ( x2 + 2010 + 2010) + ( x2 + 2010 + 2010).( x2 + 2010 2010) = 0 x4 + x2 + 2010 = 2010

t x2 = a;
x2 + 2010 = b (a, b 0) Khi ta c b2 a = 2010
Phng trnh ban u tr thnh a2 + b = 2010
Tr v vi v ta c a2 b2 + a + b = 0 (a + b)(a b + 1) =
0 ab+1 = 0 b = a+1
1 + 8037
Thay vo phng trnh u c a2 + a 2009 = 0 a =
2
p

2 + 2 8037
T suy ra phng trnh u c nghim x =
2
Cu III. (1 im)
Tnh din tch hnh phng gii hn bi hai ng y2 = x3 v y2 = (2 x)3 .
Gii
2

Din tch hnh phng l phn gii hn bi 4 th.

p
p
y = x3 ; y = x3 ; y = (2 x)3 ; y = (2 x)3 .
Da vo tnh cht i xng ta suy ra:
1
Z 1
5
2
8
x3 dx = 4 x 2 =
S=4
5 0 5
0

1
1

Cu IV. (1 im)
T mt tm tn hnh vung cnh a(cm) ngi ta mun ct ra mt hnh ch nht v hai hnh trn cng ng knh
lm thn v cc y ca mt hnh tr. Hi khi tr c to thnh c th tch ln nht bng bao nhiu, bit
rng cc cnh ca hnh ch nht song song hoc trng vi cc cnh ban u ca tm tn ?
Gii
Ta c 2 cch ct hnh to thnh hnh tr
Cch 1: Ct thnh 2 phn (x; a) v (a x; a) Phn (x; a) lm 2 y v phn (a x; a) cun dc to thnh
a

Th V = ax2 /4 a3
lm thn ( to thnh hnh tr c chiu cao bng a ). iu kin l x
+1
4( + 1)2
Cch 2: Ct nh trn. Nhng phn (a x; a) cun ngang lm thnh thn (to thnh hnh tr c chiu cao
a
l a x). iu kin l x do chu vi ca hnh trn ct ra phi bng vi phn y ca hnh ch nht.

x2

1
a
V = (a x) a3
Vy c V ln nht th ct x =
2
4
4

Cu V.

(1 im)

Gi s x, y > 0, hy tm gi tr nh nht ca biu thc A =

x3 + y3 + 7xy(x + y)
p
.
xy x2 + y2

Gii
(x + y)3 + 4xy(x + y)
p
A=
xy x2 + y2

p dng bt ng thc Cauchy c (x + y)3 + 4xy(x + y) 4 xy(x + y)2


r
r

p
xy(x + y)2
xy p
xy x2 + y2 + 2xy
2
2
2
2

xy x + y =
2xy(x + y )
=
2
2
2 2
2
Suy ra A 4.2 2 = 8 2
Du bng xy ra x = y
Cu VIa.1 (1 im)
Trong mt phng vi h ta Oxy cho hai im A(3; 1), B(1; 2) v ng thng (d) : x y = 0. Gi s C l
mt im di ng trn (d), gi P, Q l giao im ca cc ng thng AC, BC vi trc honh, trc tung tng ng.
Chng minh rng ng thng PQ lun i qua mt im c nh.
Gii
Gi D l giao im ca PQ v AB.
Theo nh l Menelaus:
DA PC QB
=1
DB PA QC
D thy trong mi trng hp ta u c:
PC QB 1
= c=1
PA QC c
theo nh l Talet. ((c, c) l ta im C)
Vy DA = DB hay D l trung im AB.
Vy PQ lun i qua im c nh l trung im AB.

B
b

Q
b

2
b

1
b

1
1

a = 3.5
b

P
3

Cu VIa.2 (1 im)
Trong khng gian vi h ta Oxyz cho ba im A(3; 1; 2), B(1; 1; 2),C(1; 3; 0). Lp phng trnh ng
trn i qua ba im A, B,C, xc nh tm v bn knh ca ng trn .
Gii
Gi I, J l trung im AB, BC th I(2, 0, 2), J(0, 2, 1). Gi O l tm ng trn i qua 3 im ABC.
Ta c O cch u 3 im ABC nn O nm trn giao tuyn d ca hai mt phng (P) v (Q) (vi (P) l mt
phng trung trc ca AB v (Q) l mt phng trung trc ca (BC), m O thuc mt phng (ABC) nn O l
giao im ca d v (ABC).
D dng vit c phng trnh mt phng (ABC) l x + y 2 = 0

Mt phng (P) nhn BA = (2, 2, 0) lm vct php tuyn v (P) i qua im I(2, 0, 2) nn phng trnh
mt phng (P) l x y 2 = 0
Tng t phng trnh mt phng (Q) l x y z + 1 = 0
Vy
ca h:
ta im O l nghim

x = 2
x + y 2 = 1
y=0
xy2 = 0

z = 3
x y z + 1 = 0

Vy O c ta (2, 0, 3). T d dng tnh c R = 3 3


Cu VIIa. (1 im)
Tm tt c cc s phc z tha mn phng trnh z4 + 3z2 + 3z + 10 = 0.
3

Gii
http://math.vn/showthread.php?t=4715

Cu VIb.1 (1 im)
Trong mt phng vi h ta Oxy cho ng trn (C) : x2 + y2 = 1 v ng thng (d) : ax + by + 1 = 0 tip xc
vi (C). Gi s M, N l hai im thuc (C) tha mn xM = 1, yN = 1. Xc nh a, b tng cc bnh phng
khong cc t M, N n (d) l nh nht.
Gii
(d) tx (C) nn a2 + b2 = 1 D thy M(1; 0), N(0; 1) t a = cos x, b = sin x vi x [0; ]
Tng khong cch t M, N n (d) l S = | a + 1| + |b + 1| = 2 a + b
Hay S = S(x) = 2 cos x + sin x trn [0; 2 ]

7
2
2
Suy ra a =
,b =
Tnh o hm v lp bng bt ta c minS = 2 2 khi x =
4
2
2
Cu VIb.2 (1 im)
Trong khng gian vi h ta Oxyz cho ba im A(3; 2; 5), B(1; 6; 3),C(1; 4; 1). Lp phng trnh ng
trn ng knh AB v i qua im C
Gii
http://math.vn/showthread.php?t=4722

Cu VIIb. (1 im)
Cho n N. Chng minh rng (1 C2n + C4n )2 + (C1n C3n + C5n )2 = 2n , trong Ckn , 1 k n l s t
hp chp k ca n phn t.
Gii
Trnh Nga
1
3
5 +)
Ta c: (1 + i)n = (1 C2n + C4n + ) + i(C
 n Cn + Cnn

n
n
n

+ sin
+ 2n sin
= 2n cos
Li c: (1 + i)n = 2n cos
4
4
4
4
n
Do : (1 C2n + C4n )2 = 2n cos2
4
2 n
1
3
5
2
n
(Cn Cn + Cn ) = 2 sin
4
Cng tng v suy ra pcm.
Smod Xt s phc z = 1 + i Khi zn = (1 + i)n =
Suy ra

|zn | =

Cnk .ik = (1 Cn2 +Cn4 ...) + i(Cn1 Cn3 +Cn5 ...)

k=0
2
4
2
1
(1 Cn +Cn ...) + (Cn Cn3 +Cn5 ...)2

C m |zn | = |z|n =

 n
2 T c c pcm

TNG HP P N THI TH S 18 CA MATH.VN


(1 im)
2x + 1
Cho hm s y =
x1
Gii
Cu I.1

(C). Kho st s bin thin v v th (C) ca hm s.

5
b

n
g

3
2

g
c

Cu I.2 (1 im)
Tm trn hai nhnh ca th im M v N sao cho tip tuyn ti M v N ct hai ng tim cn ti 4 im lp
thnh mt hnh thang.
Gii http://math.vn/showthread.php?t=4985

Ch
u

Cu II.1 (1 im)
Gii phng trnh sau trn R : sin 3x + 2 cos 3x + cos 2x 2 sin 2x 2 sin x 1 = 0
Gii http://math.vn/showthread.php?t=4987
Phng trnh cho tng ng vi
sin x(3 4 sin2 x) + 2 cos x(4 cos2 x 3) 2 sin2 x 4 sin x cos x 2 sin x = 0
sin x(4 cos2 x 1) + 2 cos x(4 cos2 x 1) 4 cos x 2 sin x 4 sin x cos x 2 sin2 x = 0
(sin x + 2 cos x)(4 cos2 x 2 sin x 3) = 0
sin x + 2 cos x = 0 hoc 4 sin2 x + 2 sin x 1 = 0
Cu II.2 (1 im)

Gii phng trnh sau trn R : x3 6 3 6 + 6x 6 = 0.


Gii http://math.vn/showthread.php?t=4988
zinxinh

2 3 2
3
3
3
3
3
2(x + 1) (x + 2) = 0 (x + 1) 2 = x + 2 x 2 + 2 = x + 2 x =
3
21
nguyentatthu

3
3
3
PT h x3+ 6x = (6x
+
6)
+
6
6x
+
6

f
(x)
=
f
(
6x
+
6)

x
=
6x + 6 x3 6x 6 = 0
i
h


i

x 3 2+ 3 4
x2 + x 3 2 + 3 4 + 3 4 + 3 16 2 = 0 x = 3 2 + 3 4

Cu III. (1 im)
p
Tnh th tch ca hnh trn xoay quanh trc honh gii hn bi th hm s y = sin10 x + cos10 x

ng thng x = 0, x =
2
1

Gii http://math.vn/showthread.php?t=4986
Ta c:

Z /2

f (sin x) =

Z /2
0

f (cos x) (d chng minh) nn V =


Z /2

Z /2

Z /2
0

Z

sin x + cos x dx = 2
10

10

/2

sin10 x.

Z /2

sinn xdx. Ta c: In =
sinn xdx =
sinn1 xd(cos x) =
Ta s tnh lun tch phn In =
0
0
0

/2 Z /2
n1
n2
= sin x cos x 0
cos x(n 1)sin x cos xdx = (n 1)In2 (n 1)In
0

(2n 1)!!
(2n)!!
.2I2 , I2n+1 =
.I1
(2n)!!
(2n + 1)!!
63
63 2
Ta tnh c I10 =
. nn V =
(dvtt)
128 4
256
Cu IV. (1 im)
Cho hnh chp t gic S.ABCD, SA vung gc vi y, ABCD l hnh ch nht, SA = 2a, AB = a, AD = 2a, O l
giao im ca hai ng cho. im P l hnh chiu ca O ln SC, Q l trung im SB. Mt phng (APQ) ct SD
ti R. Tnh th tch hnh chp OPQR
Gii http://math.vn/showthread.php?t=4989
Suy ra: nIn = (n 1)In2 Do I2n =

S
b

Q
b

A
b

P
b

b
b

I
B
b

C
b

Chn h trc to Oxyz, gc O trng vi A, AS trng vi Oz, AD trng


vi
 avi Oy,
 ABtrng
 Ox.
a
Theo gi thit ta c to A(0; 0; 0), S(0; 0; 2a), B(a; 0; 0),C(0; 2a; 0), I ; a; 0 , Q ; a; a .
2
2
(I l giao im ca AC v BD).

Tm P: ta c SC = (a;2a; 2a)
 hay uSC = (1; 2; 2)

a
a
u
=
(1;
2;
2)
:
1
x

+ 2(y a) 2z = 0.
(1)
Mt phng (P) qua I ; a; 0 v nhn
SC
2
2
ng thng SC quaS(0; 0; 2a) v nhn
u = (1; 2; 2) lm vtcp
SC

nn c phng trnh tham s (SC) : (x =t), (y = 2t), (z 


= 2a 2t)
(2)
13a 13a 5a
13a
v P
;
;
.
Gii h (1) v (2) cho ta t =
18
18
9
9

 2
8a 4a2

;
; 0 hay
n = (2; 1; 0)
Tm R: ta c n = [AP; AQ] =
9
9

Phng trnh mp(APQ) i qua A(0; 0; 0) nhn


n = (2; 1; 0) lm vtpt nn c pt: 2x y = 0
(3)

ng thng SD i qua S v nhn uSD = (0; 2a; 2a) hay uSD = (0; 1; 1) lm vtcp
nn c phng trnh tham s (SD) : (x = 0), (y = t), (z = 2a t)
(4)
R l giao im ca SD v mp(APQ) chnh l nghimca (3) v (4). Gii h trn ta c t = 0 hay R(0; 0; 2a).
1
a 5


Ta c th tch IPQR l VIPQR = [IP; IR].IQ =
.
6
3
Cu V. (1 im)
Cho a, b, c R v c2 + a2 < 4b. Chng minh a thc: P(x) = x4 + ax3 + bx2 + cx + 1 khng c nghim thc.
Gii http://math.vn/showthread.php?t=4996

2
ax 2  cx
1
+
+1 > 0
P(x) > x4 + ax3 + (a2 + c2 )x2 + cx + 1 = x2 +
4
2
2
Vy a thc P(x) khng c nghim thc.
Cu VIa.1 (1 im)
Trong mt phng Oxy, tam gic ABC ni tip ng trn tm I c ta (6; 6) v ngoi tip ng trn tm K c
ta (4; 5), bit rng A c ta (2; 3). Vit phng trnh cc cnh ca tam gic ABC
Gii
ng trn (I) ngoi tip tam gic ABC
c pt (x 6)2 + (y 6)2 = 25. ng phn gic
AK : x y + 1 = 0 ct (I) ti D(9; 10).
15
b b
[ = DKC
[ = A + C nn
D dng chng minh DCK
2
tam gic DKC l tam gic cn, tng t tam gic
DKB cng l tam gic cn
D
10
Suy ra B,C l giao im ca (I) v ng trn tm
B
D bn knh DK c phng trnh l
(x 9)2 + (y 10)2 = 50
I
Ta B,C l nghim ca h

x2 + y2 12x 12y + 47 = 0
5
b

x2 + y2 18x 20y + 131 = 0

x2 + y2 12x 12y + 47 = 0

6x + 8y 84 = 0

Cu VIa.2

10

Gii h c B(2; 9);C(10; 3) hoc hon v


Suy ra BC : 3x+4y42 = 0; AB : x = 2; AC : y = 3.

15

(1 im)

Trong khng gian vi h ta Oxyz cho


ba im A(4; 1; 2), B(1; 4; 2),C(1; 1; 5).
(x 1)2 + (y 1)2 + (z 2)2 = 9
Tm mt im M nm trn ng trn:
Sao cho MA + MB + MC ln nht
x + y + z 7 = 0

Gii http://math.vn/showthread.php?t=4991
Nhn xt u tin trong bi ton ny l cc im A, B,C thuc ng trn trn

v tam gic ABC u do AB =


BC = CA = 3 2
(x 1)2 + (y 1)2 + (z 2)2 = 9
ng trn c phng trnh
x + y + z 7 = 0
c tm K l hnh chiu ca I trn mp(P) : x + y + z 7 = 0

Do d dng tm c K(2; 2; 3), bn knh r = R2 IK 2 = 6


Ta a bi ton v hnh hc phng . Tam gic ABC u ni tip ng trn tm K .
Khng mt tnh tng qut gi s M nm trn cung BC th theo nh l Ptoleme hoc php quay , ta d dng
chng minh bi ton quen thuc MA = MB + MC

Mt khc MA 2r = 2 6 Du bng xy ra M i xng vi A qua K hay M = M1 (0; 3; 4)


Tng t vi M thuc cung AB, AC tm c 2 im M2 (3; 0; 1); M3 (3; 3; 1)

Khi max(MA + MB + MC) = 2 6


Cu VIIa. (1 im)
Bch ip c 3 hp ng n ging nhau b ngoi. Mt hp ng 2 n mu nu , mt hp ng 2 n mu trng,
mt hp ng 1 n mu trng v mt n mu . C ly ngu nhin mt hp v ly ngu nhin mt n th c
mt n mu trng.
Hi xc xut c ly trong hp c va ly cng c n mu trng.
Gii http://math.vn/showthread.php?t=4994

Cu VIb.1 (1 im)
Trong mt phng vi h ta Oxy cho tam gic ABC c ta cc nh A(1; 8); B(3; 0);C(5; 0).
Xc nh ta cc nh hnh vung ni tip tam gic ABC, bit c hai im nm trn hai cnh AB, AC, cn hai
im cn li nm trn cnh BC.
Gii http://math.vn/showthread.php?t=4992
A
b

8
7

Gi H l chn ng cao h t A xung BC.


Gi di cnh hnh vung l a.
Khi theo nh l Talet ta c:
a MQ BM
AM
MN
a
=
=
= 1
= 1
= 1
8
AH
AB
AB
BC
8
Suy ra a = 4.
T suy ra MQ l ng trung bnh trong tam gic ABH.
T tm c ta cc im ca hnh vung:
Q(2; 0), P(2; 0), M(2; 4), N(2; 4).

6
5

3
2
1

B
4

Q
b

H
b

1
1

Cu VIb.2 (1 im)
Trong khng gian vi h ta Oxyz cho im A(1; 0; 0), B(9; 2; 3) v hai mt cu:
(S1 ) : (x 7)2 + (y 14)2 + (z 3)2 = 400
(S2 ) : (x 18)2 + (y 29)2 + (z 25)2 = 1600
Vit phng trnh mt phng i qua A, B v tip xc vi (S1 ) v (S2 )
Gii http://math.vn/showthread.php?t=4993

Cu VIIb. (1 im)
1 x17 +C2 x16 + ... +C17 x +C18 . Chia ht cho a thc: x3 + 2x2 + 2x + 1.
Chng minh rng a thc: C19
19
19
19
Gii http://math.vn/showthread.php?t=4995

TNG HP P N THI TH S 19
(1 im)
2x
Cho hm s y =
x1
Gii
Cu I.1

(C ). Kho st s bin thin v v th hm s.


5
4

H
n
g

3
2

1
2

Ch
u

N
g
c

Cu I.2 (1 im)
Tm hai im thuc (C ) i xng nhau qua ng thng 2x + y 4 = 0.
Gii http://math.vn/showthread.php?t=5266
Gi s A, B l hai im i xng nhau qua (d) : 2x + y 4 = 0
1
th AB vung gc vi d nn AB c phng trnh l y = x + a
2
1
2x
Phng trnh honh giao im ca AB v (C ) l x + a =
x 2 + (2a 5)x 2a = 0.
2
x1
5 2a
xA + xB
=
Gi M(xM , yM ) l trung im ca AB th ta c: xM =
2
2


xA + xB
1 1
2a + 5
yM =
=
(xA + xB ) + 2a =
2
2 2
4
5 2a 2a + 5
3
M (d) nn 2.
+
4 = 0 suy ra a =
2
4
2
Do xA , xB l nghim ca phng trnh x 2 2x 3 = 0, hay xA = 3, xB = 1 (v hon v).
Vy A(3; 3), B(1; 1)
Cu II.1 (1 im)


Gii phng trnh :
sin x sin 4x = 2 2 cos 6 x 4 3 cos2 x sin x cos 2x.
Gii http://math.vn/showthread.php?t=5267
D=R

sin x sin 4x + sin4x( 3 cos x) = 2 2 cos( 6 x)

sin 4x( 12 sin x + 23 cos x) = 2 cos( 6 x)

sin 4x cos( 6 x) = 2 cos( 6 x) cos( 6 x) = 0


x = 3 + k (k Z)
Cu II.2 (1 im)

2x 2 + 2 x + py 1 34 = 2xy + x
Gii h phng trnh :
2y2 + 2 x + py 1 34 = xy + 2y
Gii http://math.vn/showthread.php?t=5268

fanarsenal_vietnam
iu kin x 2; y 1 Tr tng "
v 2 phng trnh c 2x 2 2y2 = 3xy + x 2y
x = 2y
(x 2y)(2x + y 1) = 0
2x + y = 1
* x = 2y T iu kin ban u suy ra x = 2; y = 1 Thay vo h ta thy khng tha mn

* y = 1 2x Thay vo phng trnh u ca h c 2x 2 + 2 x + 2x 34 = 2x(1 2x) + x

6x 2 3x + 2 x + 2x = 34(?)
Suy ra x 0

Hm s f(x) = 6x 2 3x + 2 x + 2x nghch bin vi x 0


M f(2) = 34 nn x = 2 l nghim duy nht ca pt (?)
x = 2 y = 5 H c nghim duy nht (2; 5)
Quaza (
p

2x 2 + 2 x + y 1 34 = 2xy + x
p

iu kin xc nh: x 2, y 1 Ta c:
2y2 + 2 x + y 1 34 = xy + 2y
"
x 2y = 0
Tr tng v ca hai phng trnh ta c: 2x 2 2y2 = 3xy + x 2y
2x + y = 1
Nu x = 2y th 2 x = 2y 2 nn x = 2, y = 1, khng tha mn h.

Nu 2x + y = 1 th thay vo phng trnh u ta c: 6x 2 3x 34 + 2 x + 2x = 0


2(x + 2)
x+2

= 0 x = 2
3(x + 2)(2x 5)
2x+2
2x + 2
1
1
< 0 do x < 0. Vi x = 2 th y = 5.
V 3(2x 5)

2x+2
2x + 2
Cu III. (1 im)
Z
6
tan 2x(tan4 x 1)dx
Tnh tch phn
0

Gii http://math.vn/showthread.php?t=5269
Z
Z

6
6
2 tan x
1
2
2
2 6
(tan
x

1)(tan
x
+
1)dx
=
2
tan
x.d(tan
x)
=

tan
x
I=
=
2
3
0
0 1 tan x
0
Cu IV. (1 im)
Cho hnh chp S.ABC D c ABC D l hnh ch nht , SA(ABC D), SA = AB = a, AD = 2a,
cho im M thuc SA sao cho AM = x(0 < x < a).
Tm x mt phng (MC D) chia khi chp S.ABC D thnh hai khi c t l th tch l 27
Gii http://math.vn/showthread.php?t=5270
Quaza
t h trc ta Oxyz . Ta cc im l
S(0, 0, 1), A(0, 0, 0), B(0, 1, 0), D(2, 0, 0).C (2, 1, 0), M(0, 0, xM ). i = 5.3
S
D dng tm c phng trnh mt phng MC D l
2(1 xM )
xM (x 2) + 2z = 0 nn dS/(MC D) = q
.
2 +4
xM
M
Li c MDC N l hnh thang vung ti M.
b

Bng nh l Talet v Pitago


q ta tnh c
2 + 4.
MN = 1 xM v MD = xM
q
2 +4
(2

x
)
xM
M
1 2(1 xM )
Do : VS.MDC N = . q
3
2
x2 + 4

N
b

A
b

(1 xM )(2 xM )
=
B
3
2
n y xt hai trng hp VS.MDC N = VS.ABC D hoc
9
7
VS.MDC N = VS.ABC D ri gii ra.
9
b

i = 5.3

S
b

M
b

The_gunner

V CD song song vi AB nn (MC D) song song vi AB


giao tuyn ca (MC D) v (SAB) cng song song vi AB
Dng MN song song vi AB
ax
ax
VS.MC D
=
VS.MC D =
VS.ABC D
VS.AC D
a
2a
2
VS.MC N
(a x)
(a x)2
=

V
=
VS.ABC D
S.MC D
a2
2a2
D VS.ABC
2a2 3ax + x 2
VS.MNC D =
VS.ABC
2a2
2
V (MDC ) chia hnh chp ra 2 phn c t l th tch
7
2a2 3ax + x 2
2
7
2a2 3ax + x 2
= hoc
=
nn
9
9
2a2
2a2
Kt hp vi iu kin 0 <
x < a ta c
27 585
2a
hoc x =
x=
3
18a
b

b
b

b
b

Cu V. (1 im)
Cho x, y, z l cc s thc tha mn : xyz = 1. Tm gi tr ln nht ca biu thc
S=

y2 z 2
z2x 2
x 2 y2
+
+
x 4 y2 + x 2 y2 + 1 y4 z 2 + y2 z 2 + 1 z 4 x 2 + z 2 x 2 + 1

Gii http://math.vn/showthread.php?t=5271
buihuuloc
1
1
1
+ 2
+ 2
t x 2 = a3 ; y2 = b3 ; z 2 = c3 abc = 1
Ta c S = 2
2
2
x + z + 1 y + x + 1 z + y2 + 1
1
1
1
S= 3
+ 3
+ 3
Ta c :a3 + c3 + 1 > ac (a + c) + 1 = ac (a + b + c)
3
3
a + c + 1 b + a + 1 b + c3 + 1
b
1
1
=
3
6
3
ac (a + b + c)
a+b+c
a +c +1
Tng t vi 2 biu thc cn li cng vo ta c S 6 1 Du = xy ra x = y = z = 1
CSS_MU
1
1
1
D thy S = 2
+ 2
+ 2
.
2
2
x + y + 1 y + z + 1 z + x2 + 1
S dng bt ng thc Cauchy-Schwarz, ta c
P 1 + 1 + z2
P
6 + x 2 + y2 + z 2
1 + 1 + z2

=
.
S=
(x 2 + y2 + 1)(1 + 1 p
+ z2)
(|x| + |y| + |z|)2
(|x| + |y| + |z|)2
Mt khc, ta li c 6 = 2 3 3 x 2 y2 z 2 2(|xy| + |yz| + |zx|),
6 + x 2 + y2 + z 2
2(|xy| + |yz| + |zx|) + x 2 + y2 + z 2
nn kt hp vi trn, ta suy ra S

= 1.
(|x| + |y| + |z|)2
(|x| + |y| + |z|)2
Vy gi tr ln nht ca S l 1. ng thc c c khi v ch khi |x| = |y| = |z| = 1(xyz = 1).
Cu VIa.1 (1 im)
Trong mt phng Oxy, Cho hai ng trn (C1 ) : (x 3)2 + (y 1)2 = 5, (C2 ) : x 2 + y2 4x + 2y = 5,
Gi A l giao im ca (C1 ); (C2 ) vi yA > 0.
Vit phng trnh ng thng qua A ct (C1 ); (C2 ) ln lt ti M, N sao cho AN = 2AM.
Gii http://math.vn/showthread.php?t=5272

fanarsenal_vietnam

D dng tm c(C 1) c tm I1 (3; 1) bn knh R = 5 A(1; 2)


Qua php v t tm A t s k = 2 ng trn (C 1) bin thnh ng trn (C10 ).

D dng tm c (C10 ) c tm I10 (3; 4); bn kinh R 0 = 2R = 2 5


Phng trnh (C10 )(x + 3)2 + (y 4)2 = 20
0
Ct tuyn AMN chnh l giao tuyn ca (C2 ); (C
1)
x 2 + y2 4x + 2y = 5
Cc im trn ct tuyn c ta l nghim h
x 2 + y2 + 6x 8y = 5
10x 10y = 10 x y + 1 = 0 y chnh l phng trnh ng thng AMN
Cu VIa.2 (1 im)

x = 1 + t
x = 2 u

Trong khng gian Oxyz, Cho d1 : y = 2 + 2t , d2 : y = 3 u

z = 1 t
z = 2

Vit phng trnh mt phng (P) vung gc vi d1 sao cho (P) chn d1 , d2 on thng c di nh nht.
Gii http://math.vn/showthread.php?t=5273

(P) vung gc vi d1 nn (P) c vct php tuyn


n = (1; 2; 1) nn
 (P) c pt dng x + 2y

 z+ d = 0
2 d 2 d 10 + d
4 d 1 d
Gi M l giao ca (P) vi d1 v N l giao ca (P) vi d2 suy ra M
;
;
;
; 2
,N
6
3
6
3
3
d2 16d 155
+
+
do MN dng nn MN nh nht khi MN 2 nh nht tc l d = 16
t ta c : MN 2 =
18
9
9
M(3; 6; 1), N(4; 5; 2). (P) qua M nn (P) c pt x + 2y z 16 = 0
Cu VIIa. (1 im)

Tm cc s phc z c modun bng 10 tha mn |z 4 + 3i| = 2


Gii http://math.vn/showthread.php?t=5276

|z| = 10
x 2 + y2 = 10
x 2 + y2 = 10
t z = x + yi vi x, y l cc s thc. Ta c:

|z 4 + 3i| = 2
(x 4)2 + (y + 3)2 = 4
8x 6y = 31
Gii h thu c kt qu.
Cu VIb.1 (1 im)
Trong mt phng Oxy, Cho tam gic ABC c A(5; 0) , trung tuyn C M v trung trc on BC c phng trnh ln lt l
3x + 4y 13 = 0, x y + 3 = 0 . Tm ta cc nh B, C .
Gii http://math.vn/showthread.php?t=5274
Gi N
l giao im ca trung tuyn C M v trung trc ca on thng BC , khi ta N l nghim ca h phng


3x + 4y 13 = 0
1 22
trnh
ta tm c N =
;
. T d dng tm c ng thng i xng vi ng thng
x y + 3 = 0
7 7
C M qua trung trc ca on thng
 BC l ng
 thng c phng trnh 4x + 3y 10 = 0, y
 chnh l ng
 thng i qua
10 4b
b + 5 5 2b
B, do ta im B c dng b,
,
, do trung im M ca AB c ta l
, m M thuc
3
2
3
5 2b
b+5
+ 4.
13 = 0, suy ra b = 1, vy B(1; 2), t tm c C (1; 4)
ng thng C M nn ta c: 3.
2
3
Cu VIb.2 (1 im)
Trong khng gian Oxyz, Cho mt cu (S) : x 2 + y2 + z2 2x 4y
 + 6z + 5 = 0,
3 3 1
y1
z+3
x+1
=
=
v im M
; ;
ng thng d :
.
2
1
2
2 2 2
3
Vit phng trnh mt phng (P) song song vi d , tip xc vi (S) bit khong cch t M n (P) bng .
2
Gii http://math.vn/showthread.php?t=5275

3
Mt cu c tm I(1; 2; 3) bn knh R = 3 Khong cch t M n (P) = , Khong cch t I n (P) = R = 3 Gi K l
2 

1

4 5 4
giao im ca MI v (P) d thy MK = MI suy ra to K l
; ;
3
3 3 3


4 5 4
; ;
v c vect php tuyn (A; B; C ) c dng A(3x 4) + B(3y 5) + C (3z + 4) = 0
(P) qua K
3 3 3
T cc gi thit
(P) vung gc d c V T C P(2; 1; 2) v khong cch tm I n (P) = R

| A + B 5C | = 3 A2 + B2 + C 2
ta a v h pt
2A + B 2C = 0

chn B = 2 v gii h ta c cc cp (A; B; C ) l (1; 2; 2) v (2; 2; 1)


suy ra pt ca (P)
Cu VIIb. (1 im)

1 log x 2 1 log y8 = 0
3
4 9
(x, y R)
Gii h phng trnh : 2
|x|3 + y2 2y = 0
Gii http://math.vn/showthread.php?t=5277

TNG HP P N THI TH S 21 CA MATH.VN


(1 im)
2m x
Cho hm s y =
c th l (Cm ) (m l tham s).
x+m
Kho st s bin thin v v th ca hm s khi m = 1
Gii
Cu I.1

2
b

n
g

y=

2x
x+1

1
b

1
2

g
c

Ch
u

Cu I.2 (1 im)
Gi I l giao im hai tim cn ca (Cm ) v A(0; 1). Tm gi tr ca tham s m trn (Cm ) tn ti im B
sao cho tam gic AIB vung cn ti A.
Gii
2m x0
)
im I(m; 1) l giao ca 2 ng tim cn B (Cm ) B(x0 ;
x0 + m
m 2x0

Tam gic ABI vung cn ti A nn ABAI AI AB = 0 AI = (m; 2); AB = (x0 ;


)
x0 + m
2(m 2x0 )
= 0 (1)
Suy ra mx0 +
x0 + m
mx02 + (m2 4)x0 + 2m = 0 Pt c nghim khi m = 0; x = 0 (loi) hoc m khc 0 v = m4 16m2 + 16 0

m2 8 + 4 3 |m| 2 + 6 (*)
m 2x0 2
)
Mt khc AI = AB nn AI 2 = AB2 m2 + 4 = x02 + (
x0 + m
m 2x0 mx0
T (1) suy ra
=
x0 + m
2
m2
Do pt trn tng ng vi m2 + 4 = (1 + )x02 x02 = 4
4
Nu x = 2 th m2 + 3m 4 = 0 m = 1 (loi) hoc m = 4 (tho mn (*))
Tng t nu x = 2 th m = 4
Vy c 2 gi tr ca m tho mn iu kin bi ra l m = 4; m = 4
Cu II.1 (1 im)

Gii phng trnh sau trn R: tan2 x + 3 = (1 + 2 sin x)(tan x + 2 cos x)


Gii
fanarsenal_vietnam

pt tan2 x + 2 cos2 x + 2 sin2 x + 1 = (tan x + 2 cos x)( 2 sin x + 1)


()
2
2
2
2
Vi a; b; c; d R ta c a + b + c + d (a + b)(c + d)
1

Tht vy bt (a c)2 + (a d)2 + (b c)2 + (b d)2 0;ng


Du bng xy ra a = b = c = d

p dng bt trn vo phng trnh (*) suy ra tan x = 2 cos x = 2 sin x = 1 x = + k2


4
Wall-E

KX: x 6= + k (k Z)
2

1 + 2 cos2 x (1 + 2 sin x)(sin x + 2 cos2 x)


=
PT
cos2 x
cos x
2

1 + 2 cos x
= (1 + 2 sin x)(sin x + 2 2 sin2 x)

cos x

Ta c V P = (1 + 2 sin x)(1 + 2 sin x)( 2 sin x) 0 do V T 0 cos x > 0

1
1
1
(3 2)3
Nu sin x ,theo BDT Cauchy V P = ( + sin x)( + sin x)(2 2 2 sin x)
=2 2
27
2
2
2
1
Du bng xy ra sin x =
2

1
Nu sin x < th V P < (1 2)2 ( 2 + 1) = 2 1 < 2 2
2

M 1 + 2 cos2 x 2 2 cos x > 0 suy ra V T 2 2


1
Du bng xy ra cos x =
2
1

Do V T = V P sin x = cos x = x = + k2 (k Z) (tho mn KX)


4
2

Vy pt ban u c 1 h nghim x = + k2 (k Z)
4
hoangda

Pt tng ng vi sin2 x + 3 cos2 x = cos x(1 + 2 sin x)(sin x + 2 cos2 x)

2 + cos 2x = cos x(1 + 2 sin x)(sin x + 2(1 sin2 x))

2 + cos 2x = cos x(1 + 2 sin x)(sin x(1 2 sin x) + 2)

2 + cos 2x = cos x(sin x cos 2x + 2 sin x + 2)

2 + cos 2x = sin x cos x(cos 2x + 2) + 2 cos x

(2 + cos 2x)(2 sin 2x) = 2 2 cos x


cos x
cos x
Ta thy
=
1
2 + cos 2x 1 + 2 cos2 x
1
2 sin 2x 1 Du bng xy ra khi cos x = , sin 2x = 1
2

nn pt c nghim x = + k2
4
Cu II.2 (1 im)

2x3 y2 3x2 y2 + y2 = x3
Gii h phng trnh sau trn R:
2x2 y + x2 y = 0
Gii
Nu x = 0 th y = 0 v ngc li Nu x, y khc 0.

1
1
Chia c 2 v pt (1) cho x3 y2 v pt (2) cho x2 y ri t a = ; b = (a, b 6= 0) th h cho tr thnh
x
y

2 3a + a3 = b2 (1)
2 + b a2 = 0(2)

Suy ra (a 2)(a3 + a2 2a 1) = 0
1
Nu a = 2 th b = 2 x = y =
2
Nu a3 + a2 2a 1 = 0 (). Pt () c nhiu nht 3 nghim
2

Vi 2 < a < 2 th tn ti t (0; ) sao cho a = 2 cost


Khi pt tr thnh 8 cos3 t + 4 cos2 t 4 cost 1 = 0 2 cos 3t + 2 cos 2t + 2 cost 1 = 0
t
Nhn c 2 v pt trn vi sin 6= 0 v p dng cng thc bin i tch thnh tng c
2
7t
5t
5t
3t
3t
t
t
7t
2k
sin sin + sin sin + sin sin + sin = 0 sin = 0 t =
(k Z)
2
2
2
2
2
2
2
2
7
4
6
2
4
6
2
;t =
;t =
a = 2 cos ; a = 2 cos ; a = 2 cos
l 3 nghim ca ()
V t (0; ) nn t =
7
7
7
7
7
7
Suy ra ptban u c 3 h nghim :

(x, y) =

2
2
2 cos
4 cos2
7
7
Cu III. (1 im)

Tnh tch phn

I=

Z e
1


2 ;

4
4
2 cos
4 cos2
7
7


2 v

6
6
2 cos
4 cos2
7
7

x ln x
dx
(1 + x2 )2

Gii




Z
Z
Z 
dx
1
1
1 e 1
x
ln x e 1 e
1 e
+
ln x.d 2
=
+

=
dx
I=
2 1
2(1 + 
x2 ) 1 2 1 x(x2 + 1) 2(e2 + 1) 2 1 x x2 + 1
x + 1
e
1
1
1 1
1
1
1
2
=
=
+
ln(x
+
1)
+ ln(e2 + 1) + ln 2
ln
x


2
2
2(e + 1) 2
2
2(e + 1) 2 4
4
1
Cu IV. (1 im)
Cho hnh chp S.ABCD c y ABCD l hnh vung cnh a, cnh SA vung gc vi mt y, SA = a. Gi s
[ = 45 . Tm v tr ca M v N sao
M, N l hai im di ng trn cc cnh BC,CD tng ng sao cho MAN
cho tng th tch cc khi chp S.BAM, S.MCN, S.NDA t gi tr ln nht.
Gii
V VS.ABCD l c nh nn tng th tch cc khi S.BAM, S.MCN, S.NDA max VS.AMN min SAMN min
x y
+
y
x
a
tan MAB = v tan NAD = v MAB + NAD = 45o tan 45o = a xy
a
a
1 2
a

(x + y)2
2
2
a(x + y) = a xy a
x + y 2a( 2 1)
4

1
1
2p 4
SAMN = AM.AN. sin 45o =
a + a2 (x2 + y2 ) + x2 y2 = a(x + y) a2 ( 2 1)
2
2
4
Du = xy ra khi x = y = a( 2 1)
Cu V. (1 im)
Cho a, b, c l cc s thc sao cho (a + b)(b + c)(c + a) 6= 0.
bc
ca
1
4abc
ab
+
+
+
.
Chng minh rng
2
2
2
(a + b)
(b + c)
(c + a)
4 (a + b)(b + c)(c + a)
Gii
b
c
a
=x;
= y v
=z
t
a+b
b+c
c+a
ta c ngay iu kin xyz = (1 x)(1 y)(1 z) 2xyz + x + y + z = 1 + xy + yz + zx
1
BT x(1 x) + y(1 y) + z(1 z) + 4xyz
4


9
3 2
2
2
2
x + y + z + 2xy + 2yz + 2zx 3(x + y + z) + 0 x + y + z
0 ( ng )
4
2
b
c
3
a
+
+
=
Du = xy ra vi mi a, b, c R v tho mn
a+b b+c c+a 2
Cu VIa.1 (1 im)

Trong mt phng vi h ta Oxy cho hai im ba ng thng (d1 ) : 2x 3y + 5 = 0, (d2 ) : x + 4y 6 = 0,


(d3 ) : x + y 4 = 0. Tm ta cc nh ca hnh vung ABCD bit rng A (d1 ),C (d2 ), B, D (d3 ).
Gii
+ B, D d3 B(4 b, b), D(4 d, d)
+ A d1 A(3a 5/2, 2a)
+ C d2 C(6 4c, c)

+ Ta c: - BD AC AC
u3 vi AC = (4c 3a 17/2, c 2a),
u3 = (1, 1)

AC. u3 = 0 a + 5c = 17/2(1)
- Trung im AC l I((3a 4c + 3, 5)/2, (2a + c)/2) d3 : x + y 4 = 0
(3a 4c + 3, 5)/2 + (2a + c)/2 4 = 0 5a 3c = 9/2(2)
- T (1)(2) a = 12/7, c = 19/14 A(37/14, 24/7),C(4/7, 19/14)
+ Tm I(45/28, 67/28) l trung im BD b + d = 67/14(3)
+ IA2 = IB2 (29/28)2 + (29/28)2 = (b + 67/28)2 + (b 67/28)2

vi IA = (29/28, 29/28) v IB = (b + 67/28, b 67/28)


2b2 67/7b + 456/49 b = 24/7vb = 19/14(4)
T (3)(4) b = 24/7, d = 19/14 B(4/7, 24/7), D(37/14, 19/14)
b = 19/14, d = 24/7 B(37/14, 19/14), D(4/7, 24/7)
Cu VIa.2 (1 im)
Trong khng gian vi h to Oxyz cho A(3; 3; 1), B(0; 2; 1) v mt phng ( ) : x + y + z 7 = 0 . Gi ()
l ng thng nm trong ( ) v mi im ca () cch u hai im A v B. Tm to im C thuc ()
sao cho tam gic ABC c din tch nh nht.
Gii
T gt () l giao tuyn ca mp ( ) v mp trung trc AB

+ Gi ( ) l mt phng trung trc ca AB ( ) qua I(3/2, 5/2, 1) v VTPT AB = (3, 1, 0) ( ): 3x+y7 = 0


+ Phng trnh ta g ( ) v ( ): ( ): x + y + z 7 = 0 ( ): 3x + y 7 = 0 M(0, 7, 0)
y7
z
x

=
=
+ () qua M(0, 7, 0) VTCP [AB,
n ] = (1, 3, 2) vi
n = (1, 1, 1) ():
1
3
2

+ C () C(t, 7 + 3t, 2t) AC = (t 3, 3t + 4, 2t 1) AB = (3, 1, 0)


1
1

1 p
+ SABC = [AC, AB] =
(2t + 1)2 + (6t + 3)2 + (10t + 15)2 =
140t 2 + 340t + 235
2
2
2
Xt f (t) = 140t 2 + 340t + 235 f 0 (t) = 280t + 340 f 0 (t) = 0 t = 17/14
SABC nh nht f (t) nh nht t = 17/14 C(17/14, 47/14, 17/7)
Cu VIIa. (1 im)
Gi z1 , z2 l hai nghim ca phng trnh 2010z2 2009z + 2010 = 0. Tnh gi tr M = |1 z1 .z2 |2 |z1 z2 |2

Gii
Phng trnh trn c 2 nghim phc z1 ; z2 Khi ta c z1 = z2 ; z1 .z2 = 1
M = |z1 .z2 z22 |2 |z1 z2 |2 = |z1 z2 |2 (|z1 )2 1| = |z1 z2 |2 .|z1 .z1 1| = 0
Cu VIb.1 (1 im)
Trong h Oxy cho ng thng (d) : x + 2y + 2 = 0 ct ng trn (C) : x2 + y2 2x + 4y + 4 = 0 ti A, B.
Gi 1 , 2 ln lt l tip tuyn ca ng trn (C) ti A, B. Tnh tch cc khong cch t im M(1; 1)
n 1 , 2 .
Gii
+ Ta c phng trnh ta giao im ca (d) v (C):
(d) : x + 2y + 2 = 0 (C) : x2 + y2 2x + 4y + 4 = 0 A(6/5, 2/5) v B(2, 0)

+ (C) c tm I(1, 2) Ta c IA = (11/5, 8/5) v IB = (3, 2)


4

+ Gi s A 1 , B 2 1 qua A(6/5, 2/5) VTPT IA = (11/5, 8/5) hay


n = (11, 8)
1 : 11(x + 6/5) + 8(y + 2/5) = 0 1 : 11x + 8y 10 = 0

2 qua B(2, 0) VTPT IB = (3, 2) 2 : 3(x + 2) + 2y = 0 2 : 3x + 2y 6 = 0


319
| 11x + 8y 10| | 3x + 2y 6|
. p
=
+ M(1, 1) d(M, 1 ).d(M, 2 ) = p
2405
(11)2 + 82
(3)2 + 22
Cu VIb.2 (1 im)
Trong khng gian vi h ta Oxyz cho h mt phng (Pm ) : 2mx + (3m 1)y (m + 1)z 1 = 0, vi m l
tham s. Tm qu tch cc im Hm l hnh chiu vung gc ca im H(1; 1; 3) trn (Pm ).
Gii
Mt phng (Pm ) c pt 2mx + (3m 1)y (m + 1)z 1 = 0
pt m(2x + 3y z) y z 1 = 0
Suy ra (Pm ) lun i qua cc im c to tho mn h phng trnh

2x + 3y z = 0
y + z + 1 = 0

1
t y = t th z = 1 t; x = 2t (Pm ) lun i qua ng thng() c nh c phng trnh
2

x = 2t

c vtcp
u = (2; 1; 1)
y
=
t

z = 1 t
im Hm l hnh chiu ca H trn (Pm ) HHm suy ra Hm thuc mp(Q) qua H(1; 1; 3) v vung gc vi

(Q) nhn
u lm vtpt pt ca mp(Q) : 2(x 1) (y 1) + (z 3) = 0 2x y + z 4 = 0
(Q)
ct ti im K c to l nghim ca h pt
1

x
=

2t

y=t

z = 1 t

2x y + z 4 = 0
3
1 4t t 1 t 4 = 0 6t = 6 t = 1 Suy ra K( ; 1; 2)
2
Mt khc HHm Pm suy ra HHm KHm nn Hm thuc mt cu (S)
r ng knhHK
1
101
5 1
25
(S) c tm I l trung im HK I( ; 1; ) v bn knh HI =
+
=
4 2
16
4
4
Do vy cc im Hm tho mn iu kin bi ra lun
thuc
ng
trn
tm
I
l
giao
tuyn ca mp(Q) v (S).

2x y + z 4 = 0
Cc im Hm c to tho mn h phng trnh
(x 5 )2 + (y 1)2 + (z 1 )2 = 101
4
2
16
Cu VIIb. (1 im)
z + |z|
Cho s phc z tha mn z + z = 2|z|2 . Tm tp hp cc im biu din ca s phc w =
.
1 + |z| + |1 z|
Gii

LI GII THI TH S 23 CA MATH.VN

Cu II.1

(1 im)

Gii phng trnh sau trn R:

n
v
.
h

Cu I.1 (1 im)
Cho hm s y = x3 + 3x2 3. Kho st s bin thin v v th (C) ca
hm s.
Li gii

Cu I.2 (1 im)
Tm cc im A, B,C, D trn (C) sao cho ABCD l mt hnh vung tm
I(1; 1).

t
a

Li gii

Chuyn h to theo vect OM(1; 1) th ta c h trc to mi MXY


x = X + 1 v y = Y 1
Phng trnh ng cong (C) i vi h to MXY l
Y 1 = (X + 1)3 + 3(X + 1)2 3 Y = X 3 + 3X
Gi 2 phng trnh ng cho ca hnh vung mi A0 B0C0 D0 l
1
Y = kX v Y =
X
(k 6= 0)
k
A,C c to l nghim ca h

X 3 + 3X = kX XA0 = 3 k v XC0 = 3 k
B, D c to l nghim car
h
r
1
1
1
X 3 + 3X =
X XB0 = 3 + v XD0 = 3 +
k
k
k
K A0 B0C0 D0 l hnh vung l A0 B0 vung gc A0 D0 v A0 B0 = A0 D0

k4 3k3 + 3k + 1
= 0 k = 1+ 2

1+ 5
1 5
k = 1 2 k =
k=
2
2
Th li cc nghim ta thy tho mn. Sau thay k vo tm uc A0 , B0 ,C0 , D0 .
T ta tm c to cc im A, B,C, D

1
1 cos x. cos 2x

= 4 sin2 xsin x1.


sin 2x
cos x

Li gii
Cch 1
1
1 cos x(2 cos2 x 1) cos x 1
1 cos x. cos 2x

+1 =
+
Ta c
sin 2x
cos x
sin 2x
cos x
(2 cos2 x + 2 cos x + 1)
1
= (1 cos x)(

)
sin 2x
cos x
2 cos x(1 cos2 x) 1 cos x
+
(1 2 sin x)
=
2 sin x cos x
sin 2x
1 cos x
= sin x +
(1 2 sin x)
sin 2x
PT cho tng ng vi
1 cos x
sin x +
(1 2 sin x) = 4 sin2 x sin x
sin 2x
1 cos x
(1 2 sin x) = sin x(4 sin x 2)
sin 2x
1 2 sin x = 0 hoc 1 cos x = 4 sin2 x cos x
cos x(1 4 sin2 x) = 1 cos 3x = 1
Cch 2
K ............
1 cos x cos 2x + sin x sin 2x
1

=
+ (1 2 cos 2x)
sin 2x
cos x
1 cos 3x 1 + cos x 2 cos x cos 2x

=
sin 2x
cos x
1 cos 3x 1 + cos x (cos x + cos 3x)

=
sin 2x
cos x
cos 3x = 1 hoc cos x = sin 2x

Cu II.2 (1 im)

Gii h phng trnh sau trn R: 5x2 + 4x x2 3x 18 = 5 x.


Li gii
Cch 1

x2 3x 18 0
x6
x0

5x2 + 4x = 5 x + x2 3x 18
p
5x2 + 4x = 25x + x2 3x 18 + 10 x (x2 3x 18)
p
2x2 9x + 9 = 5 x (x2 3x 18)
2

2x2 9x + 9 = 25x x2 3x 18
4x4 61x3 + 192x2 + 288x + 81 = 0

(4x + 3) (x 9) x2 7x 3 = 0

3
x=

x=9
x=9

61
7
7
61
x= +
x= +

2 2
2
2

7
61
x=
2
2
Cch 2
K: x 6. i v kh du bnh phng gim cn c
p
2x2 9x + 9 = 5 x(x 6)(x + 3)

t a = x2 6x v b = x + 3 th
2a2 5ab + 3b2 = 0 hay (a b)(2a 3b) = 0

I=
0

Z e1

n
v
.
h
Z e1

Ta c
0

2 ln (x + 1)d (ln (x + 1))


e1


Z e1

x
2
dx + ln (x + 1)
= e12 e1
x+1
0
0
 

Z e1 
1
dx + 1
= e12 e1
1
x+!
1
0
e1

= e 2 e 1 (x ln (x + 1))
0

ln2 (x + 1)dx

e1 Z e1


xd ln2 (x + 1)
ln (x + 1)dx = x.ln (x + 1)
0
2

dx

Li gii
Th tch vt th trn xoay c to ra khi quay H quanh trc Ox.
Z e1

Z0e1  x + 1


ln (x + 1)
= e12
ln (x + 1)
dx
x+1
0
Z e1
Z e1
ln (x + 1)
= e12
ln (x + 1)dx + 2
dx
x+1
0
0
!
e1 Z e1
=
e

2 x ln (x + 1)
xd ln (x + 1)
0

t
a

Cu III. (1 im)
Gi H l hnh phng gii hn bi th hm s y = ln(x + 1), trc honh
v ng thng x = e 1. Tnh th tch vt th trn xoay c to ra khi
quay H quanh trc Ox.

= e12

Z e1
x ln (x + 1)

= e 2 I = (e 2)

Cu IV. (1 im)
Cho hnh lng tr ng ABC.A0 B0C c y ABC l tam gic vung ti A
v ng cao AH = a. Mt phng (ACB0 ) hp vi y mt gc v cch
im B mt khong l a. Xc nh gi tr ca th tch ca khi lng
tr ABC.A0 B0C t gi tr nh nht.
Li gii
Cu V. (1 im)
Cho x, y, z > 0 tha mn (x y)2 + (y z)2 + (z x)2 = 2. Chng minh
rng


1 1 1
2(x + y + z 1)
(x + y + z)
+ +
9+
.
x y z
3xyz

Li gii

Cch 1
BT cn chng minh tng ng vi
3[ x2 (y + z) 6xyz] + 2 (x + y + z)[(x y)2 + (y z)2 + (z x)2 ]
(2z x y)(x y)2 + (2x z y)(z y)2 + (2y x z)(x z)2 + 2 0
Ti y th bi ton tr nn n gin ri ;) . t a = x y; b = y z; c = z x th
a + b + c = 0 v a2 + b2 + c2 = 2
BT trn tr thnh (c b)a2 + (b a)c2 + (a c)b2 + 2 0
(a b)(a c)(b c) 2
t P = |(a b)(a c)(b c)|
T iu kin ca a,b,c suy ra a2 + b2 + (a + b)2 = 2 a2 + b2 + ab = 1
Trong 3 s a, b, c lun c 2 s cng du theo nguyn l Dirichle. Khng mt tnh
tng qut gi s 2 s l a, b th ab 0
Ta c P2 = (a2 + b2 2ab)(2a + b)2 (2b + a)2 = (1 3ab)(2 + 3ab)2
p dng BT Cauchy cho 3 s dng :
3ab
3ab
33
)(1 +
)
=1
(1 3ab)(1 +
2
2
27
Suy ra P2 4
Du bng xy ra ab = 0 a = 0 hoc b = 0
Khi tng ng b = c = 1; 1 hoc a = c = 1; 1
Do P 2 l BT ng. Suy ra BT ban u ng
Cch 2
BT cn chng minh tng ng vi
3[ x2 (y + z) 6xyz] + 2 (x + y + z)[(x y)2 + (y z)2 + (z x)2 ]
(2z x y)(x y)2 + (2x z y)(z y)2 + (2y x z)(x z)2 + 2 0
Ti y th bi ton tr nn n gin ri ;) . t a = x y; b = y z; c = z x th
a + b + c = 0 v a2 + b2 + c2 = 2
BT trn tr thnh (c b)a2 + (b a)c2 + (a c)b2 + 2 0
n ch ny bin i vi b + c = a v bc = a2 1
(3a2 1)(c b) + 2 0
c, b l nghim ca h phng trnh b + c = a v b2 + c2 = 2 a2

2
2
K c nghim l a
3
3

v c b = 4 3a2 hoc c b = 4 3a2

Xt hm f (a) = (3a2 1)( 4 3a2 ) + 2 vg(a) = (3a2 1)( 4 3a2 ) + 2


th f (a) 0. Du = khi a = 0
g(a) 0. Du = khi a = 1 hoc a = 1
T suy ra BT u ng, du bng xy ra khi (x; y; z) = (2; 1; 1) v cc hon v
Cch 3
Gi thit c th vit li x2 + y2 + z2 (xy + yz + zx) = 1
()
(x + y + z)2 1 = 3(xy + yz + zx)
Bt ng thc cn chng minh l
3(x + y + z)(xy + yz + zx) 27xyz + 2(x + y + z) 2
(x + y + z)[(x + y + z)2 1] 27xyz + 2(x + y + z) 2
(x + y + z)3 27xyz + 3(x + y + z) 2
khng mt tnh tng qut gi s x = max(x; y; z)
Khi dt y + z = 2t. Hin nhin t x
[2x (y + z)]2
Ta c 2 = (x y)2 + (x z)2 + (y z)2 (x y)2 + (x z)2
2
2x 2t 2 x t + 1
T () ta cng c x2 + (y + z)2 x(y + z) 3yz = 1
Suy ra 3yz = x2 + 4t 2 2xt 1
Bt ng thc tr thnh (x + 2t)3 9x(x2 + 4t 2 2xt 1) + 3(x + 2t) 2
x3 + 6x2t + 12xt 2 + 8t 3 9x3 + 36xt 2 18x2t 6x + 6t 2
8x3 24x2t + 24xt 2 8t 3 6(x t) 2 0
4(x t)3 3(x t) 1 0 [(x t) 1][(2(x t)2 + 1]2 0
Bt ng thc ny ng bi x t + 1
Du bng xy ra x = t + 1
Khi (y = z), (x = y + 1) : v cc hon v

n
v
.
h

t
a

Tng t M 2 = 16|u|2 + 9|v|2 + 12(uv + vu)


Do M 2 + N 2 = 25(|u|2 + |v|2 ) = 5000

Suy ra M 2 = 5000 2010 = 2990 M = 2990 > 0

Cu VIa.1 (1 im)
Trong mt phng vi h ta Oxy cho hai im M(4; 1), N(0; 5) nm
trn hai ng thng AB, AC tng ng v ng thng (d) : x 3y + 5 = 0
cha ng phn gic trong gc A ca tam gic ABC. Tm ta cc nh
2 5
A, B, C bit trng tm G( ; ).
3 3

n
v
.
h

Cu VIb.1 (1 im)
Trong mt phng vi h trc ta Oxy cho tam gic ABC c phng
trnh cc ng thng (AB) : x y + 2 = 0; (AC) : 2x + y + 1 = 0; (BC) :
3
4x y 7 = 0. Lp phng trnh ng thng (d) i quaim M( ; 6) v
2
chia tam gic ABC thnh hai phn c din tch bng nhau.

Li gii
Gi M1 , N1 ln ltl ccim 
i xng
 vi M, N qua d. T ta thy M1 AC
8 31
4
v N1 AB v M1
,
, N1
5 5
17


Li gii.
12 269
Ta suy ra c pt AB : 9x + 4y + 32 = 0 v AC : 7x y 5 = 0 A ,
37
37
Ta d dng tm c A(1; 1); B(3; 5);C(1; 3).


32 9b
v C(c; 7c 5) ,ta c ABC c trng tm G nn tnh c T hnh v ta d dng nhn ra ng thng d cn lp s phi ct cnh AB v AC
ta gi B b;
4
S 0 0
1




ln lt ti B0 ;C0 sao cho AB C =
552 12194
1742 19039
SABC
2

B
;
v C
;
1369 1369
1369
1369
x = 1 + t
x = 1 + u
pt tham s AB :
pt tham s ca AC :
y = 1 + t
y = 1 2u
Cu VIa.2 (1 im)
Gi s B0 (1 + t; 1 + t);C0 (1 + u; 1 2u)
Trong khng gian vi h ta Oxyz cho mt phng (P) : 2x y+z3 = 0


Ta thy AB = (4; 4); AB0 = (t;t) m AB; AB0 cng chiu nn t > 0.
v hai im A(1; 1; 2), B(1; 4; 1). Lp phng trnh ng thng ()
1
AB0 AC0 1
S 0 0
qua B nm trong (P) sao cho cch im A mt khong cch ln nht.
.
=
Tng t u > 0 AB C = nn
SABC
2
AB AC
2
AB0 AC0 1
t u 1
Suy ra
.
= . = tu = 4
Li gii
4 2 2
AB AC0  2


0
0
5
5
Mt khc MB = t ;t 5 ; MC = u ; 2u 5
2



2
Cu VIIa. (1 im)
5
5

Suy ra t
(2u 5) = (t 5) u
5t 20u + 6tu = 0
Cho hai s phc u, v tha mn |u| = |v| = 10 v |3u 4v| = 2010. Tnh
2
2

M = |4u + 3v|.
tu = 4
3 + 34
2
Gii h
ta c 5u 6u 5 = 0 u =
t = 20u 24
5
Li gii
5
!

0
Ta c |z|2 = z.z
2 34 19 2 34 31
Suy ra MC =
;
10
5
Do t N = |3u4v| N 2 = (3u4v)(3u4v) = 9|u|2 +16|v|2 12(uv+vu)

t
a





3
Phng trnh d l 2 2 34 31 x
+ (2 34 19)(y 6) = 0
2
Cu VIb.2 (1 im)

n
v
.
h

x1
=
Trong khng gian vi h ta Oxyz cho hai ng thng (d1 ) :
2
x
y2
z
y z+1
=
v (d2 ) :
=
= . Lp phng trnh ng thng ()
1
1
3
1
2
qua im M(1; 1; 0) v to vi (d1 ); (d2 ) mt tam gic cn c y nm
trn ().
Li gii
Cu VIIb. (1 im)
m(x + 1)2
Cho hm s y =
c th l (Cm ), vi m l tham s khc 0. C
x2
tn ti hay khng mt im M sao cho (Cm ) va t cc i ti M, va t
cc tiu ti M vi hai gi tr khc nhau ca tham s m ? V sao ?.
Li gii
K: x 6= 2

t
a

m(x 5)(x + 1)
(x 2)2
V bng bin thin hs. T bng suy ra
Nu m > 0 th C l (1; 0) v CT l (5; 12m)
Nu m < 0 th CT l (1; 0) v C l (5; 12m)
Nh vy im M(1; 0) tho mn ycbt

R rng th m 6= 0, khi f 0 (x) =

LI GII THI TH S 24 CA MATH.VN

n
v
.
h

Cu I.1

(1 im)
x2 (x 6)
+ 4.
Cho hm s y =
8
Kho st v v th hm s cho.
Li gii
th hm s y =

x2 (x 6)
+ 4.
8

6
5
4
3
2
1

0
1

m
2

t
a
5

Cu I.2 (1 im)
Tm cc im trn th sao cho tip tuyn ti ct v to vi th mt
27
min phng c din tch bng .
2
Li gii

Gi M(m; y(m)) l im thuc th (C) cn tm. Ta c y0 =

3x2 12x
8

3m2 12m
(x m) + y(m)
8
Honh giao im ca (C) v (d) l nghim PT
3m2 12m
(x m) + y(m) = y(x)
8
2
(x m) (x + 2m 6)

= 0 x = m hay x = 2m + 6
8
? TH1: m < 2m + 6 m < 2
Z 2m+6
(x m)2 (x + 2m 6)
S1 =
dx
8
m
Z

1 2m+6 
=
(x m)3 + (3m 6)(x m)2 dx
8 m
(x m)4 2m+6 (m 2)(x m)3 2m+6 27(m 2)4
=

=


32
8
32
m
m
T tm c m = 0
? TH2: m > 2m + 6 m > 2
Z m
27(m 2)4
(x m)2 (x + 2m 6)
S2 =
dx = S1 =
8
32
2m+6
T tm c m = 4
Vy c 2 im M tho mn c honh l m = 0, m = 4 tc 2 im cc tr

Suy ra tip tuyn th ti M l (d) : y =

Cu II.1 (1 im)
Gii phng trnh:

3
2 2 cos 4x. sin(2x ) cos 8x
1
4
= 2 sin 2x +
cos 2x
cos 2x

Li gii
iu kin cos 2x 6= 0. Ta c:


3
2 2 cos 4x. sin 2x
cos 8x
1
4
= 2 sin 2x +
cos
cos 2x
 2x


3
2 2 cos 4x. sin 2x
cos 8x = 2 sin 2x cos 2x + 1
4
2 cos 4x (sin 2x + cos 2x) = cos 8x + 2 sin 2x cos 2x + 1 = 2cos2 4x + 2 sin 2x cos 2x
(cos 4x + cos 2x) (sin 2x + cos 4x) = 0
1

Li gii

Z ln 8
Z ln 8 

x.ex
x.ex
x +1 =

dx
=
2
dx
=
2
xd
I=
e
x
ln 3 2 ex +1
 ln 3

ln 3 e + 1
Z ln 8
Z
ln
8
ln
8


x
x
x
e + 1dx = 2 3 ln 8 2 ln 3
e + 1dx
= 2 x e + 1
Z ln 8

Cu II.2 (1 im)
Gii bt phng trnh:

4 17x + 53 12x < (2 x + 5 + 1)2 + 27.

n
v
.
h
ln 3

Z ln 8

Li gii

ln 3

ln 3

Tnh I1 =
ex + 1dx.
53
ln
3
iu kin: x

2udu
ex
17

x + 1 th du =
dx.
Suy
ra
dx
=
.
e
t
u
=
Ta c: 4 17x + 53 12x < 4(x + 5) + 1 + 4 x + 5 + 27
2 1
u
2 ex + 1

Z 3 2
Z 3
Z 3


4.16(x + 3)
u du
1
1

16x + 48 > 4 17x + 53 x + 5 16(x + 3) >


Do : I1 = 2
=2

du =
du +
17x + 53 + x + 5
2


u1 u+1
2 u 1
2
2
4
3
3

16(x + 3) 1
>0
(1) = 2 + ln(u 1)|2 ln(u + 1)|2 = 2 ln 2 + ln 3
17x + 53 + x + 5
Vy I = 2 (10 ln 2 3 ln 3 2)

53
Nu x < 3
17
p
Cu IV. (1 im)
4

(1) 1 <
(9x + 21) > (17x + 53) (x + 5)
Cho hnh tr c bn knh y bng R v trc OI = h (h < R). Mt phng
17x + 53 + x + 5

x < 21
(P) ty qua O, to vi mt phng y gc 45o , ct hai y theo cc dy
21

x<
9
AB,CD (AB qua O).
(9x + 21)2 > (17x + 53) (x + 5)
9
Chng minh rng rng th tch khi IABCD c gi tr khng i.
53
Kt hp vi khong ang xt: x < 3
17
4
Li gii

Nu x > 3
(1) 1 >
17x + 53 + x + 5
p
K hiu cc im nh hnh v. Ta c: IHCD. Li c OImp(ICD) OICD.
(9x + 21) < (17x + 53) (x + 5)
(2)
Suy ra: CDmp(IOH) CDOH. Suy ra ABCD l hnh thang cn v gc gia
21
Nu x th (2) ng.
d = 45 , suy ra tam gic OHI vung cn ti I.
mt phng (P) v mp(ICD) l OHI
9
21
11
Theo nh l Pytago ta tnh c:
Nu 3 < x < th (2) < x < 1.

9
4
h
OH = h 2, OK = ,CD = 2CH = 2 R2 h2
11
21
Kt hp vi khong ang xt ta c < x < .
2


 4
 9

1
1 2
53
11
Vy: VI.ABCD = IKSABCD = h R + R2 h2

Vy nghim ca bt phng trnh l , 3 , +

3
3
17
4
Cu III.

(1 im)

Z ln 8

Tnh tch phn

I=

ln 3

t
a

Cu V. (1 im)
1
Chng minh rng vi a < 1 bt phng trnh x. sin > a lun c 2010
x
nghim nguyn phn bit.

x.ex

dx.
ex + 1


3x 5
1
x = 1

= 1
1
1  
2
suy
ra

ta
c
h

x
+
x
Khi n N th = 0;
.
1
2

x
=
15
=
8
n
2
2
2
Gi M l im cui ca trn vng trn n v, T l giao im tip tuyn ti
A (1; 0) ca ng trn n v vi ng thng OM.
Cu VIa.2 (1 im)
Ta c din tch tam gic OAT ln hn din tch hnh qut OAM do vy: tan >
Trong khng gian vi h ta cc vung gc Oxyz cho hnh chp t
sin
tc
> cos .
gic u S.ABCD vi S(1; 2; 1), B(3; 7; 1), D(1; 5; 5). Gi I l tm mt

By gi xt hai trng hp:


cu ngoi tip hnh chp S.ABCD.
1
Lp phng trnh mt phng qua BI v song song vi AC.
? Nu a 0 th v vi mi n N ta c n sin > 0 do mi s nguyn dng
n
u l nghim
 ca bt phng trnh ni n trong . ? Nu 0 < a < 1 khi tn
Li gii
ti 0;
sao cho a = cos , v Z+ l tp khng b chn nn tn ti m sao
2
 
Gi G l tm ca hnh vung nn G(1; 6; 3) v phng trnh mt cn tm l

1
1
1
> a.
cho m > tc l ta c > > > 0 ni khc i cos
(P) : ax + by + cz + 3a 7b c = 0 (i qua B)

2
m
m

 
D dng tnh BD = (4; 2; 4), SB = (4; 5; 2), SG = (2; 4; 4) v BG = 3, SB =

1
1
1

Vi mi n m ta c > > > > 0 nn cng dn n cos


> a.
3 5, SG = 6
2
m n
n


By gi theo mnh c khng
Do AC BD, AC SB nn AC cng phng vi [BD, SB] = 12(2; 2; 1) V
  nh trn
 kia
 ta s c:
1
1
ACk(P) nn 2a + 2b = c
n sin
> cos
> a.
n
n
Gi l gc gia mt phng (P) v mt phng (ABCD)
Tc [m; +) Z+ thuc vo tp nghim ca bt phng trnh ni n trong .
d = BIG
d = 2BSG
d
Ta suy ra = IBG
Hai kh nng xt cho ta iu phi chng minh

d cos BSG
d = 2 BG SG = 4
Khi cos = 2 sin BSG
SB SB 5
Cu VIa.1 (1 im)
4
|a 2b 2c|
Suy ra
=
5 3 a2 + b2 + c2
Trong mt phng vi h ta Oxy cho tam gic ABC c A (3; 4) , B Oy.
Kt hp vi trn ta c phng trnh (11a + 10b)(5a 2b) = 0
ng phn gic trong ca gc C c phng trnh (d) : x + 3y + 5 = 0 v
Chn a = 2, b = 5 c c = 14 v c PT (P) : 2x + 5y + 14z 43 = 0

d (B; d) = 2d (A; d) . Tm ta cc nh cn li ca tam gic ABC
Li gii

n
v
.
h

t
a

Li gii
Gi A0 l im i xng ca A qua d suy ra A0 thuc BC
D dng tm c A0 = (1; 8)
Li c d (B; d) = 2d (A; d) .
Nn BC = 2AC hay A0 l trung im ca BC
Gi C(3x1 5; x1 ), B(0, x2 )

Cu VIIa. (1 im)
Cho s phc z tha mn iu kin |z| = 1. Tm gi tr ln nht, gi tr nh
nht ca A = |1 + z| + 3|1 z|.
Li gii
t z = a + bi th a2 + b2 = 1, 1 + z = 1 + a + bi, 1 z = 1 a bi
p
p



Ta c: A = (1 + a)2 + b2 + 3 (1 a)2 + b2 = 2 1 + a + 3 1 a

Xt hm f (a) = 1 + a +
3 1 a vi a [1, 1]
1a3 1+a
4
Ta c: f 0 (a) = p
.
f 0 (a) = 0 a = .
5
2 (1 + a)(1
a) 

4
T ta suy ra max f (a) = f
= 2 10,
5
min f (a) = min{ f (1), f (1)} = 2
Cu VIb.1 (1 im)
Trong mt phng Oxyz cho hnh ch nht ABCD c din tch bng 34;
M(6; 1) l trung im cnh BC. ng thng : 15x + 8y 48 = 0 i
qua tm ca hnh ch nht v ct ng thng AD ti mt im thuc Oy.
Xc nh ta cc nh ca hnh ch nht.
Li gii.
T gi thit suy ra N(0, 6) AD. Gi I l tm ca hnh ch nht ABCD.
Gi H l im i xng ca N qua I thH thuc BC,
48 15a
H thuc nn ta im H c dng a,
.
8


a 96 15a
,
.
Suy ra ta im I l
2
16


M l trung im BC nn MI vung gc vi MH hay MI.
MH = 0
a

a=8
(112 15a) (56 15a)

6 (a 6) +
=0
80
2
16.8
a=
17
T da vo gi thit din tch ABCD ta d dng tm c ta cc nh

Mt cu c tm O = (1; 2; 3), R = 6 Gi (I, r) l ng trn giao tuyn ca P v

(O). Khong cch O n P l 6 suy ra bn knh r = 30 ng thng ct S


ti A, B v thuc (P) nn ct (I) ti
r dy AB
r
AB2
321

2
=
suy ra khong cch I n l d = r
IM = (1; 1; 9)
4
11
r


d
321
=
suy ra IM = 83. Gi l gc gia v IM suy ra sin =
IM
913
r
592

suy ra cos =
. Gi
u = (a; b; c) l VTCP ca suy ra 2a + b + c = 0,
913
r
|a + b 9c|
592
=
da vo ta c pt
913
83 a2 + b2 + c2
2
Thay b vo pt ta c 11(a + 10c) = 592(a2 + c2 + (2a + c)2 )


n
v
.
h
Cu VIIb. (1 im)

t
a

Cu VIb.2 (1 im)
Trong khng gian Oxyz cho mt cu
(S) : (x 1)2 + (y 2)2 + (z 3)2 = 36 v (P) : 2x + y + z 1 = 0.
Vit phng trnh ng thng i qua M(2; 3; 6), nmtrong mt phng
36 11
.
(P) v ct mt cu (S) ti hai im A, B sao cho AB =
11
Li gii

Vit phng trnh tip tuyn ca th hm s y =

x2 x + 1
,
x1

bit tip tuyn i qua M(5; 5)


Li gii



xo2 xo + 1
Tip tuyn ca t hs ti im A xo ;
l:
x

1
o


1
xo2 xo + 1
y = 1
(x

x
)
+
o
(xo 1)2
x0

1
xo2 xo + 1
1
(5 xo ) +
Tip tuyn ny i qua M(5; 5) 5 = 1
(xo 1)2
xo 1
3
5
xo = 3 suy ra pttt l y = x +

4
4

You might also like